Ευκλειδης Β 70

84
Henri Cartan (1904-2008) Θέματα "ΘΑΛΗΣ 2008"

description

Ευκλειδης Β 70

Transcript of Ευκλειδης Β 70

Page 1: Ευκλειδης Β 70

• Henri Cartan (1904-2008)

• Θέματα "ΘΑΛΗΣ 2008"

Page 2: Ευκλειδης Β 70

ΕΛΛΗΝΙΚΗ ΜΑΘΗΜΑΤΙΚΗ ETAIPEIA Τεύχοc; 70 • Οκτώβριος • Νοέμβριος • Δεκέμβριος 2008· Έτ οc; λη' - Ευρώ: 3,50

e-mail: [email protected] www.hms.gr

ΜΑΘΗΜΑΤΙΚΟ ΠΕΡΙΟΔΙΚΟ ΓΙΑ ΤΟ ΛΥΚΕΙΟ

ΠΕΡΙΕΧΟΜΕΝΑ -1' Elie, Henri Cartan και η Ελλάδα

-1' Μαθηματικοί Διαγωνισμοί

-1' Homo Mathematicus

ΜαθΙJΗατικά Α' Τάξης -1' Άλγεβρα: Δευτερομαθμιες εξισώσεις με πραγματικούς

4

16

συντελεστές 20 -1' Γεωμετρία:Παραλληλόγραμμα- Τραπέζια 27

Μαθηματικά Β' Τάξης -1' Άλγεβρα: Πολυώνυμα 33 -1' Γεωμετρία: Μετρικές σχέσεις - Εμβαδά 40 -1' Κατεύθυνση: Εύρεση πρωτευόντων στοιχείων τριγώνων

από δευτερεύοντα στοιχεία του 44

Μαθηματικά Γ' Τάξης -1' Μαθηματικά Γενικής Παιδείας: Στατιστική 47 -1' Μαθηματικά Κατεύθυνσης: Γενικές ασκήσεις 51 -1' Με αφορμή μια άσκηση του σχολικού βιβλίου 58

• • • • • • • • • • • • • • • • • • • • • • -1' Το Βήμα του Ευκλείδη -1' Ο Ευκλείδης προτείνει ... -1' Τα Μαθηματικά μας Διασκεδάζουν

60 72 77

Γράμμα της Σύνταξης Αγαπητοί μαθητές και αuνάδελφοι,

Με αφορμή τα Χραιατούγεννα και το νέο έτος εuχό·

μαατε αε όλοuς Χρόνια Πολλά και εuτuχιαμένα.

Στοuς μαθητές να πετύχοuν όλα αuτά για τα οποία α-

γωνίζονται και ατοuς δαακάλοuς τοuς αντοχή για αuμπα-

ράατααη και κατανόηαη

Με αuναδελφικούς χαιρετιαμούς

Ο Πρόεδρος της Συντακτικής Επιτροπής Γ.Σ. Ταααόποuλος

Ο Αντιπρόεδρος της Συντακτικής Επιτροπής Β. Εuαταθίοu

Σύνθεση εξωφύλλου:

Μια νέα απόδειξη του Πυθαγορείου Θεωρήματος που περιέχει όλα τα γράμματα της Αλφαβήτου

Να αποδειχθεί: l/4(AB2+ ΑΓ2) + l/4Br2 • Το σχήμα της άσκησης είναι στη σύνθεση του εξωφύλλου (Γ. Μενδωνίδης) Υπόδειξη ·Λύση: (Σε σύνtομη αναφορά)

( ..• ) Φέρνουμε τις ημι.ευθείες ΒΟ και ΓΟ ώσrε: fBO=BΓ0=4SO και κατασκειιάζοιιμε τα ΔΕΖΗ και ΘΙΚΛ (._)Το ΞΕSΛ είναι

ρόμβος και τα ΕΙΙΡ, ΡΣS, ΛΣΤ, ΞΙΠκαι ΑΜΝ είναι ίσα τότε το ΙΙΡΣΤ είναι ρόμβος ΞΙΠ + ΕΙΙΡ=Β+ Γ=9()0 οπότε ο ρόμβος είναι τετράγωνο( ... ) Κατασκευάζουμε το ορθοyώνιο ΡΦΣS και διαπιστώνουμε ότι ΕΊ =Et, Ε'2 =E:z, Ε'3 =Ε-.3, Ε'4 =Ε4, οπότε

με πρόσθεση κατά μέλη προκύπτει (ΔΕΖΗ)/4+(ΘΙΚΛ)/4=(ΙΙΡΣτ) επομένως 1/4(ΑΒ2+ΑΓ2)+ l/4Br2 .

• • • • • • • • • • • • • • • • • • • • • • • • • • • • • • • • • • • • • • • • • • • • • • • • • • • • • • • • • • • • • • • • • • • • • • • • • • • • • •

ΕΚΔΟΣΗ ΤΗΣ ΜΑΘΗΜΑ1ΙΚΗΣ ΕJΑΙΡΕΙΑΣ ΠΑΝΕΠΙΣΤΗΜΙΟΥ 34 106 79 ΑΘΗΝΑ 1'η.\: 210 3617784 - 3616532 Fax: 2103641025

Εκτελεστική Γραμματεία

Πρόεδρος: Τασσ6πουλος Γιώργος

Εκδότης: Αντιπρόεδρος: Αλεξανδρής Νικόλαος Ευσταθίου Βαγγέλης

Διευθυντής: Τυρλής Ιωάννης

Επιμέλεια Έκδοσης: Τασσ6πουλος Γιώργος

Ευσταθίου Βαγγέλης

Γραμματέας: Χριστ6nουλος Πανσγιώτης Μέλη: Αργυρδκης Δ.

Δρούτσας π. Λουρίδας Σ. Ταπειν6ς Ν.

Κωδικός ΕΛ. ΤΑ.: 2055 ISSN: 1105- 7998

Αθανασ6πουλοc;Γεώ�οc; Αναστασίου Γιάννης Αvδρουλακάκηc; Νίκοc; Αvτωv6πουλοc; Νίκοc; Αρyυράκηc; Δημήτριος Βακαλ6πουλοc; Κώστας Δρούτσαc; Παvαyιώτηc; Ευσταθίου ΒαyyεΆηc; Ζαχαρ6πουλοc; Κων/νοc; Ζώτοc; ΒαyyεΆηc; Κακκαβάc; Απ6οτολοc; Καλίκαc; Σταμάτης Κανέλλοc; Χρήστος Καραyκούνηc; Δημήτρης Καρακατσάνηc; Βασfληc; Καρκάvηc; Βασίλης Κατσούληc; Γιώρyοc; Κερασαρίδηc; Γιάννης Καρδαμίτσηc; Σπύροc; Κηπουρ6c; Χρήστος Κλάδη Κατερίνα

Συντακτική επιτροπή Κόvτζιαc; Νίκοc; Κοτσιφάκηc; Γιώρyοc; Κουτρουμπέλαc; Κώσταc; Κυριαζής Ιωάννης Κυριακ6πουλοc; Αντώνης Κυριακ6πουλοc; Θανάσης Κυβερνήτου Χρυστ. Λαζαρίδης Χρήστος Λάππαc; Λευτέρης Λουρίδας Σωτήρης Μαλαφέκαc; Θανάσης Μαvωλάκου Στοματική Μαυρσyιαννάκηc; Λεωνίδας Μενδριvόc; Γιάννης Μεταξάς Νικόλαος Μπρίνοc; Παναyιώτηc; ΜυΑωνάc; Δημήτρης Μώκοc; Χρήστος Παvουσάκηc; Νίκοc; Ρέyκληc; Δημήτρης

Σαίτη Εύα

Σταϊκοc; Κώσταc;

Στάϊκοc; Παvαyιώτηc;

Στρατής Γιάννης

Ταπεινός Νικόλαος

Τασσότrουλοc; Γιώρyοc;

Τζιώτζιοc; Θανάσης

Τριάvτοc; Γεώρyιοc;

Τσαyκάρηc; Ανδρέας

Τσατούραc; Ευάyyελοc;

Τσικαλουδάκηc; Γιώρyοc;

Τσιούμαc; Θανάσης

Τυρλήc; Ιωάννης

Φανέλη Άννυ

Χαραλαμποτrούλου Λίνα

Χαραλάμπους Θάvοc;

Χριστιάc; Σπύροc;

Χριστ6πουλοc; Παvαyιώτηc;

Ψύχαc; Βαyyέληc; ••••••••••••••• •••••••••••••••••••••••••••••••••••••••••••••••••••••••••••• •••

• Τα διαφημιζ6μενα βιβλία δε σημαίνει ότι προτείνοvται mιό την Ε.Μ.Ε. • Οι συνερyάτες, τα άρθρα, οι προτεινόμενες ασκήσεις, οι λύσεις ασκήσεων κτλ. πρέπει να στέΑvοvται έyκαιρα, σrα yραφεία της Ε.Μ.Ε.

με την ένδειξη "Για τον ΕυκΑεlδη β'", Τα χειρόγραφα δεν εmσrρέφονrαι. τιμή Τεύχους ευρώ 3,50 Ετήσια συνδρομή (12,00 + 2,00 Ταχυδρομικά = ευρώ 14,00) Ετήσια συνδρομή για Σχολεία ευρώ 12,00 Το αvτίπμο yια τα τειJχη που παραyyέΑvοvται σrε'Ανεται με απλή εmταyή σε διαταyή Ε.Μ.Ε. Τ αχ. Γραφείο Αθήνα 54 Τ.Θ. 30044 ή

πληρώνεται σrα γραφεία της Ε.Μ.Ε.

Εκτύπωση: ΙΝΤΕΡΠΡΕΣ Α.Ε. τηλ.: 210 8160330 Υnειίθuνος τunογpαφείοu: Β. Σωτηριάδης

Page 3: Ευκλειδης Β 70

ο

Elie Cartan (1869-1951) και Henri Cartan (1904-2008). Οι σχέσεις του,ς με την Ελλάδα.

Σ. Π. Ζερβός, ομότιμος καθηγητής Πανεπιστημίου Αθηνών

Γάλλος Elie Cartan υπήρξε ένας από τους μεγαλύτερους Γεωμέτρες (ιδιαιτέρως, δια­φορογεωμέτρες) όλων των εποχών. Ο πιό σημαντικός άμεσος συνεχιστής του θεμε­λιώδους έργου του Νορβηγού

Sophus L ie (συνεχείς ομάδες μετασχημα­τισμών, που καλούνται τώρα, "ομάδες L ie") . Σε πολλά άλλα, βασικά, πρωταγω­νιστής. Με σημαντική συμβολή και στην Γενική Σχετικότητα.

Το 1914 ο Ε. Cartan παρουσίασε μιά απ' τις πιό μεγαλοφάνταστες και τολμηρές θεωρίες του. Αρχίζει έτσι: "Σε μια πρό­σφατη Ανακοίνωση, ο κ. Ζερβός γενίκευσε ένα θεώρημα του κ. Hilbert . . . " (Ο Hilbert υπήρξε ο μεγαλύτερος Γερμανός μαθημα­τικός από τα τέλη του 1 9ου αιώνα ως τώ­ρα.) Ο Έλληνας μαθηματικός Παναγιώτης Ζερβός (Κεφαλονιά, 1 878 - Αθήνα, 1 952. Καθηγητής στο ΠΑ: 1 9 1 8- 1 920 και 1 923-1 949) συνέχισε να εργάζεται ερευνητικά στο ίδιο γενικό αντικείμενο. Σχετικά, στο κλασικό σύγ­γραμμα του "Linear Analysis and Representation Theory" (Spήnger, 1 973), ο Gaal γράφει: Τα κύ­ρια στοιχεία (main items) της κλασικής βιβλιογραφίας γιά τις ομάδες Lie είναι τα έργα των Sophus Lie και Elie Cartan, ένας τόμος από τον Kowalefski και το βιβλίο "Το πρόβλημα του Monge" από τον Π. Ζερβό στην (γαλλική) σειρά (μονογραφιών) Memorial des Sciences Mathematiques ( 1 930). Οι διάφοροι τόμοι και εκδόσεις ενός βιβλίου του Cheνalley είναι πιό πρόσφατοι (Claude Cheνal­ley , μεγάλος μαθηματικός) . Μιά πλατειά και σε βάθος θεώρηση των παραπάνω θεμάτων βρίσκε­ται στην ομιλία του Π. Ζερβού στο Α ' Διαβαλκανικό Συνέδριο των μαθηματικών (Αθήνα, 1 934). Έχει περιληφθεί και στο βιβλίο "Πως μεταβαίνουμε από τα κλασικά μαθηματικά στα νεότε­

ρα", από εμέ και τον Πέτρο Β . Κρικέλη . Δεύτερη έκδοση, επαυξημένη, 2007 (ΖΚ). Το 1928-1930 ο Ε. Cartan δίδασκε, στο Παρίσι, Μιγαδική προβολική γεωμετρία (σημα­

ντική και για την Νεότερη Φυσική) . Μαθητής και, φυσικά, θαυμαστής του ο Κωνσταντίνος Πα­παϊωάννου ( 1 899- 1 979. Από το 1 93 1 καθηγητής στο ΠΑ και από το 1 950 και στο ΕΜΠ. Με κλασικά συγγράμματα Μηχανικής και Θερμοδυναμικής), θα το διδάξει, ως Ελεύθερο Μάθημα, στο ΠΑ, το Α' εξάμηνο του 1 93 8- 1 939 . Το κείμενο του βρίσκεται στο ΖΚ. Συντομογραφία: Henri = Henri Cartan.

Μια κορυφαία υπηρεσία του Ε. Cartan στα Μαθηματικά υπήρξε η απ' αυτόν "κάλυψη" της

ΕΥΚΛΕΙΔΗΣ Β' 70 τ.2/1

Page 4: Ευκλειδης Β 70

-------------- Μαθηματικές Αναφορές -------------­

γύρω απ' τον γιο του Henή παρέας από ικανότατους νέους Γάλλους μαθηματικούς που έθεσαν ένα γενικό σκοπό διαφορετικό από τον δικό του : Εμπνεόμενοι από τα "Στοιχεία Γεωμετρίας" του Ευ­κλείδη να οικοδομήσουν, με συλλογική εργασία, ένα ανά"λ.ογο έργο, "Στοιχεία Γεωμετρίας", για τα σύγχρονα Καθαρά Μαθηματικά. Από τυχαίους λόγους διαλέξανε για τον "συγγραφέα" το όνομα του γενναίου Ελληνογάλλου στρατηγού του 1 870- 1 87 1 Βούρβαχη (Bourbaki) . Το έργο, μνημειώ­δες.

Η αντιφώνηση του Elie Cartan στο Ιωβtλαίο του (γεγονός πα­γκόσμιο), το 1 939, αρχίζει συγκινητικά. Θυμάται τους γονείς του, ο­ρεινούς γεωργοκτηνοτρόφους, και τους δασκά"λ.ους του στο Δημοτι­κό· την εξαίρετη μαθηματικό αδελφή του, που πέθανε πρόωρα· τον τόσο ταλαντούχο μουσικοσυνθέτη γιο του Jean Cartan, που πέθανε μόλις 25 χρόνων.

Η μουσική στους Cartan: Ο Henri, λαμπρός πιανίστας, και ο εξίσου μ' αυτόν προικισμένος στα Μαθηματικά φίλος του Jean Dieudonne , λαμπρός βιολιστής, οικιακό ντουέτο.

Αρχαία Ελλάδα και Bourbaki: Κορυφαίος μαθηματικός στην αρχική παρέα ο Andre Weil, θεωρεί ότι η απ' αυτόν, νεαρό, ανά­γνωση της αρχαίας ελληνικής ποίησης στο πρωτότυπο τον ώθησε να βάλει υψηλά κριτήρια στη ζωή του. Από Henri και Weil η πρώ­

τη σκέψη για την συλλογική προσπάθεια. (Η ηρωική και, μεταθανάτια, διάσημη συγγραφέας Σι­μόνη Weil, αδελφή του Andre . Από εύπορους Γαλλοεβραίους γονείς, αρχικά αναρχίζουσα εθε­λόντρια αδελφή στους Δημοκρατικούς, στον Ισπανικό Εμφύλιο, θα καταλήξει μαχόμενη Χρι­στιανή .)

1940. Έτος στρατιωτικής ήττας της Γαλλίας· και, σε λίγους μήνες, παγκόσμιας δόξας της Ελλάδας λόγω του Αλβανικού Έπους. Παράλληλα και ανεξάρτητα, εμφανίζεται το πρώτο τεύ­χος του Bourbaki, με αρχαία ελληνική μετώπη στο εξώφυλλο . Θα διεκδικεί, στον Πρόλογο, την ανανεωμένη πνευματική συνέχιση του έργου του Ευκλείδη .

1943. Ο Louis Cartan, φυσικός, γιος του E.Cartan, μετέχει ενεργά στην Αντίσταση . Συλ­λαμβάνεται από τους Ναζί και εκτελείται.

1945-1946. Σε αυστηρή συμμαχική στρατιωτική κατοχή , οι πανεπιστημιακοί μαθηματικοί της Γερμανίας περνάνε δύσκολα. Το 1 946 ο Henri πρωτοστατεί στην προσπάθεια για ανασυ­γκρότηση των γερμανικών μαθηματικών. Μερικές δεκαετίες αργότερα, "πλήρης" σε παγκόσμιες τιμές για το ερευνητικό έργο του, πρωταγωνιστεί στην ίδρυση ευμι•r:τ••���ίίJ� πανεπιστημιακιον Οατμ

Έρευνα· ενδεικτικό : Φλέγον πρόβλημα της γενιάς του, η γενίκευση της έννοιας του ορίου ακολουθίας, με τρόπο που να καλύπτεται και η περίπτωση των μη μετρικών τοπολογικών χώ­ρων. Με τους ορισμούς του φiλτρ�J:υ και του υπερφi_λτμου και της αντίστοιχης θεωρίας δίνει ο Henri μια "κρυστάλλινη" και οριστική λύση . Τελικά, τα φίλτρα και τα υπερφίλτρα θα παίξουν γόνιμο ρόλο και στην Άλγεβρα και την Μαθηματική Λογική .

Θα επιδιώκει, πάντα, ο Henri, την απόλυτη διαύγεια και το ξεκαθάρισμα του "τι οφείλεται σε τι" (ευκλείδεια παράδοση) . Αυτά, και στα τέσσερα κύρια εκτεταμένα πεδία έρευνάς του: α\

Αναλυτικές συναρτήσεις μίας ή περισσοτέρων μιγαδικών μεταβλητών. Συναφή τοπολο-:·χ�: .:.c:_ ματα. Αυτόμορφες συναρτήσεις. β) Αναλυτικοί χώροι. γ) Αλγεβρική τοπολογία. δι 0_:··.:·-.-.�

ΕΥΚΛΕΙΔΗΣ Β' 70 τ.2/2

Page 5: Ευκλειδης Β 70

------------- Μαθηματικές Αναφορές -------------

άλγεβρα. Στο α) συνεργάστηκε με τον Κωνσταντίνο Καραθεοδωρή , τον οποίο τιμούσε ιδιαίτερα. Δύο

συγγράμματα Μιγαδικής Αναλύσεως, εξαίρετα αλλά όσο γίνεται διαφορετικά στο πνεύμα, είναι το δίτομο "Θεωρία Συναρτήσεων" του Καραθεοδωρή και το "Στοιχειώδης θεωρία των ανα­

λυτικών συναρτήσεων μίας ή περισσοτέρων μιγαδικών μεταβλητών" (στα γαλλικά) του Henri . Στο δ) έχουμε το κλασικό σύγγραμμα "Ομολογική άλγεβρα" από τον Henri και τον με­γάλο Πολωνοεβραίο τοπολόγο Samuel Eilenberg.

Το Σεμινάριο Henri Cartan και, αργότερα, το Σεμινάριο Cartan-Cheνalley , στο Παρίσι, έ­παιξαν μεγάλο ρόλο στην ανάπτυξη των διεθνών Μαθηματικών. (Ο Cheνalley , απ' την αρχική παρέα.) Λαμπρές "πολυγραφημένες" εκδόσεις τους στο Inst itut Henri Poincan�· όπου και οι τό­μοι του διεθνώς πρωτοποριακού "Σεμιναρίου" Bourbaki .

Δεν είναι εδώ η θέση ν' αναφερθούμε σε Μαθητές, κάποτε ακόμα μεγαλύτερους απ' τους Δασκά­λους, που βγήκαν απ' αυτό τον χώρο. Ο μέγιστος, ο Alexander Grothendieck ( 1 928-) θα είναι ένας από τους μεγαλύτερους "μαθηματικούς στρατηλάτες" ό'λmν των εποχών. (τους θυμάμαι, το 1 956- 1 958, να περπατούν, αυτός και ο Henri, σε πολύκοσμο πεζοδρόμιο , συζητώντας αδιάκοπα μαθηματικά.)

Στο Συμπόσιο που έγινε στο ΠΑ το 1 983 προς τιμήν του μεγάλου μαθηματικού Marc Κrasner (Οδησσός, 1 9 1 2-Παρίσι, 1 985) επίτιμοι πρόεδροι ήταν ο Henri και ο μεγάλος Αμερικα­νός μαθηματικός Saund ers Mac Lane. Η ως προς τις γαλλικές συμμετοχές επιτυχία του Συμπο­σίου οφείλεται προπαντός στον Σπύρο Σπαθή, εξαίρετο Έλληνα μαθηματικό στο Πανεπιστήμιο Παρίσι VI .

Η ομιλία μου στο ΠΑ, κατά την αναγόρευση του Henri ως Επίτιμου διδάκτορα του, συ­μπληρωμένη, εγκρίθηκε απ' αυτόν και είναι, από πολλά χρόνια, έτοιμη για δημοσίευση . Θα πε­ριέχεται στην δεύτερη, ουσιωδώς επαυξημένη, έκδοση του τόμου μου ""Mathemat ics reνisited , mathemat icians remembered· · ( 1 99 1 ), που θα γίνει, αν θέλει ο Θεός, το 2009. Το μακρύ άρθρο μου για τον Marc Κrasner σ' εκείνο τον τόμο περιείχε και αναφορές στον Henri .

Με μόνιμο ενδιαφέρον για την, σε υψωμένο επίπεδο, διδασκαλία των βασικών πανεπιστη­μιακών Μαθηματικών, ο Henri έγραψε ένα θαυμάσιο συνοπτικό "Διαφορικό Λογισμό" σε χώ­ρους Banach (στα γαλλικά) .

Ανάμνηση: Μου έγραφε, ο Henri : 'Ή φιλία των πατεράδων μας επέρασε στους γιούς τους." (Ο προαναφερθείς Π. Ζερβός, πατέρας μου .)

Ξαναβλέπω, νοερά, την πρώτη τάξη της Ecole Normale Supeήeure, το 1 955- 1 956 . Ο Henri διδάσκει Αλγεβρική το­πολογία. Μαθητές από εξετά­σεις: Adήen Douady (άμεσος χαρακτηρισμός μου: Μικρός

Poincare) , Paul-Andre Meyer (μελλοντικός κορυφαίος στο τρίδυμο : Δυναμικό, Μέτρο, Πιθανότη­τες. Επιστημονικό "alter ego" του μεγάλου Laurent Schwartz). Τώρα, βιολογικά, Απόντες· κοντά στον τότε Δάσκαλό τους.

ΕΥΚΛΕΙΔΗΣ Β' 70 τ.2/3

Page 6: Ευκλειδης Β 70

Μαθ,qμστtκο1 A1ayωvtoμoi. Μ θ Ι ο· λ. LI:.-Q'- - . - · α····κε·- ' - . _ ··ntιιυδ·-. ' • · - ····· , ' , ... qμ .. . . . 11. . . . . . . . . .. :!; ..... . . ... . . . 1J'P ' : ' • • ' ..... · .!;

Επιτροπή Διαγωνισμών της Ε.Μ.Ε.

11" ΜΕΣΟΓΕΙΑΚΗ ΜΑΘΗΜΑΤΙΚΗ OnYMΠIAQA 2008 εις μνήμην PETER Ο' HOLLORAN

Πρόβλημα 1 Έστω c Α ο παρεγγεγραμμένος κύκλος του τριγώνου ABC με κέντρο ι. ο οποίος εφάπτεται της

πλευράς BC στο σημείο D και στις προεκτάσεις των πλευρών AC και ΑΒ στα σημεία Ε και Ζ, αντίστοιχα. Αν η ευθεία ED είναι κάθετη προς την πλευρά ΑΒ (την οποία τέμνει στο σημείο Η) και η ευθεία ZF είναι η κάθετη από το Ζ προς την ευθεία ΗΙ. (την οποία τέμνει στο σημείο F), να

υπολογίσετε τις γωνίες BFD και AFE .

Λύση

Έστω Κ το σημείο τομής των ευθειών Ι. C και ED . Επειδή οι ευθείες CE και CD είναι

εφαπτόμενες από το C του κύκλου έπεται ότι I.C .l ED, οπότε το τετράπλευρο ΖΙ.ΚΗ έχει από τις

υποθέσεις τρεις γωνίες ορθές και είναι ορθογώνιο. Άρα είναι κι. 11= ΖΗ .

Από το ορθογώνιο τρίγωνο EC!a λαμβάνουμε: ( Ι.Ζ)2 = { Ι.Ε )2 = (I.C)(I.K) = (I.C) (ZH) � Ι.Ζ

= ΖΗ, α. I.z από την οποία προκύπτει ότι τα τρίγωνα ΗΖΙ. και ΖΙ. C είναι όμοια.

Επομένως, οι γωνίες zfn. και CZI. είναι ίσες, οπότε οι ευθείες ΗΙ. και ZC είναι κάθετες, δηλαδή

τα σημεία Ζ, F και C είναι συνευθειακά και

ΕΥΚΛΕΙΔΗΣ Β' 70 τ.2/4

Page 7: Ευκλειδης Β 70

--------- Μαθηματικοί Διαγωνισμοί- Μαθηματικές Ολυμπιάδες --------­

HBD=BCI. ο ( 1 ) Επιπλέον, επειδή το τετράπλευρο CDFI. είναι εγγράψιμο (αφού I}C = I.f>c ), έπεται ότι:

� �

BCI. = HFD. (2) Από τις ( 1 ) και (2) έπεται ότι Ηfω = HFD , οπότε το τετράπλευρο BHDF είναι εγγράψιμο και θα

έχουμε DFB = 1 80 °- BHD = 1 80 °- 90 ° = 90 ° . Επιπλέον, έχουμε ΒΗΙ. = Hi. C [ εντός εναλλάξ , ΑΖ 11 Ι. C ]

Hi.C=FEC [γιατί FI.EC εγγράψιμο] ,

(3)

(4) οπότε από τις (3) και (4) έπεται ότι ΒΗΙ. = FEC και το τετράπλευρο AHFE είναι εγγράψιμο .

Επομένως θα είναι: AFE =AHE = 90 ° ο

Π ρόβλημα 2 Α ν p και q είναι μη μηδενικοί ακέραιοι και η εξίσωση

y4 + 2(2p - 1) y2 + 8qy -( 4p - 1 ) = ο έχει μία ακέραια ρίζα ετερόσημη του q , να αποδείξετε ότι η εξίσωση

χ3 + px + q = 0 έχει μία τουλάχιστον άρρητη ρίζα.

Λί>ση Γράφουμε την εξίσωση ( 1 ) στη μορφή

(y2 - 1 )2 + 4p(y2 - 1 ) + 8qy = O

(1)

(2)

(3)

από την οποία εύκολα προκύπτει ότι οι αριθμοί ±1 δεν είναι ρίζες της εξίσωσης ( 1 ) . Πράγματι, αν οι αριθμοί y = ±1 ήταν ρίζες της ( 1 ) , τότε θα είχαμε q = Ο, άτοπο.

Επίσης από την (3) προκύπτει ότι, αν y είναι η ακέραια ρίζα της ( 1 ) , τότε πρέπει αυτή να είναι της

μορφής y = 2α + 1, α Ε Ζ , δηλαδή περιττός ακέραιος. Πράγματι, αν ήταν ρίζα της ( 1 ) ο άρτιος ακέραιος

y = 2α, α ΕΖ , τότε ο όρος ( y2 - 1 )2 θα ήταν περιττός, ενώ το άθροισμα 4 p { y2 -1 ) + 8qy είναι άρτιος,

οπότε δεν μπορεί να αληθεύει η εξίσωση (3). Επομένως, η ακέραια ρίζα της εξίσωσης ( 1 ) είναι της μορφής y = 2α + 1, α ΕΖ και ισχύει

2α2 (α+ 1/ + 2pα(α+ 1 ) + q (2α+ 1 ) = 0 . (4) Γ ράφουμε την εξίσωση (2) στη μορφή f (χ) = Ο, όπου f (χ ) = χ3 + px + q ,

οπότε θα είναι f (α) = α3 + pα+ q. Από την τελευταία, με πολλαπλασιασμό των δύο μελών επί 2 ( α+ 1 )2 προκύπτει ότι

2 (α+ 1 )2 f (α) = 2α3 (α+ 1 )2 + 2pα(α+ 1 )2 + 2q (α+ 1 )2 , η οποία λόγω της (4) γίνεται 2 ( α+ 1 )2 f ( α) = -2α2 (α+ 1 )2 + q ( α+ 1 ) . (5)

Επειδή οι αριθμοί 2α + 1 και q είναι από την υπόθεση ετερόσημοι, έπεται ότι και οι αριθμοί α + 1 και q είναι ετερόσημοι. , οπότε από την (5) προκύπτει ότι f ( α) < Ο .

Ομοίως λαμβάνουμε ότι 2α2 f ( α+ 1 ) = 2α2 (α+ 1/ - qα < Ο , από την οποία λαμβάνουμε τη σχέση f (α + 1 ) > Ο .

Επειδή η πολυωνυμική συνάρτηση f (χ ) = χ3 + px + q είναι συνεχής στο IR. , άρα και στο κλειστό

διάστημα [α, α+ 1] , και ισχύουν f (α) < Ο και f (α+ 1 ) > Ο , από το θεώρημα του Bolzano, έπεται ότι η

εξίσωση (2) έχει μία τουλάχιστον μία πραγματική ρίζα στο ανοικτό διάστημα (α, α + l ) με άκρα δύο

διαδοχικούς ακέραιους. Η ρίζα αυτή δεν μπορεί να είναι ρητός αριθμός, έστω της μορφής

ΕΥΚΛΕΙΔΗΣ Β' 70 τ.2/5

Page 8: Ευκλειδης Β 70

--------- Μαθηματικοί Διαγωνισμοί- Μαθηματικές Ολυμπιάδες ---------ω =

μ , (μ, v) = 1 , γιατί, αν ήταν, τότε σύμφωνα με γνωστό θεώρημα θα είχαμε ότι μ lq και vl l , οπότε v θα ήταν ω = ±μ Ε Ζ , άτοπο. Άρα η εξίσωση (2) έχει μία τουλάχιστον άρρητη ρίζα.

Π ρόβλημα 3 Η τετράδα θετικών ακέραιων ( a, b, c,d) λέγεται ((ενδιαφέρουσα;;, αν οι αριθμοί

a2 , b2 , c2 , d2 + 1 με τη σειρά που δίνονται, είναι διαδοχικοί όροι αύξουσας αριθμητικής προόδου.

α) Βρείτε μία τουλάχιστον «ενδιαφέρουσα» τετράδα. β) Ο αριθμός όλων των τετράδων που είναι «ενδιαφέρουσες>> είναι πεπερασμένος ή

άπειρος; Να εξηγήσετε την απάντησή σας.

ΛίJση (α) Παρατηρούμε ότι η τετράδα ( a, b, c , d) είναι ενδιαφέρουσα, αν, και μόνον αν,

2b2 = a2 + c2 και 2c2 = b2 + d2 + 1 .

Θεωρώντας την σχέση ( 1 ) ως εξίσωση με αγνώστους a, b, c , αν θέσουμε

α c χ = - < 1 και y = - > 1 , b b

( 1 ) (2)

τότε λαμβάνουμε την εξίσωση χ2 + y2 = 2, x,y Ε Q. (3)

Μία προφανής λύση της (3) είναι το ζεύγος ( x,y) = (1 , 1 ). Στη συνέχεια θα προσπαθήσουμε

να προσδιορίσουμε λύσεις ( x,y ) της (3) τέτοιες ώστε 1 - χ = m (y - 1 ) ,m Ε Q, δηλαδή λύσεις της μορφής ( 1- m (y - 1 ) ,y ) , y Ε Q.

Μ , ξ' (3) , , 1 , m2 + 2m - 1 ε αντικατασταση στην ε ισωση προκυπτει οτι y = η y = 2 ,

m + 1 , , , λ' ( ) ( 1 l ) λ' ( ) [ 2m + 1 - m2 m 2 + 2m - l ) ιΠ\

οποτε προκυπτει η γνωστη υση x,y = , και οι υσεις x,y = 2 , 2 , m Ε �. m + 1 m + 1

'Εστω m =Ρ, p,q ΕΖ με p > q . Τότε η τριάδες ( a, b, c ) με q (4)

είναι λύσεις της εξίσωσης ( 1 ), για κάθε p,q ΕΖ με p > q . Έτσι πρέπει να προσδιορίσουμε ένα

τουλάχιστον κατάλληλο ζεύγος p,q για το οποίο η τριάδα ( a, b, c ) ικανοποιεί την εξίσωση (2). Χρησιμοποιώντας τις σχέσεις που δίνονται στη (4) στην εξίσωση (2), λαμβάνουμε την εξίσωση

2(p2 + 2pq - q2 ) - (p2 + q2 )2 - 1 = d2 <=> p4 + 8p3q + 2p2q2 - 8pq3 + q4 - 1 = d2 . Για να βρούμε μία τουλάχιστον λύση με p > q , επιλέγουμε p = q + 1 , οπότε η εξίσωση γίνεται

4q4 + 24q3 + 32q2 + 1 2q = d2 <=> 4q ( q + ι )( q2 + sq + 3 ) = d2 , για την οποία με δοκιμές εύκολα λαμβάνουμε μία λύση q = 3, d = 36. Τότε είναι p = 4 και

προκύπτει η τετράδα ( a, b, c , d) = ( 1 7 , 25, 3 1 , 3 6). β) Θα αποδείξουμε ότι ο αριθμός των τετράδων που είναι ενδιαφέρουσες είναι άπειρος.

Έστω ( a, b, c , d) μία ενδιαφέρουσα τετράδα. Εύκολα διαπιστώνουμε ότι, για κάθε θετικό ακέραιο k , οι

αριθμοί

k2a2 , k2b2 , k2 c2 , k2 (d2 + 1 ) αποτελούν μία αύξουσα αριθμητική πρόοδο . Επομένως, για να κατασκευάσουμε μία νέα ενδιαφέρουσα τετράδα, αρκεί να προσδιορίσουμε θετικό ακέραιο r τέτοιο ώστε να ισχύει

ΕΥΚΛΕΙΔΗΣ Β' 70 τ.2/6

Page 9: Ευκλειδης Β 70

--------- Μαθηματικοί Διαγωνισμοί- Μαθηματικές Ολυμπιάδες --------­

k2 ( d2 + 1 ) = r2 + 1 . (5)

Μία κατάλληλη επιλογή για τον θετικό ακέραιο r είναι στη μορφή r = kd + s , οπότε η εξίσωση (5)

γίνεται k2 ( d2 + 1 ) = r2 + 1 <;::::} k2 d2 + k2 = k2 d2 + 2kds + s2 + 1 <;::::} k2 = 2kds + s2 + 1 . Επιλέγοντας k = s2 + 1 η εξίσωση μετά την απλοποίηση γίνεται s2 + 1 = 2sd + 1 <;::::} s = 2d . Για s = 2d λαμβάνουμε k = 4d2 + 1 και r = 4d3 + 3d , οπότε η τετράδα ( ka, kb , kc , r) είναι επίσης

ενδιαφέρουσα. Επαναλαμβάνουμε τη διαδικασία αυτή ξεκινώντας από την τετράδα (ka , kb , kc, r) , οπότε

κάθε φορά λαμβάνουμε μία καινούρια διαφορετική τετράδα με μεγαλύτερους θετικούς ακέραιους. Έτσι καταλήγουμε σε άπειρο αριθμό τέτοιων τετράδων.

Π ρόβλημα 4 Έστω ότι χ, y, z ε [Ο, ι) και χ+ y + z = ι . Ν α αποδείξετε ότι:

+ - -+ - -< -

�z Rfx 3 χ + yz y + zx - 2 ·

ΛίJση.

Θα χρησιμοποιήσουμε την αντικατάσταση m2 = ___!,[__ , n2 =......Ε_ , p2 = � , m, n, p � Ο . z + xy x + yz y + zx Από την ισότητα χ + y + z = 1 λαμβάνουμε:

οπότε

z + xy = ( 1 - χ) ( 1 - y) , y + zx = ( 1 - z ) ( 1 - χ) , χ + yz = ( 1 - y) ( 1 - z) ,

m2 + n2 + p2 + 2mnp = xy yz zx 2 x2 y2 z2 = + + + ( 1 - x) ( 1 - y) ( 1 - y) ( 1 - z) ( 1 - z) ( 1 - x) ( 1 - x/( 1 - y)2 ( 1 - z)2

xy(l - z) + yz( 1 - x) + zx( 1 - y) + 2xyz xy - xyz + yz - xyz + zx - xyz + 2xyz - - - 1 - (1 - χ ) ( 1 - y ) ( 1 - z) - 1 - χ - y - z = xy + yz + zx - xyz - ·

Επομένως , αρκεί να αποδείξουμε ότι 3 m + n + p � - , 2

υπό τις συνθήκες m, n, p � Ο και m2 + n2 + p2 + 2mnp = 1 . Από τις παραπάνω συνθήκες προκύπτει ότι m � 1, n � 1, p � 1 , ενώ η δεύτερη μπορεί να γραφεί ως

( m + n + p )2 - 2 ( m + n + p) + 1 = 2 ( 1 -m ) ( 1 - n ) ( 1 - p) , από την οποία, με χρήση της ανισότητας αριθμητικού-γεωμετρικού μέσου, έπεται ότι

( 1 )2 2 ( 1 - m + 1 - n + 1 - p )3 m + n + p - � 3 Α ν θέσουμε m + n + p = w � Ο , τότε από την τελευταία ανισότητα λαμβάνουμε

( w - 1 )2 � 2 ( 3�w )3 <;::::} 27 (w - 1/ � 2 (3 -w)3 <;::::} 2w3 + 9w2 - 27 � Ο . 3

Μία από τις πιθανές ρητές ρίζες του πολυωνύμου f ( w) = 2w3 + 9w2 - 27 είναι ο ρητός αριθμός 2 , ο οποίος παρατηρούμε ότι μηδενίζει το πολυώνυμο, οπότε με το σχήμα Horner ή με τη συνήθη διαίρεση πολυωνύμων λαμβάνουμε

f (w) = 2w3 + 9w2 - 27 � Ο<;::::} (w - l) ( w2 + 3w + 9) � 0<;::::} w � l<;::::} m + n + p � l , 2 2 2 αφού ισχύει w2 + 3w + 9 > Ο, για κάθε w ε IR .

ΕΥΚΛΕΙΔΗΣ Β' 70 τ.2!7

Page 10: Ευκλειδης Β 70

--------- Μαθηματικοί Διαγωνισμοί- Μαθηματικές Ολυμπιάδες ---------

"0 ΘΑnΗΣ" ΣΑΒΒΑΤΟ. 1 NOEMBPIOY 2008

Α' nνκειον Α 1 , θ , θ ' 1 θ , , , θ , 1. ν στο S ενος αρι μου χ προσ εσουμε το 4" του αρι μου αυτου προκυπτει αρι μος

μικρότερος κατά 1255 του αριθμού χ. Να βρεθεί ο αριθμός χ. Λ1Jση Σύμφωνα με την υπόθεση θα έχουμε την εξίσωση

χ χ χ χ 5χ 1 255 . 8 - + - + 1 255 =χ<=> χ----= 1 255 <=>-= 1 255 <=>χ= = 2008 . 8 4 8 4 8 5

2. Να προσδιορίσετε τους ακέραιους x ,y και z που είναι τέτοιοι ώστε: Ο� χ� y � z , xyz + xy + yz + zx + χ+ y + z = 44 .

ΛίJση Η τελευταία εξίσωση γράφεται:

xyz + xy + yz + zx + χ + y + z = 44 <:::> xyz + xy + yz + zx + χ + y + z + 1 = 45 <:::> xy( z + 1) + χ( z + 1) + y( z + 1) + ( z + 1) = 4 5 <:::> ( xy + χ + y + 1) ( z + 1) = 4 5 <:::> (χ + 1) ( y + 1) ( z + 1) = 4 5 . ( 1 )

Επειδή οι x ,y, z είναι μη αρνητικοί ακέραιοι και χ::; y::; z , έπεται ότι: 1::;x + 1::;y + 1::;z + 1 . (2)

Από τις ( 1 ) και (2) και αφού 45 = 1 · 3 · 3 · 5 προκύπτουν οι περιπτώσεις (χ + 1 ,y + 1, z + 1) = (1, 3, 1 5) ή( 1, 5, 9)ή (3, 3 ,5) ή ( 1, 1 ,45)<=> ( x ,y,z) = (Ο, 2, 14) ή (Ο, 4, 8) ή ( 2, 2, 4) ή ( 0, 0, 44) .

3. Να βρεθούν οι γωνίες των ισοσκελών τριγώνων που έχουν τη παρακάτω ιδιότητα: "υπάρχει ευθύγραμμο τμήμα που συνδέει μία κορυφή με την απέναντι πλευρά ώστε να δημιουργούνται μέσα στο ισοσκελές τρίγωνο, δύο ισοσκελή τρίγωνα". (Να εξετάσετε όλες τις δυνατές περιπτώσεις)

Λύση Έστω ισοσκελές τρίγωνο ΑΒΓ ( ΑΒ = ΑΓ ) . I 11 περίπτωση. Θεωρούμε σημείο Δ στη πλευρά ΒΓ ώστε τα τρίγωνα ΑΔΒ και ΑΔΓ να είναι ισοσκελή .

Διακρίνουμε τις υποπεριπτώσεις :

• Αν είναι ΒΜ = Β και Γ ΑΔ = Γ ΔΑ τότε ισχύουν οι ισότητες των γωνιών ( σχ. 3) : Α1 = Β = Γ = χ και Α2 = Δ1 = 2χ , (ως εξωτερική γωνία του τριγώνου ΑΒΔ, οπότε από τη σχέση Α + Β+ Γ = 1 80 καταλήγουμε στην εξίσωση : 5χ = 1 80° <:::>χ= 36°. Στη περίπτωση αυτή είναι Α = 1 08° και Β =Γ = 36° .

• Στην περίπτωση που είναι πάλιν ισοσκελή τα τρίγωνα ΑΔΒ και ΑΔΓ με ίσες γωνίες ΒΜ = ΒΔΑ και Γ ΑΔ =Γ , τότε προκύπτουν οι ίδιες γωνίες για το τρίγωνο ΑΒΓ.

Στην περίπτωση που είναι πάλιν ισοσκελή τα τρίγωνα ΑΔΒ και ΑΓ Δ με ίσες γωνίες ΒΜ = ΒΔΑ και Γ ΑΔ = Γ ΔΑ , τότε προκύπτουν οι γωνίες Α = 90' και Β = Γ = 45° , οπότε το τρίγωνο ΑΒΓ είναι ορθογώνιο ισοσκελές. Πράγματι, από τις ισότητες Β = ΒΑΔ και Γ = ΔΑr έπεται ότι:

Β + Γ = Α� 1 80° - Α = Α� Α = 90°, οπότε θα είναι Β =Γ = 45° .

ΕΥΚΛΕΙΔΗΣ Β' 70 τ.2/8

Page 11: Ευκλειδης Β 70

--------- Μαθηματικοί Διαγωνισμοί- Μαθηματικές Ολυμπιάδες ---------

--1+--

Α r -· ·i/ 2

χ 2χ

Σχήμα 3α Σχήμα 3β 2'� περίπτωση. Θεωρούμε σημείο Δ στη πλευρά ΑΓ ώστε τα τρίγωνα ΑΔΒ και

ΑΓ Δ να είναι ισοσκελή και διακρίνουμε τις υποπεριπτώσεις:

Α

Δ fπ' I 7 I

/

φ Αν ΑΒΔ = Α και ΒΔΓ = Γ , τότε (σχ. 4) ισχύουν

οι ισότητες των γωνιών: Α = Β1 = Β2

= χ και Δ1 = Γ = 2χ , αφού η γωνία Δ1 είναι εξωτερική στο τρίγωνο ΔΑΒ, οπότε χ + Β2 = B = f' = 2x � B 2 = χ.

χ .

. \

1 \ 2χ

.... , ... . �χ

·•

Γ

/ / h Από τη σχέση A +B +f' = 180° καταλήγουμε j:.'�-�-:::- • � • .• ο ..

Β

/ 7 ,ι---....., f/ i •

Γ

στην εξίσωση : 5χ = 1 80° <::::> χ = 36° . b

Στη περίπτωση αυτή είναι: Α = 36° και Β = Γ = 72° . ιι Αν ΑΒΔ = Α = χ και ΒΔΓ = ΓΒΔ = y , τότε θα έχουμε y = 2x και

Λ π Λ Λ 3π 3χ + 2y = π .οπότε λαμβάνουμε τελικά τις γωνίες Α = Ί' Β = Γ = 7

4. Α ν οι πραγματικοί αριθμοί χ, y και z ικανοποιούν τις ισότητες χ2 _Υ = z2, y2 _ z = χ2, z2 _χ = y2 ,

να αποδείξετε ότι: (α) χ3 + y3 + z3 = 3xyz. (β) Ένας τουλάχιστον από τους χ, y, z ισούται με Ο. ΛίJση (α) Με πρόσθεση κατά μέλη των τριών δεδομένων ισοτήτων λαμβάνουμε:

χ2 + / + zz - (χ + Υ + z) = χ2 + / + z2 � χ + Υ + z = Ο ' ( 1 ) από την οποία προκύπτει άμεσα το ερώτημα (α), αφού τότε είναι z = - (χ + y) και

χ3 + / + z3 = χ3 +/ +[-(χ + y)] 3 = χ3 + / - χ3 - y3 - 3xy ( x + y )= -3xy (-z) = 3xyz. (β) Από την ισότητα χ + y + z = Ο προκύπτει ότι z = -χ - y , οπότε η ισότητα χ 2 - y = z2 γίνεται

χ 2 - y = (χ + y)2 <::::> -y = 2xy + / <::::> y · (y + 2x + 1 ) = 0 <::::> y = Ο ή y = -2χ - 1 .

Γ

• Για y = Ο λαμβάνουμε χ + z = Ο <::::> z = -χ, οπότε η δεύτερη και η τρίτη των δεδομένων

σχέσεων γίνονται: χ = χ 2 <::::> χ = Ο ή χ = 1 , οπότε έχουμε τις τριάδες ( x , y, z ) = (0, 0, 0) ή (1 , 0, - 1 ) .

• Για y = -2x - 1 απότην ( l ) λαμβάνουμε z = -x - y = x + 1, οπότε με αντικατάσταση των y, z στις αρχικές σχέσεις προκύπτει η εξίσωση

χ (χ + 1 ) = 0 <::::> χ = Ο ή χ = - 1 .

ΕΥΚΛΕΙΔΗΣ Β' 70 τ.2/9

Page 12: Ευκλειδης Β 70

--------- Μαθηματικοί Διαγωνισμοί- Μαθηματικές Ολυμπιάδες --------­

Έτσι λαμβάνουμε και τις τριάδες ( x , y, z ) = (0, - 1 , 1 ) ή ( - 1, 1 , 0) . Από την εύρεση όλων των δυνατών τριάδων προέκυψε ότι σε κάθε περίπτωση ένας τουλάχιστον από

τους x ,y, z ισούται με Ο .

2"ς τρόπος για το (β) Οι δεδομένες ισότητες χ2 - y = z2 , y2 - z = χ2 , z2 - χ = y2 με πολλαπλασιασμό επί y2 , z 2 και χ2 ,

αντίστοιχα, γίνονται ( χ2 - z2 )y2 = /, (/- χ2 ) z2 = z3 , ( z2 - y2 )χ2 = χ3 ,

από τις οποίες με πρόσθεση κατά μέλη λαμβάνουμε : χ3 + / + z3 = Ο . Λόγω του (α) λαμβάνουμε xyz = Ο , δηλαδή ένας τουλάχιστον από τους x ,y, z ισούται με Ο .

β' l\YKEIOY 1. Δεκαπέντε θετικοί ακέραιοι αριθμοί, με ψηφία περισσότερα από 2, έχουν το τελευταίο διψήφιο

τμήμα τους τον αριθμό 1 5 . Να αποδείξετε ότι το άθροισμα τους είναι πολλαπλάσιο του 2 5 .

Λύση Κάθε θετικός ακέραιος που τελειώνει σε 1 5 είναι της μορφής: 1 ΟΟχ + 1 5 , όπου χ μη αρνητικός

ακέραιος. Άρα το άθροισμα των δεκαπέντε θετικών ακεραίων θα είναι:

S = (1 00χ1 + 1 5) + (1 00χ2 + 1 5) + . . · + (1 00χ1 5 + 1 5) = 1 00(χ1 + χ2 + . . . + χ1 5 ) + 1 5 · 1 5 =

= 25· 4(χ1 + χ2 + . . · + χ1 5 ) + 25· 9 = 25· [4(χ1 + χ2 + . . · + χ1 5 ) + 9] ,

δηλαδή είναι πολλαπλάσιο του 25 .

Π αρατήρηση : Η "κεντρική ιδέα" της άσκησης είναι ότι: ο θετικός ακέραιος που το τελευταίο διψήφιο τμήμα του είναι "αβ ", έχει τη μορφή 1 00χ +αβ . Με όμοιο τρόπο καταλήγουμε στο συμπέρασμα ότι: ο θετικός ακέραιος που το τελευταίο τριψήφιο τμήμα του είναι" αβy ", έχει τη μορφή 1 000χ +αβy .

3. Δίνεται τραπέζιο ΑΒΓΔ με ΑΔ 11 ΒΓ και Γ= Δ= 90" . Φέρουμε το ύψος ΑΕ και από το Ε κάθετη προς την διαγώνιο ΒΔ που την τέμνει στο σημείο Ζ. Να προσδιορίσετε το μέτρο της γωνίας

Λ

ΑΖΓ . Λύση (1 ος τρόπος)

Επειδή είναι ΑΕΓ = Γ = Δ = 90" το τετράπλευρο ΑΕΓ Δ είναι ορθογώνιο, οπότε οι διαγώνιοι του είναι ίσες και διχοτομούνται, δηλαδή το σημείο Κ είναι μέσον των ΑΓ και ΕΔ και

ΑΓ = ΕΔ . Α

;ι:_', I

Β

( 1 )

Γ

Επειδή είναι ΕΖ .l ΒΔ το τρίγωνο ΕΖΔ είναι ορθογώνιο και η ΖΚ είναι η διάμεσος αυτού προς την υποτείνουσα.

Άρα είναι ΖΚ = ΕΔ.

2

ΕΥΚΛΕΙΔΗΣ Β' 70 τ.2/10

(2)

Page 13: Ευκλειδης Β 70

--------- Μαθηματικοί Διαγωνισμοί- Μαθηματικές Ολυμπιάδες

Από τις (1) και (2) έπεται ότι ΖΚ = ΑΓ, δηλαδή η διάμεσος του τριγώνου ΑΖΓ προς την πλευρά ΑΓ 2 ισούται με το μισό της πλευράς ΑΓ. Επομένως είναι ΑΖΓ = 90 ° .

2"' Τρόπος Το τετράπλευρο ΑΔΓΕ είναι ορθογώνιο παραλληλόγραμμο, οπότε θα είναι εγγεγραμμένο σε κύκλο

με κέντρο το σημείο τομής των διαγωνίων του Ο .

Εφόσον ΕΖΔ = ΕΜ = 90°, το τετράπλευρο ΕΖΑΔ είναι εγγράψιμο και κατά συνέπεια τα σημεία Α,Δ,Γ , Ε, Ζ είναι ομοκυκλικά. Άρα ΑΖΓ = 90° (διότι βαίνει στη διάμετρο ΑΓ ).

3. Βρείτε τις τριάδες θετικών ακέραιων (χ, y , z) με χ � y � z που ικανοποιούν τις εξισώσεις: χ2 (y - z ) + y2 ( z - χ) + z2 (χ - y ) = 2 , χ + y + z = 300 .

Λl'1ση Έχουμε

x2 ( y - z) + y2 ( z - x) + z2 ( x - y) = 2<::::> x2 y - x2 z + /z -/x + z2 x - z2 y = 2 <::::> ( x2 y - y2 x ) - ( x2 z -/z ) + ( z2 x - z2 y) = 2<::::> xy ( x - y) - z ( x - y) ( x + y ) + z2 ( χ - y) = 2

<::::> ( χ - y)[ xy - z ( x + y) + z2 ] = 2 <::::> ( χ - y) ( xy - zx - zy + z2 ) = 2<::::> ( χ - y) ( y - z ) ( x - z) = 2 .

Από την τελευταία εξίσωση προκύπτει ότι οι ακέραιοι χ- y, y - z , χ - z είναι διάφοροι από το Ο. Επιπλέον, από την υπόθεση χ� y � z έπεται ότι χ - y �Ο και χ - z � y - z > Ο και αφού ( χ - y) + ( y - z) = χ - z ,

έπεται ότι οι δυνατές τιμές για τις διαφορές χ - y, y - z , χ - z είναι: χ - y = 1, y - z = 1 , χ - z = 2 . Επειδή η τρίτη εξίσωση προκύπτει με πρόσθεση κατά μέλη της πρώτης και της δεύτερης, κάθε λύση

του συστήματος της πρώτης και δεύτερης εξίσωσης είναι και λύση της τρίτης εξίσωσης, οπότε από το προηγούμενο σύστημα λαμβάνουμε: χ - y = 1, y - z = 1 <::::> χ = y + 1, z = y -1, όπου y θετικός ακέραιος. Έτσι έχουν προκύψει οι τριάδες θετικών ακέραιων

( x, y, z) = ( k + 1,k,k -1), όπου k θετικός ακέραιος. Από την εξίσωση χ + y + z = 300 λαμβάνουμε: (k + 1) + k + (k -1) = 300 <::::> 3k = 300 <::::> k = 100,

οπότε η ζητούμενη τριάδα είναι μόνον η ( χ, y, z) = ( 1 Ο 1, 100, 99) .

4. Δίνεται ευθύγραμμο τμήμα ΑΒ . Θεωρούμε τυχόν σημείο Μ εκτός του ΑΒ και τέτοιο ώστε η κάθετη από το Μ προς την ευθεία ΑΒ να την τέμνει σε εσωτερικό σημείο του ευθύγραμμου τμήματος ΑΒ. Φέρουμε ευθύγραμμα τμήματα ΑΓ και ΒΔ τέτοια ώστε

ΕΥΚΛΕΙΔΗΣ Β' 70 τ.2/11

Page 14: Ευκλειδης Β 70

--------- Μαθηματικοί Διαγωνισμοί- Μαθηματικές Ολυμπιάδες ---------ΑΓ .l ΑΜ και ΑΓ = ΑΜ , ΒΔ .l ΜΒ και ΒΔ = ΜΒ ,

και επιπλέον τα σημεία Γ, Μ και Δ να βρίσκονται στο ίδιο ημιεπίπεδο ως προς την ευθεία ΑΒ. Να αποδείξετε ότι το μέσον Κ του ευθύγραμμου τμήματος ΓΔ είναι σταθερό σημείο, δηλαδή είναι ανεξάρτητο από τη θέση του σημείου Μ.

ΛίJ ση Από τα σημεία Γ, Μ και Δ φέρουμε καθέτους ΓΕ, Μ Η και ΔΖ προς την ευθεία ΑΒ . Τότε οι οξείες

γωνίες Μ ΑΗ και ΑfΈ έχουν πλευρές κάθετες, οπότε είναι ίσες. Για τον ίδιο λόγο είναι ίσες και οι γωνίες Μ ΒΗ και ΒΔz . Έτσι από την υπόθεση Α Γ = ΑΜ προκύπτει ότι τα ορθογώνια τρίγωνα Α ΗΜ , ΓΕΑ είναι ίσα, οπότε θα έχουμε: ΓΕ = Α Η ( 1 )

:=ο

ΕΑ = Μ Η. (2)

11/1

Ομοίως από την υπόθεση ΒΔ = ΜΒ και ΒΔ .l ΜΒ προκύπτει ότι τα ορθογώνια τρίγωνα Μ ΗΒ, ΒΖΔ είναι ίσα, οπότε θα έχουμε :

ΔΖ = ΗΒ ΒΖ = Μ Η.

(3) (4)

Έστω ότι η κάθετη από το μέσον Κ της ΓΔ τέμνει την ευθεία ΑΒ στο σημείο Ο. Τότε η Κ Ο θα είναι η διάμεσος του τραπεζίου ΓΕΖΔ, οπότε θα ισχύει:

ΟΚ = ΓΕ + ΔΖ . 2

Λόγω των ( 1 ) και (3) η σχέση (5) γίνεται

ΟΚ = ΓΕ + ΔΖ = Α Η+ ΗΒ = ΑΒ. 2 2 2

(5)

(6)

Επιπλέον, το μέσον Ο της ΕΖ είναι και μέσον της ΑΒ, αφού από τις σχέσεις (2) και (4) προκύπτει ότι ΕΑ = ΒΖ, οπότε θα έχουμε ΟΑ = ΟΕ - ΑΕ = ΟΖ - ΒΖ = ΟΒ . (7)

Επομένως το σημείο Κ βρίσκεται πάνω στη μεσοκάθετη του ευθύγραμμου τμήματος ΑΒ σε απόσταση από το μέσον Ο ίση προς το μισό του ΑΒ. Άρα είναι σταθερό σημείο, δηλαδή είναι ανεξάρτητο από τη θέση του σημείου Μ.

2"' τρόπος Θεωρούμε την ευθεία ΑΒ ως άξονα των πραγματικών αριθμών στο μιγαδικό επίπεδο και το μέσον του ευθύγραμμου τμήματος ΑΒ ως την αρχή των αξόνων. Έστω ότι το σημείο Μ είναι η εικόνα του μιγαδικού αριθμού z , το σημείο Β είναι η εικόνα του πραγματικού αριθμού α , οπότε το σημείο Α θα είναι η εικόνα του πραγματικού αριθμού -α . Τότε στο διάνυσμα ΑΜ aντιστοιχίζεται ο μιγαδικός αριθμός z +α και επειδή είναι Α Γ .l ΑΜ, ΑΓ = ΑΜ έπεται ότι ( ΑΜ, ΑΓ ) = 90' , οπότε στο διάνυσμα Α Γ aντιστοιχίζεται ο μιγαδικός αριθμός ί ( z + α) . Επομένως στο διάνυσμα Ο Γ = ΟΑ + Α Γ , άρα και στο σημείο Γ, aντιστοιχίζεται ο μιγαδικός αριθμός -α+ ί ( z +α) .

ΕΥΚΛΕΙΔΗΣ Β' 70 τ.2/12

Page 15: Ευκλειδης Β 70

--------- Μαθηματικοί Διαγωνισμοί- Μαθηματικές Ολυμπιάδες --------­

Δ(α-ί(z-α)) __.----t

Κ(αί) -------� / Γ(-α+i(z+α)} -�--------

----------r-··-\ \ IVI(z)

-��---

�>--------

π! + !

2 ! �---------__;;-:Ί-- -----

χ' ο χ

Με το ίδιο σκεπτικό, αλλά με την παρατήρηση ότι (ΒΜ,ΒΔ ) = -90° , καταλήγουμε ότι στο σημείο Δ aντιστοιχίζεται ο μιγαδικός αριθμός α- ί ( z - α) .

Επομένως το μέσον Κ του ευθύγραμμου τμήματος Γ Δ είναι εικόνα του μιγαδικού αριθμού -α + ί ( z + α) + α - ί ( z - α) . ---'----'-----'----.:... =α ι 2 '

οπότε το σημείο Κ είναι σταθερό, δηλαδή ανεξάρτητο του μιγαδικού αριθμού z , άρα ανεξάρτητο από τη θέση του σημείο Μ.

Γ' nYKEIOY 1, Αν οι θετικοί ακέραιοι α και β έχουν 120 κοινούς θετικούς διαιρέτες, να προσδιορίσετε

το πλήθος των κοινών θετικών διαιρετών των αριθμών Α = 4α + 5 β και Β = 3α + 4β. ,'1,{\r;;η Θα αποδείξουμε ότι τα σύνολα των θετικών ακέραιων κοινών διαιρετών των αριθμών α, β και των

αριθμών Α και Β ταυτίζονται. Έστω ότι ο θετικός ακέραιος δ είναι κοινός διαιρέτης των αριθμών α, β . Τότε από τις σχέσεις

δ iα και δ lβ λαμβάνουμε ότι ο δ διαιρεί και κάθε γραμμικό συνδυασμό τους , οπότε δ l (4α+ 5β) = Α και δ l (3α+ 4β) = Β ,

δηλαδή ο δ είναι κοινός διαιρέτης των Α και Β . Αντίστροφα, έστω ότι ο θετικός ακέραιος δ είναι κοινός διαιρέτης των ακεραίων Α και Β . Τότε από

τις υποθέσεις δ i Α = 4α + 5 β και δ i Β = 3α + 4 β έπεται ότι δ I Α - Β = α + β , οπότε προκύπτει ότι: δ i5(Α - Β) - Α = α και δ iΑ - 4(Α - Β) = β ,

οπότε ο δ είναι κοινός διαιρέτης και των αριθμών α και β. Επομένως και οι αριθμοί Α και Β έχουν 1 20 κοινούς θετικούς ακέραιους διαιρέτες.

2. Να προσδιορίσετε το πλήθος και το άθροισμα των άρτιων θετικών ακέραιων που βρίσκονται μεταξύ των αριθμών Α = η2- η+ 1 και Β = η2 + η+ 1, όπου n θετικός ακέραιος.

ΛίJση Έχουμε Α = n(n - 1 ) + 1 και Β= n(n + 1 ) + 1 , οπότε και οι δύο αριθμοί είναι περιττοί, αφού τα γινόμενα διαδοχικών ακέραιων n(n - 1 ) και n(n + 1 ) είναι άρτιοι ακέραιοι. Επιπλέον, είναι Β - Α = 2n>O, οπότε Α< Β. Έστω Α + 1 , Α + 3 , . . . , Α + (2κ - 1 ), όπου κ θετικός ακέραιος, οι άρτιοι ακέραιοι που βρίσκονται μεταξύ των περιττών Α και Β . Τότε πρέπει Α+(2κ-1 ) = Β - 1 , δηλαδή Β - Α = 2κ � 2n = 2κ� κ = n.

Επομένως μεταξύ των αριθμών Α και Β βρίσκονται n άρτιοι ακέραιοι, οι οποίοι είναι οι Α + 1 , Α + 3 , . . . , Α + (2n - 1 ),

( 1 + 2n - 1 ) · n ενώ το άθροισμά τους είναι Σ = nA + [ 1 + 3 + . . . + (2n - 1 )] = n3 - n2 + n + = n3 + n . 2 3. Ν α προσδιορίσετε τις τριάδες ακέραιων (χ, y, z) με χ � y � z που ικανοποιούν την

ΕΥΚΛΕΙΔΗΣ Β' 70 τ.2/13

Page 16: Ευκλειδης Β 70

--------- Μαθηματικοί Διαγωνισμοί- Μαθηματικές Ολυμπιάδες ---------

εξίσωση : xy {x - y ) + yz {y - z ) + zx {z - χ) = 6. Ποιες από τις τριάδες αυτές έχουν άθροισμα τετραγώνων ελάχιστο;

Λύση Το πρώτο μέλος της δεδομένης εξίσωσης γράφεται:

.xy(x- y) + ;.z(y- z) +zx(z- x) = x2y-.xy2 + y2z-;.z2 + z2x-zx2 =.xy(x- y) + ( x- y) z2 -(χ2 - y2)z = (χ- y)[ .xy+z2 -xz-;.z ]= (χ- y)[x(y - z) - z(y- z)] = ( χ- y) (y - z)( x- z) .

Άρα η δεδομένη εξίσωση γίνεται: ( χ - y ) ( y - z ) ( χ - z) = 6 . Από την τελευταία μορφή προκύπτει ότι οι ακέραιοι χ - y, y - z , χ - z είναι διάφοροι από το Ο .

Επιπλέον από την υπόθεση χ � y � z έπεται ότι χ - y � Ο και χ - z � y - z > Ο , και αφού οι θετικοί

διαιρέτες του 6 είναι οι 1 , 2, 3, 6, έπεται ότι οι δυνατές τιμές για τις διαφορές χ - y, y - z , χ - z , είναι:

χ - Υ = 1, y - z = 2 , χ - z = 3 ( 1 ) ή x - y = 2, y - z = 1 , x - z = 3 (2) ή x - y = 1, y - z = 1 , x - z = 6 (3)

Επειδή η τρίτη εξίσωση προκύπτει με πρόσθεση κατά μέλη της πρώτης και της δεύτερης, η περίπτωση (3) δεν είναι αποδεκτή . Τα συστήματα ( 1 ) και (2) είναι αποδεκτά , αφού κάθε λύση του συστήματος της πρώτης και δεύτερης εξίσωσης είναι και λύση της τρίτης εξίσωσης, οπότε : • Από το σύστημα ( 1 ) λαμβάνουμε: χ - y = 1, y - z = 2<::> χ = y + 1, z = y - 2,

όπου y θετικός ακέραιος. Έτσι έχουν προκύψει οι τριάδες θετικών ακέραιων

( x, y, z) = ( k + 1 , k, k - 2) , όπου k θετικός ακέραιος.

• Από το σύστημα (2) λαμβάνουμε τελικά: ( x, y, z) = ( k + 2, k, k - 1 ) , όπου k θετικός ακέραιος.

Στην πρώτη περίπτωση οι τριάδες ( x, y, z) = (k + 1, k, k - 2) , k εΖ, έχουν άθροισμα τετραγώνων

s = (k + 1 )2 + k2 + (k - 2)2 = 3k2 - 2k + 5 ' που είναι τριώνυμο ως προς k και έχει ελάχιστο για k = _!_ \l Ζ . Λόγω της μονοτονίας της 3

συνάρτησης S ( k) = 3k2 - 2k + 5 εξετάζουμε τις τιμές της στους γειτονικούς ακέραιους του _!_ και έχουμε 3 S ( Ο) = 5 και S ( 1 ) = 6 , οπότε η ελάχιστη τιμή του S λαμβάνεται για κ=Ο από την τριάδα

( x, y, z) = ( 1 , 0, -2) . Στην δεύτερη περίπτωση οι τριάδες ( x, y, z) = ( k + 2, k, k - 1 ) , k εΖ, έχουν άθροισμα τετραγώνων

s = ( k + 2 )2 + k2 + ( k - 1 )2 = 3k2 + 2k + 5 ' που είναι τριώνυμο ως προς k και έχει ελάχιστο για k = _ _!_ \l Ζ. Λόγω της μονοτονίας της συνάρτησης 3 S ( k) = 3k2 + 2k + 5 εξετάζουμε τις τιμές της στους γειτονικούς ακέραιους του _!_ και έχουμε S (Ο) = 5 3 και S ( - 1 ) = 6 , οπότε η ελάχιστη τιμή του S λαμβάνεται για κ=Ο από την τριάδα ( χ, y, z) = ( 2, Ο, - 1 ) .

Επομένως η ελάχιστη τιμή του αθροίσματος των τετραγώνων των μελών των τριάδων που ικανοποιούν την δεδομένη εξίσωση είναι 5 και λαμβάνεται από τις τριάδες

( x, y, z) = ( 1, 0, -2) και ( x, y, z) = (2, 0, - 1 ) . 4. Δίνεται ευθύγραμμο τμήμα ΑΒ . Θεωρούμε τυχόν σημείο Μ εκτός του ΑΒ και τέτοιο ώστε η

κάθετη από το αυτό προς την ευθεία ΑΒ να την τέμνει σε εσωτερικό σημείο του ευθύγραμμου τμήματος ΑΒ. Φέρουμε ευθύγραμμα τμήματα ΑΓ και ΒΔ τέτοια ώστε

ΑΓ .l ΑΜ και ΑΓ = 2 · ΑΜ , ΒΔ .l ΜΒ και ΒΔ = 2 · ΜΒ

ΕΥΚΛΕΙΔΗΣ Β' 70 τ.2/14

Page 17: Ευκλειδης Β 70

--------- Μαθηματικοί Διαγωνισμοί- Μαθηματικές Ολυμπιάδες --------­

και επιπλέον τα σημεία Μ, Γ και Δ να βρίσκονται στο ίδιο ημιεπίπεδο ως προς την ευ θεία ΑΒ. Να αποδείξετε ότι το μέσον Κ του ευθύγραμμου τμήματος ΓΔ είναι σταθερό σημείο, δηλαδή είναι ανεξάρτητο από τη θέση του σημείου Μ.

ΛίJση Από τα σημεία Γ, Μ και Δ φέρουμε καθέτους ΓΕ, ΜΗ και ΔΖ προς την ευθεία ΑΒ. Τότε οι οξείες

γωνίες ΜΑΗ και ΑfΈ έχουν πλευρές κάθετες, οπότε είναι ίσες. Για τον ίδιο λόγο είναι ίσες και οι

γωνίες ΜΒΗ και ΒΔz . Έτσι τα ορθογώνια τρίγωνα ΑΗΜ , ΓΕΑ είναι όμοια, οπότε θα έχουμε:

οπότε προκύπτουν οι ισότητες

� Γ

Ί \

\

ΓΕ _ ΑΕ _ ΑΓ _2 ΑΗ-ΜΗ-ΑΜ - '

ΓΕ = 2 · ΑΗ

ΕΑ = 2 · ΜΗ .

κ •

/Ιil I

/ i:...o::..:: ___ ._..,i 'ο-· ____.,._.__ _ __,/

ο tΞ·.

/ ι i /

/Δ I ' /'

z

Ομοίως τα ορθογώνια τρίγωνα ΜΗΒ, ΒΖΔ είναι όμοια, οπότε ομοίως θα έχουμε:

ΔΖ = 2 · ΗΒ ΒΖ = 2 · ΜΗ .

(3) (4)

(1)

(2)

Έστω ότι η κάθετη από το μέσον Κ της ΓΔ τέμνει την ευθεία ΑΒ στο σημείο Ο. Τότε η ΚΟ θα είναι η

διάμεσος του τραπεζίου ΓΕΖΔ, οπότε θα ισχύει:

ΟΚ = ΓΕ + ΔΖ

. 2

Λόγω των (1) και (3) η σχέση (5) γίνεται

ΟΚ = ΓΕ + ΔΖ

2

2 · ΑΗ + 2 · ΗΒ __ 2 ·_ΑΒ __ ΑΒ ___ 2____

2 - .

(5)

(6)

Επιπλέον, το μέσον Ο της ΕΖ είναι και μέσον της ΑΒ, αφού από τις σχέσεις (2) και (4) προκύπτει ότι

ΕΑ = ΒΖ, οπότε θα έχουμε

ΟΑ = ΟΕ - ΑΕ = ΟΖ - ΒΖ = ΟΒ . (7)

Επομένως το σημείο Κ βρίσκεται πάνω στη μεσοκάθετη του ευθύγραμμου τμήματος ΑΒ σε απόσταση

από το μέσον Ο ίση προς το ΑΒ. Άρα είναι σταθερό σημείο, δηλαδή είναι ανεξάρτητο από τη θέση του

σημείου Μ.

Παρατήρηση: Το πρόβλημα αυτό μπορεί να λυθεί με χρήση της γεωμετρικής αναπαράστασης των

μιγαδικών αριθμών.

ΕΥΚΛΕΙΔΗΣ Β' 70 τ.2/15

Page 18: Ευκλειδης Β 70

HIJMIJ MArHHEMAr!CUS Η Homo Mathematicus είναι μια στήλη στο περιοδικό μας, με σκοπό την ανταλλαγή απόψεων και την ανάπτυξη προβληματισμού πάνω στα εξής θέματα: 1) Τι είναι τα Μαθηματικά, 2) Πρέπει ή όχι να διδάσκονται, 3) Ποιοι είναι οι κλάδοι των Μαθηματικών και ποιο το αντικείμενο του καθενός, 4) Ποιες είναι οι εφαρμογές τους, 5) Ποιες επιστήμες ή κλάδοι επιστημών απαιτούν καλή γνώση των Μαθηματικών για να μπορέσει κάποιος να τους σπουδάσει.

Για τους συνεργάτες της σηlJ.ης: παράκληση! τα κείμενα της στήλης αυτής, ως προς το περιεχόμενό τους και ως προς το επίπεδό τους, θα πρέπει να είναι συμβιβαστά με τα ενδιαφέροντα και το επίπεδο κατανόησης από μέρους των παιδιών.

επιμέλεια : Καρκάνης Βασίλης, Κερασαρίδης Γιάννης

I. "τι είναι τα ΜαΟηματικά; «Μια σχετικά πρόσφατη άποψη στη φιλοσοφία πρακτικές των ανθρώπων που ασκούν μαθηματικές

των μαθηματικών είναι η εκδοχή της δραστηριότητες, την ιστορία και τις εφαρμογές των διαψευσιμότητας, η οποία τονίζει την πρακτική της μαθηματικών, τη θέση των μαθηματικών στον μαθηματικής δραστηριότητας και ανθρώπινη όψη ανθρώπινο πολιτισμό, συμπεριλαμβανομένων των της μαθηματικής γνώσης. Η άποψη της ζητημάτων που συναρτώνται με τις αξίες και την διαψευσιμότητας αντιμετωπίζει τα μαθηματικά ως εκπαίδευση . Εν ολίγοις, αποδέχεται πλήρως το προϊόν κοινωνικών διαδικασιών. Η μαθηματική ανθρώπινο πρόσωπο και το ανθρώπινο συστατικό γνώση θεωρείται διαψεύσιμη και αέναα ανοιχτή σε των μαθηματικών (Daνis, Ρ. J. and Hersh, R. (1980) αναθεωρήσεις, τόσο σε ότι αφορά τις αποδείξεις "The Mathematical Experience", London: των θεωρημάτων της, όσο και σε ότι αφορά τις Penguin)». θεμελιώδεις της έννοιες (Lakatos, Ι. (1976) "Proofs Πηγή: Paul Ernest (Unίνersίty of Exeter, UK) «The

and Refutations", Cambridge: Cambridge Nature of Mathematίcs and Teachίng», "Philosophy of

University Press). Η άποψη αυτή κατά συνέπεια Mathematics Education Newsletter", 9, 1996 αποδέχεται, ως θεμιτά φιλοσοφικά αντικείμενα, τις

Γιάννης Κερασαρίδης

Ι I. 'Ά υ η) το ξ:3ρατε;" α) Σε ποιον μεγάλο μαθηματικό ανήκει το παρακάτω

απόκομμα; «η απόλυτη αυστηρότητα έχει επιτευχθεί, όλες οι ασάφειες σχετικά με το συνεχές στην ανάλυση έχουν τελικά διαλυθεί από τις φιλοσοφίες περί αριθμού του 19ου αιώνα, που στηρίζονταν στη θεωρία των άπειρων κλάσεων.... Όλα τα Μαθηματικά, έχουν αναχθεί τελικά στους φυσικούς αριθμούς και σε συλλογισμούς της παραδοσιακής λογικής. Το όνειρο των Πυθαγορείων έχει πραγματοποιηθεί. Στο εξής, οι δειλοί μαθηματικοί πρέπει

να προχωρούν άφοβα, σίγουροι πως τα θεμέλια κάτω από τα πόδια τους είναι απόλυτα στερεά»

β) Σε ποια διάσημη ομάδα μαθηματικών ανήκουν οι παρακάτω; Claude Cheνalley (1909 - 1984), Jean Delsarte (1903- 1968), Jean Dίeudonne (1906- 1992), Charles Ehresmann (1905- 1979), Szolem Mandelbrojt (1899 - 1983), Rene de Possel (1905 - 1974), Andre Weίl (1906 -1998), Henrί Cartan (1904-2008)

η απάντηση στο τέλος της στήλης

Ι Ι I. "οι συνεργάτες της στιίλιις γμάφουν-ερωτοι5ν"

Πplύτο Οέμα: Μαθηματικά και Πχνη (τρίτη συν{χεια)

/Jρολεγl)μει>α. Σήμερα σας παρουσιάζουμε, το τρίτο μέρος της ενδιαφέρουσας εργασίας «Τέχνη και Μαθηματικά στο Μουσείο Ηρακλειδών)), του Απόστολου Παπανικολάου

/

D

Μέρος Γ ΜΑΘ Η Μ ΑτΙΚΆ ΚΑΙ ΤΕΧΝΗ ΣΤ Η Ν ΑΝΑ Γ Ε Ν Ν ΗΣΗ (1400-1700 περίπου μ.Χ.)

Ο άνθρωπος ξυπνάει από το λήθαργο αιώνων και δεδομένων ιστορικών ευνοϊκών συνθηκών όπως π.χ. της άλωσης της Κωνσταντινούπολης (που έστειλε πολλούς Έλληνες λογίους στη Δύση-μεταλαμπαδευτές της ελληνικής παιδείας και μαζί τους αρκετά βιβλία της κλασικής Ελλάδας), της εφεύρεσης της τυπο­γραφίας, της διείσδυσης των Αράβων στην Ευρώπη μέσω της Ισπανίας αλλά και άλλων παραγόντων, επανεξετάζει προσεκτικά και κριτικά τα κλασικά κείμενα , γνωρίζει τον

ΕΥΚΛΕΙΔΗΣ Β' 70 τ.2/16

Page 19: Ευκλειδης Β 70

------------- HIJMIJ MArHEMAriCVJ'

Πλάτωνα, τον Ευκλείδη, τον Αρχιμήδη και τον Πάππο. Το αποτέλεσμα όλης αυτής της διεργασίας είναι η κατάκτηση της γραμμικής προοπτικής. Μαγεύεται από τις δυνατότητές της και ασχολείται έντονα μαζί της για περίπου τρεις αιώνες με νατουραλιστικές αναπαραστάσεις τοπίων και προσώπων. Τι είναι όμως γραμμική προοπτική;

Εδώ έχουμε τους καλλιτέχνες (Giotto, Bruneleschi) να προηγούνται των Μαθηματικών και να τους δείχνουν νέους δρόμους. Οι παράλληλες (στην πραγματικότητα) ευθείες του βάθους σχηματίζονται συγκλίνουσες σε ένα σημείο, το σημείο φυγής των καλλιτεχνών («επ' άπειρον» σημείο αργότερα για τους Μαθηματικούς) .

Δε θα τολμούσε Μαθηματικός να τμήσει, να νοήσει παράλληλες γραμμές που να τέμνονται. Το επτασφράγιστο μυστικό των καλλιτεχνών αναλαμβάνουν να μελετήσουν εντυπωσιασμένοι οι τότε μαθηματικοί και καλλιτέχνες της εποχής, ο Alberti, ο Da vinci και αργότερα ο Durer. Η γραμμική προοπτική τίθεται σε μαθηματική βάση για να παραχθούν κατόπιν και άλλες γεωμετρίες, η

παραστατική και η προβολική γεωμετρία και αργότερα η ελλειπτική και η υπερβολική γεωμετρία , με διαφορετική έννοια παραλληλίας από αυτήν της ευκλείδειας.

Το μονοσήμαντο και συγκεκριμένο, σίγουρο νόημα των πινάκων της Αναγέννησης , η ευφορία του μπαρόκ και του ροκοκό βρίσκεται σε πλήρη αντιστοιχία με την σιγουριά της αιτιοκρατίας (ντετερμινισμός) του Καρτέσιου και τον απόλυτο χώρο και χρόνο του Νεύτωνα. Δεδομένες αρχικές συνθήκες παράγουν συγκεκριμένα αποτελέσματα. Το σύμπαν θεωρείται μια μηχανή απόλυτα ελεγχόμενη από μαθηματικούς τύπους και κανόνες .Η γλώσσα του Σύμπαντος είναι μαθηματική έλεγε ο Γαλιλαίος.

ΤΟ ΠΕΡΑl:ΜΑ ΑΠΟ ΤΗΝ Α:\ΑΓΕΝΝΗΣΗ ΠΡΟΣ τl--!1:\ ΜΟ�ΤΕΡΝΛ ΤΕΧΝΗ (1700- i 900 μοΧο πεp[που)

Ο Ιμπρεσιονισμός ( 1 874-1 900), η τεχνοτροπία aσυνείδητης καλλιτεχνικής αναπαράστασης που απεικονίζει την άμεση, τη φευγαλέα εντύπωση μαθηματικών δομών καθόσον μια σύγχρονη ενός φυσικού τοπίου ή ενός αντικειμένου, είναι το επιστημονική έρευνα δημοσιευμένη στο βαθμιαίο πέρασμα της Τέχνης στο αφηρημένο . Αν διαδικτυακό αρχείο arXiν.org (στην οποία ο και βασικός εκπρόσωπος είναι ο Monet (1840 - καθηγητής Φυσικής Jose-Luis Aragon, με ομάδα 1 926) η έρευνά μας επικεντρώνεται κυρίως στον ερευνητών στο Πανεπιστήμιο του Μεξικού Cezanne ( 1 839 -1 906) και τον Van Gogh ( 1 853- προσπάθησε να ποσοτικοποιήσει πίνακες σαν τον: 1 890) . «Έναστρη νύχτα») διαπιστώνει ότι οι χαοτικές

Στον Cezanne γιατί είναι αυτός που πρόσθεσε δίνες που χαρακτηρίζουν αυτόν και παρόμοιους καθαρά γεωμετρικά στοιχεία στον ιμπρεσιονισμό πίνακες ακολουθούν με ακρίβεια τις μαθηματικές και ουσιαστικά ενέπνευσε την κατοπινή περιγραφές των αναταράξεων σε ρευστά υλικά, τεχνοτροπία του κυβισμού : «Eνerything in nature όπως οι στροβιλισμοί του νερού σε ένα ταραγμένο takes its form from the sphere, the cone and the ρυάκι ή οι πραγματικοί ανεμοστρόβιλοι. cylinder».O Van Gogh αποτελεί ένα παράδειγμα

MO�Tf:Ι>J\A ΤΕΧ:\Η ΚΛΠ ΓΕΩΜΕη>JΛ (2011' ιηώνας) Μιλώντας για μοντέρνα Τέχνη εννοούμε την των στοιχειωδών γεωμετρικών σχημάτων.

ολοκληρωτική άρνηση της δημιουργίας Οι συνεχιστές αυτής της νοοτροπίας της ρεαλιστικών - νατουραλιστικών αναπαραστάσεων γεωμετρικής αφαίρεσης με τον Malevich (1878 -φυσικών τοπίων ή προσώπων. Το εντυπωσιακό 1 935), τον Kandisky (1866 - 1944), τον είναι, ότι αυτή η τάση προς αφαίρεση της Mondrian, ( 1 872 - 1944) δημιούργησαν νέες λεγόμενης «μοντέρνας τέχνης», σε ένα μεγάλο της τεχνοτροπίες που ονομάστηκαν σταδιακά τμήμα εκφράζεται με μια σημαντική τάση σουπρεματισμός, κονστρουκτιβισμός, «γεωμετρικοποίησης» της ζωγραφικής, αλλά και νεοπλαστικισμός, εξπρεσιονισμός, και την καθιέρωση απλών λιτών γεωμετρικών αφηρημένος εξπρεσιονισμός. γραμμών στην Αρχιτεκτονική . Χαρακτηριστικό είναι το απόσπασμα από κείμενο

Στις αρχές του εικοστού αιώνα οι κυβιστές του Montrian: «η ομορφιά εκφράζεται πιο καλά (Picasso, Braque) συνεχιστές του ιμπρεσιονιστή από μαθηματική άποψη επομένως είναι η θέση από Cezanne φέρνουν στο επίκεντρο της τέχνης τα την οποία το καλλιτεχνικό ταμπεραμέντο του γεωμετρικά σχήματα ως βασικά συστατικά ενός μέλλοντος πρέπει να αναπτυχθεί, η θέση από την πίνακα λες και είχαν διαβάσει τον διάλογο οποία το νέο ύφος πρέπει να προκύψει. ». Καθώς «Φίληβος» του Πλάτωνα όπου ο Σωκράτης επίσης και το απόσπασμα από κείμενο του υποδεικνύει την αισθητική δύναμη της απλότητας Kandinsky: «Eνerythίng can be portrayed ... as α

ΕΥΚΛΕΙΔΗΣ Β' 70 τ.2/17

Page 20: Ευκλειδης Β 70

-------------- HtJMtJ MArHEMA7ΊCUS

mathematίcal formula. Από τα παραπάνω ελπίζουμε να έγινε φανερή η αλληλεπίδραση Μαθηματικών και τέχνης στη μοντέρνα τέχνη του 2rf" αιώνα .Περισσότερα όμως γι αυτό το θέμα καθώς και ιδιαίτερη αναφορά στο έργο του Ολλανδού καλλιτέχνη Escher ο οποίος

όπως χαρακτηριστικά έχει πει «διέσχιζε συνεχώς το σύνορο μεταξύ Μαθηματικών και Τέχνης», θα έχουμε στην επόμενη συνέχεια, κλείνοντας τη σύντομη αυτή αναφορά μας στη συσχέτιση Τέχνης και Μαθηματικών. στο επόμενο το τέταρτο και τελευταίο μέρος

Δ ει5τερο fΗμα: Υπερπράξεις και Μιγαδικοί αριΟμοί Προλεγδμενα. Στο τεύχος αυτό παρουσιάζουμε το δεύτερο και τελευταίο μέρος του ενδιαφέροντος σημειώματος του φίλου της στήλης Δρ Νίκου Ανταμπούφη για τις "υπερπράξεις". Υπενθυμίζουμε πως το σημείωμα αυτό, όπως και το πρώτο μέρος, προορίζεται για μαθητές Γ Λυκείου, που έχουν διδαχθεί τους μιγαδκούς.

«Υπερπράξεις και ΜιγαδιhΊJί αριΟμοί», Δρ. Ν ίκος ΑνταμποίJφης Μέρος δεύτερο (τελευταίο ) : ΙΣΧΥ Ρ Η ΚΑΙ ΑΣΘ Ε Ν Η Σ Π ΡΟΣΕΤΑ Ι Ρ ΙΣτΙ ΚΟΤΗΊΆ

Στο σύνολο των ([ των μιγαδικών αριθμών ορίζουμε την πρόσθεση Πλήρους Κύκλου[ Ι ] ως εξής:

χ ΕΘ y = {w Ε C : Ιwl = Ιx+yl } , δηλαδή υπεράθροισμα είναι το σύνολο των μιγαδικών w που έχουν ίδιο μέτρο με τον μιγαδικό x+y. Γεωμετρικά, το άθροισμα x+y παριστάνεται από σημείο Μ του μιγαδικού επιπέδου και το υπεράθροισμα από τον κύκλο με κέντρο την αρχή

ο και ακτίνα I ΟΜ ι . Για παράδειγμα, στο 2ΕΘ3

περιέχονται όλοι οι μιγαδικοί με μέτρο 5 όπως οι

2+3i, .J2 _ .J23 i. Το υπεράθροισμα (2ΕΘ3)ΕΘ 1 παριστάνει τα σημεία μεταξύ των ομόκεντρων κύκλων με κέντρο Ο και ακτίνες 4 και 6 (κυκλικός δακτύλιος) . Με ανάλογο τρόπο ορίζεται και ο πολλαπλασιασμός Πλήρους Κύκλου[ 1 ] ως εξής:

χ Θ y = {w Ε C: Ιw l = Ι x ·yl } .

Για παράδειγμα, θα ισχύει: (2Θ3)Θ4 = {z Ε C : Ι zl = 24 } .

Η άλγεβρα των πράξεων οδηγεί στην ισχύ ή όχι μιας ιδιότητας. Για παράδειγμα στην πρόσθεση

ιδιότητα ενώ στη διαίρεση δεν ισχύει. Η άλγεβρα των Υ περπράξεων φανερώνει και άλλες δυνατότητες που υπάρχουν. Για παράδειγμα, στον πολλαπλασιασμό Πλήρους Κύκλου τα σύνολα (2Θ3)Θ4 και 2Θ(3Θ4) είναι ίσα και γενικότερα[ ! ] (χΘy)Θz x®(y®z) . Μιλάμε για ισχυρά προσεταιριστική υπερπράξη [2], [3 ] . Αντίθετα για την πρόσθεση Πλήρους Κύκλου έχουμε: 2ΕΘ(3Ε!Η ):;t:(2ΕΘ3)ΕΘ 1 διότι είναι ο κυκλικός δακτύλιος μεταξύ των κύκλων (0, ρ=2) και (0, ρ=6) . Γενικότερα[ ! ] τα σύνολα (x®y) Θz και χΘ(yΘz) έχουν κοινά στοιχεία (δηλαδή μη κενή τομή) αλλά δεν είναι πάντοτε ίσα. Μιλάμε για ασθενώς προσεταιριστική (WASS) υπερπράξη [5]

((χΘy)Θz n xΘ(yΘz):;t:0. Οι υπερπράξεις με την ασθενή ισχύ των

ιδιοτήτων κατασκευάζουν τις Ην - δομές [5] που είναι η μεγαλύτερη κλάση των υπερδομών και εισήχθησαν από τον Θωμά Βουγιουκλή στα τέλη της δεκαετίας του 80 . Πολλά στοιχεία για τις υπερδομές μπορεί να αντλήσει κανείς από τον ηλεκτρονικό τόπο ''Α Thesaurus of Hyperstructures" (http://aha.eled.duth.gr) .

πραγματικών αριθμών ισχύει η προσεταιριστική Β ιβλιογραφικές αναφορές

• Ανταμπούφης, Ν.: Συμβολή στη μελέτη των Υπερδομών με εφαρμογές στην Υποχρεωτική Εκπαίδευση, Διδακτορική Διατριβή, Δημοκρίτειο Πανεπιστήμιο Θράκης, (2008) .

• Corsini, Ρ.: Prolegomena ofHypergroup Theory, Aviani Editore.,( 1 993). • Corsini, Ρ., Leoreanu, V. : Applίcations of Hyperstructure Theory, Kluwer Academic Publishers, (2002) . • Marty, F., Sur une generalίsatίon de !α notίon de groupe, 81h Congres Math .Scandinaves, Stockholm,( l 934),45-49 • Vougiouklis, Th. : Hyperstructures and theίr Representations, Hadronic Press., ( 1 994) .

Τρίτο (Jι:μα: Επιχειρησιαιι-ιί Έρευνα και ΜαΟηματικά Προλεγt)μενα. Σήμερα παρουσιάζουμε ένα σημείωμα του φίλου της στήλης Τηλέμαχου Μπαλτσαβιά από

την Κεφαλλονιά

« Γ ΡΑ Μ Μ Ι ΚΟΣ Π ΡΟΓΡΑ Μ Μ Ατ Ι Σ Μ ΟΣ», τι είναι και γιατί διδάσκεται σε τόσα πολλά Τριτοβάθμια Εκπαιδευτικά Ιδρύματα

Διαβάζοντας τη λέξη «προγραμματισμός» ίσως δεν είναι σωστό. Όπως θα δούμε παρακάτω, δίνεται η εντύπωση ότι πρόκειται για κλάδο της πρόκειται για κλάδο των Εφαρμοσμένων επιστήμης των ηλεκτρονικών υπολογιστών. Αυτό Μαθηματικών. Όταν παίρνουμε αποφάσεις στην

ΕΥΚΛΕΙΔΗΣ Β' 70 τ.2/18

Page 21: Ευκλειδης Β 70

--------------------------- ΗΟΜΟ ΜΑτΗΕΜΑτ�VS --------------------------προσωπική μας ζωή , οι πιο πολλοί από μας χρησιμοποιούμε πρακτικές μεθόδους που δεν επιδέχονται σαφή περιγραφή . Κάποτε έκαναν το ίδιο και μεγάλοι οργανισμοί-όπως εταιρείες και κυβερνήσεις-με αποτέλεσμα να μην παίρνονται οι καλύτερες δυνατές αποφάσεις. Λίγο μετά το τέλος του Β ' Παγκοσμίου Πολέμου, μια οργανωμένη μέθοδος μαθηματικών αποφάσεων, που ονομάστηκε Γραμμικός Προγραμματισμός, άλλαξε την κατάσταση . Μαθηματικές τεχνικές του κλάδου αυτού, κωδικοποιημένες σε προγράμματα Η/Υ, άρχισαν να συμπληρώνουν τη διαίσθηση των διευθυντικών στελεχών. Το όφελος από τη χρήση του Γραμμικού Προγραμματισμού οφείλεται στη εξοικονόμηση τεράστιων χρηματικών ποσών. Μάλιστα, για πρώτη φορά η αύξηση της κερδοφορίας οφειλόταν όχι σε τεχνολογική πρόοδο (πιο αποδοτικές και εξελιγμένες μηχανές), αλλά σε οργάνωση της παραγωγής με τον καλύτερο δυνατό τρόπο.

Για να μπούμε σε Μαθηματική περιγραφή, το πρόβλημα του Γραμμικού Προγραμματισμού έγκειται στην μεγιστοποίηση ή ελαχιστοποίηση της γραμμικής συνάρτησης η μεταβλητών:

z=c ιx ι+c2x2+ ... +cvxv με x 1,x2, .... ,xv να είναι οι μεταβλητές οι οποίες είναι μη αρνητικές, και c 1,c2, ... ,cv να είναι σταθεροί συντελεστές, κάτω από τους περιορισμούς:

α ι ι χ ι+α1 2χ2+ ... +αιv Xv:5 (=, :Ξ::)b ι α2 ιΧ ι+α22χ2+ ... +α2ν Xv:5 (=, 2:)b2

αv ι χ ι+αv2χ2+ ... +αw Xv:5 (=, 2:)bv Οι συντελεστές αίj,είναι επίσης σταθεροί για

όλα τα ί= 1 ,2, . . . ,μ , και j=l,2, . . . ,ν . Η λύση του

Ιlα. "Α υτδ το ξι:ρατε; " fοι απαντιίσειςf

προβλήματος αυτού επιτυγχάνεται πολύ αποτελεσματικά με τη μέθοδο (αλγόριθμο) simplex, η οποία οδηγεί σε συγκεκριμένο αποτέλεσμα έπειτα από πεπερασμένο πλήθος προκαθορισμένων (τυποποιημένων) βημάτων. Εμπνευστής της μεθόδου θεωρείται ο Αμερικανός George Dantzig. Αρχικά, τα ερεθίσματα που οδήγησαν στην ανάπτυξη της μεθόδου αυτής είχαν να κάνουν με την ανάπτυξη στρατιωτικών σχεδιασμών κατά τη διάρκεια του Β ' Παγκοσμίου Πολέμου. Ο στρατός των Ηνωμένων Πολιτειών εκτελούσε μεγάλες επιχειρήσεις, όχι μόνο στα πεδία των μαχών, αλλά και μέσα στο έδαφος της χώρας, επιχειρήσεις που είχαν να κάνουν με την υποστήριξη της πολεμικής προσπάθειας. Μετά το 1950 οι εφαρμογές της μεθόδου στράφηκαν στη βιομηχανία.

Η ανάπτυξη της τόσο επιτυχημένης μεθόδου simplex βασίστηκε στη Γραμμική Άλγεβρα, δικαιώνοντας για μια ακόμα φορά την άποψη που θέλει τα Θεωρητικά Μαθηματικά να προηγούνται χρονικά, κι αρκετές φορές υπομονετικά, των Εφαρμοσμένων. Το μικρό αυτό σημείωμα δεν διεκδικεί φυσικά δάφνες πληρότητας. Το μόνο που φιλοδοξεί είναι να δώσει μια επιφανειακή ενημέρωση και ίσως ένα ερέθισμα για περισσότερο ψάξιμο .

Βιβλιογραφία 1 1 1 Γιαλλέλης Μανωλάκης " Τα Σύγχρονα Μαθηματικά

στη ζωή μας" ("Consortίum for Mathematίcs and ίts applίcatίons"), W. Η. Freeman & Co, 1 990.

1 2 1 Σ. Κουνιά, Δ. Φακίνου, <<Γραμμικός Προγραμματισμός», Εκδόσεις Ζήτη, Θεσσαλονίκη, Β ' Έκδοση 200 1 .

α) Ανήκει στον Henrί Poίncare και χαρακτηρίζεται σαν "πλάνη του Poίncare" πηγή : E.T.Bell, «The Deνe1opment of Mathematίcs», σελ. 172 και σελ. 295

β) Στην ομάδα Bourbaki πηγή : Νίκος Λυγερός προς Γιάννη Κερασαρίδη

I V. Π ένθος Ο Henri Cartan, ένας από τους Eilenberg και διεξήγαγε το Seminaire Cartan

σημαντικούς μαθηματικούς του εικοστού στο Παρίσι από το 1 948 ως το 1 964. Ο Cartan αιώνα, "έφυγε" στις 1 3 Αυγούστου 2008, σε εκλέχτηκε σε περισσότερο από μία δωδεκάδα ηλικία 1 04 ετών. Ο Cartan, γιος του ακαδημίες στην Ευρώπη, τις ΗΠΑ, και την μαθηματικού Elie Cartan, ήταν ένα από τα Ιαπωνία, και έλαβε το Wolf Prize το 1 980. ιδρυτικά μέλη της ομάδας Bourbaki και είχε Εκτός από την εργασία του στα Μαθηματικά, σημαντικές συνεισφορές σε πολλούς τομείς είναι επίσης γνωστός για τις προσπάθειές του των μαθηματικών, συμπεριλαμβανομένης της για την αποκατάσταση των σχέσεων μεταξύ Σύνθετης Ανάλυσης, της Αλγεβρικής των μαθηματικών της Γαλλίας και της Τοπολογίας και της Ομόλογης Άλγεβρας. Γερμανίας μετά από τον Β ' Παγκόσμιο Συνυπέγραψε Ομόλογη Άλγεβρα με το Samuel Πόλεμο.

πηγή : Νίκος Λυγερός προς Γιάννη Κερασαρίδη

ΕΥΚΛΕΙΔΗΣ Β' 70 τ.2/19

Page 22: Ευκλειδης Β 70

•••ιιιι•τ••• ,. �ι• �� Ί�lι Ί•ιι Αιι••Ι•ιι

Δευτεροβάθμιες εξισώσεις με πραγματικούς συντελεστές

Βλαχούτσικος Γιώργος - Ευσταθίου Ευάγγελος - Καρδαμίτσης Σπύρος

Ορισμός : Κάθε εξίσωση της μορφής αχ2 + βχ + γ = Ο με α, β, γ Ε JR , α ::;:. Ο λέγεται εξίσωση δευτέρου

βαθμού με άγνωστο χ. Ορισμός : Ο αριθμός ρ θα λέγεται ρίζα μιας εξίσωσης f (x) = Ο αν την επαληθεύει δηλαδή αν f (ρ) = Ο . Μ Ο ΡΦΕΣ ΕΞΙ ΣΩΣ ΕΩΝ :

1) αχ2 = 0 2) αχ2 + βχ = Ο 3) αχ2 + γ = Ο 4) αχ2 + βχ + γ = Ο με α, β, γ Ε :!R και α ::ι:. Ο . Επίλυση της δευτεροβάθμιας εξίσωσης λέγεται ο υπολογισμός των ριζών της. Έτσι για κάθε μια από τις παραπάνω μορφές γενικά θα έχουμε : 1 ) αχ2 = Ο <=> ρ, = ρ2 = 0 2) αχ2 + βχ = Ο α ::;:. Ο <::::> χ ( αχ + β) = Ο <::::> χ = Ο ή χ = -� . α 3) αχ2 + Υ = Ο <::::> χ2 = _ .l (1) .

α ί) Αν .Ι < Ο δηλαδή _ .l > Ο η εξίσωση (I) έχει δύο ρίζες αντίθετες ρ , 2 = ± [Υ= ± Fγ I · α α

' ν-� ν ι�ι ίί) Αν .l > Ο δηλαδή _ .l < Ο η εξίσωση (I) είναι αδύνατη αφού για κάθε χ Ε JR είναι χ 2 � Ο . α α

4) αχ2 + βχ + γ = Ο , α, β, γ Ε :!R, α ::ι:. Ο <::::> 4α2χ 2 + 4αβχ + 4αγ = 0 <::::> 4α2χ2 + 4αβχ + β2 = β2 - 4αγ <=> (2αχ + β)2 = β2 - 4αγ . Την παράσταση β2 - 4αγ , ονομάζουμε διακρίνουσα Δ. I) Ε Ι ΔΟΣ Ρ Ι ΖΩΝ της αχ2 + β χ + γ = Ο . Το πρόσημο του αριθμού Δ = β2 - 4αγ χαρακτηρίζει το ΕΙΔΟΣ των ριζών της εξίσωσης.

Έτσι προκύπτουν οι παρακάτω περιπτώσεις .

ί) Αν Δ<Ο Η εξίσωση αχ2 + βχ + γ = Ο( ε) <::::> . .. <::::> (2αχ + β)2 = Δ που είναι αδύνατη επομένως η (ε) δεν

έχει ρίζες στο R. ίί) Αν Δ=Ο Η εξίσωση αχ2 + βχ + γ = Ο( ε) <::::> . . . <::::> (2αχ + β γ = Ο <::::> 2αχ + β = Ο <::::> χ = _ _Ι_ επομένως η (ε) έχει δύο ρίζες ΠΡΑΓΜΑΠΚΕΣ ΚΑΙ ΙΣΕΣ ρ, = ρ2 = _ _Ι_ . 2α 2α ίίί) Αν Δ>Ο Η εξίσωση αχ2 + βχ + γ = Ο( ε) <::::> . . . <::::> (2αχ + β)2 = Δ <::::> 2αχ + β = ±.JΔ <::::> 2αχ = -β ± .JΔ <::::> χ = -β ± .JΔ επομένως η (ε) έχει δύο ρίζες ΠΡ ΑΓΜΑ ΠΚΕΣ 2α ΚΑΙ ΑΝΙΣΕΣ που δίνονται από τον τύπο ρ, 2 =

-β ± .JΔ . ' 2α ΕΥΚΛΕΙΔΗΣ Β' 70 τ.2/20

Page 23: Ευκλειδης Β 70

Μαθηματικά για την Α ' Λυκείου

Χρήσιμες προτάσεις στο "είδος των ρ ιζών" Π ρόταση I Ικανή και αναγκαία συνθήκη ώστε η αχ 2 + β χ + γ = Ο α, β, γ ε QJ , α * Ο να έχει δύο ρίζες ΡΗΤΕΣ είναι Δ = λ2 , λ ε QJ . Π ρόταση 2 Ικανή και αναγκαία συνθήκη ώστε η αχ2 + βχ + γ = Ο α, β, γ ε QJ , α * Ο να έχει δύο ρίζες ΑΡΡΗΤΕΣ της μορφής κ ± Jμ είναι Δ>Ο και Δ * λ2 , λ ε QJ . Π ρόταση 3 Ικανή συνθήκη ώστε η αχ 2 + β χ + γ = Ο α, β, γ ε IR. , α * Ο να έχει ρίζες ΠΡΑΓΜΑ τΙΚΕΣ και ΑΝΙΣΕΣ είναι α · γ < Ο . Η συνθήκη α · γ < Ο δεν είναι αναγκαία, δηλαδή αν Δ>Ο δεν προκύπτει πάντοτε α · γ < Ο όπως στο παράδειγμα. Η χ2 - 4χ + 3 = 0 έχει Δ = 4 > 0 και α · γ = 3 > 0 .

1 1 ) l: Χ � Σ F: Ι Σ Ρ Ι ΖΩ Ν και συντελεστών της εξίσωσης : αχ2 + βχ + γ = Ο . Τύποι του Vieta

-β +JΔ -β -JΔ Αν η Διακρίνουσα Δ � Ο τότε ρ 1 = , Ρ2 = . 2α 2α Εύκολα διαπιστώνουμε ότι S = ρ 1 + ρ2 = _Ι ( 1 ) , Ρ = ρ1 ρ2 = ]_ (2) και αρ1 2 + βρ 1 + γ = Ο (3), α α αρ/ + βρ2 + γ = Ο (4) . Επίσης για την διαφορά των ριζών έχουμε I Ρ ι - ρ2 1 = JΔ ( 5) που προκύπτει ως εξής : 2α Ισχύει

( )2 -β2 - 4αγ Δ , I I JΔ ρι - ρ2 = 2 =-2 ετσι Ρι - ρ2 = -� -� . α α α

2 ( β )2 γ β2 4γ τότε (Ρι - ρ2 ) = - α - 4 α = α2 ----;; ή

,, " τ · β γ · λ 'ζ · .:....zοι.ιο : ους τυπους ρ1 + ρ2 = -- , ρ 1 • ρ2 = - τους χρησιμοποιουμε για να υπο ογι ουμε την τιμη α α

ΣΥΜΜΕΤΡΙΚΏΝ ΡΗΤΩΝ ΣΥΝΑΡΤΉΣΕΩΝ που περιέχουν τους αριθμούς ρ1 , ρ2 χωρίς να επιλύσουμε την εξίσωση . Δηλαδή S1 = Ρι + ρ2 = _Ι , S2 = ρ1 2 + ρ2 2 = (Ρι + ρ2 )2 - 2ρ1 ρ2 = . . . α S3 = ρ1 3 + ρ/ = (ρ1 + ρ2 )3 - 3ρ1 ρ2 (ρ 1 + ρ2 ) = . . . , και γενικά υπολογισμός του αθροίσματος Sv = Ρ� + Ρ� . 1 1 1 ) ΣΧΗ Μ ΑτΙ Σ Μ ΟΣ 2"" ΒΑΘ Μ Ο Υ Ε Ξ Ι Σ ΩΣ Η l: Λ Π Ο τ Ι Σ Ρ Ι Ζ Ε Σ Τ Η Σ

Η εξίσωση ax2 + βχ + γ = Ο και Δ � Ο έχει ρίζες ρ1 , ρ2 πραγματικές που πληρούν τις S = ρ1 + ρ2 = _Ι , α Ρ = ρ1 • ρ2 = ]_ και ισοδύναμα γράφεται: α (χ2 +Ι χ +]_) = Ο <::::> χ2 - (-Ι) χ +]_ = Ο α α α α α <=? Χ2 - (ρ 1 + ρ2 ) χ + ρ1 · ρ 2 = 0 <::::> χ 2 - Sχ + ρ = Ο . Άρα η εξίσωση με ρίζες ρ1 , ρ2 έχει τη μορφή ω2 - Sω + Ρ = Ο (1). Έτσι από την (Ι) μπορούμε να βρούμε δύο αριθμούς όταν γνωρίζουμε το άθροισμά τους S και το γινόμενο τους Ρ. Οι αριθμοί αυτοί θα είναι οι ρίζες της (Ι) . IV ) TO Π ΡΟl: Η Μ Ο ΤΩΝ Ρ Ι ΖΩ Ν ΊΉl: αχ2 + βχ + γ = Ο Στον παρακάτω πίνακα: Το πρόσημο των πραγματικών ριζών ρ1 , ρ2 της αχ 2 + β χ + γ = Ο βρίσκεται από το πρόσημο των S = ρ1 + ρ2 και Ρ = ρ1 ρ2 χωρίς να της υπολογίσουμε.

ΕΥΚΛΕΙΔΗΣ Β' 70 τ.2/21

Page 24: Ευκλειδης Β 70

Μαθηματικά για την Α ' Λυκείου

Αν ]_ > Ο τότε ρ1 και ρ2 είναι ομόσημες ειδικά αν: α {-Q_ > 0 => ρ1 , Ρ2 Ε �+ α . β -- < 0 => ρι , ρ2 Ε �_ α

β -- > Ο=> ρ1 = Ο και ρ2 Ε �+ α Αν ]_ = Ο τότε ρ1 = Ο και ρ2 = _Q_ , ειδικά αν: _Q_ < Ο => ρ1 = Ο και ρ2 Ε �-α α α

_Q_ = Ο => Ρι = ρ2 = Ο α _Q_ = Ο => ρι = -ρ2 α

Αν ]_ < Ο τότε ρ1 Ε �+ , ρ2 Ε �- , ειδικά αν: α _Q_ > Ο => lρι l > lρ2 l α

Μ ΕΘΟΔΟΛΟ ΓΙΑ ΓΙΑ ΤΗ Ν Λ ΥΣΗ ΑΣΚΉΣΕΩΝ Στ tΣ ΕΞ ΙΣΩΣ Ε Ι Σ 2"" ΒΑΘΜΟΥ I) Αν μας ζητάνε να λύσουμε μια εξίσωση 2'"' βαθμού • Κάνουμε πρώτα πράξεις ώστε να φέρουμε την εξίσωση στην μορφή ax 2 + βχ + γ = Ο κατόπιν

β ' Δ ' ' ' ζ ' -β ± .JΔ λ 'ζ 'ζ ρισκουμε την ιακρινουσα και απο τον τυπο των ρι ων ρ1 2 = υπο ογι ουμε τις ρι ες. · 2α • Α ν η εξίσωση είναι κλασματική πρέπει πρώτα να βρίσκουμε το σύνολο στο οποίο ορίζεται η εξίσωση . • Αν η εξίσωση έχει απόλυτα αρχικά προσδιορίζουμε τα πρόσημα των ποσοτήτων που περιέχουν τα απόλυτα, ώστε διακρίνοντας περιπτώσεις για τις τιμές του χ, να προκύπτει κάθε φορά εξίσωση χωρίς απόλυτα. Παραδείγματα

1 . Να λύσετε την εξίσωση : 2xz - 3.Jι - 2χ + ΧΖ + χ + 2 = ο(ι) Λύση

Η ( l ) <=> 2x2 - 3J(x - lγ + x + 2 = 0 <::> 2x2 - 3 l x - l l + x + 2 = 0 ( II ) • Αν x � l τότε η ( 11 ) <=> 2x2 - 3 ( x - l ) + x + 2 = 0 <::> 2x2 - 3x + 3 + x + 2 = 0 <::> 2χ2 - 2χ + 5 = 0 και

Δ = -36 < Ο . Άρα αν χ � 1 η εξίσωση είναι αδύνατη . • Αν x < l τότε η ( I1 ) <=> 2x2 + 3 ( x - l ) + x + 2 = 0 <::> 2x 2 + 3x - 3 + x + 2 = 0 <::> 2χ2 + 4χ - 1 = 0 ,

Δ = 1 6 - 4 · 2 · (- 1 ) = 24 > 0 τότε χ = -4 ±.J24 = -4 ± 2.J6 = -2 ±.J6 που είναι και οι δύο δεκτές 1 '2 4 4 2 -2 +.J6 αφού - < 1 . 2

Σχόλιο ! Πρέπει να υπολογίζουμε προσεκτικά την Διακρίνουσα γιατί μερικές φορές είναι το ανάπτυγμα α2 + β2 ± 2αβ = (α ± β)2 οπότε το αποτέλεσμα έχει απλούστερη μορφή .

2 . Να επιλύσετε την εξίσωση : αχz - (αz + α + 1) · χ + α + 1 = Ο, α Ε � Λύση

ί) Αν α = Ο έχουμε Ox2 - l · x + l = O <::> x = l ί ί) Αν α * Ο η εξίσωση είναι δευτέρου βαθμού οπότε υπολογίζουμε την Διακρίνουσα Δ = [ - (α2 + α + l )Τ - 4 · α · (α + 1) = (α2 + α + 1 )2 - 4α2 - 4α

ΕΥΚΛΕΙΔΗΣ Β' 70 τ.2/22

Page 25: Ευκλειδης Β 70

Μαθηματικά για την Α ' Λυκείου

= α4 + α2 + Ι + 2α3 + 2α2 + 2α - 4α2 - 4α = α4 + α2 + Ι + 2α3 - 2α2 - 2α = (α2 + α - Ιγ τ ' 'ζ δ' ' ' -β ± JΔ ' οτε οι ρι ες ινονται απο τον τυπο ρ, 2 = , ετσι

· 2α (α2 + α + Ι )2 ± �(α2 + α - Ι )2 (α2 + α + Ι )2 ± (α2 + α - Ι ) Ι ρι ,2 = 2α = 2α . Δηλαδή ρ, = α , ρ2 = α + Ι

3 . Να λύσετε την εξίσωση : χ2 - ( .J3 + l)x + .J3 = Ο Λύση Η Διακρίνουσα Δ = [ -( .J3 + Ι ) τ - 4 · Ι · .J3 = ( .J3 + Ι )2 - 4 · Ι · .J3 = (.J3)2 + 12 + 2 · I · ..f3 - 4 · I · .J3 = (../3)2 + I 2 - 2 · I · .J3 = (.J3- I)2

, (.J3+I) ±�(-J3-I)2 .J3 + I ± (.J3 - I) , .J3 + I +.J3 - I r;; = .J3 + I -.J3 + I = I Ετσι χ1 2 = αρα χ 1 = "\j ,j , χ 2 . ' 2 · Ι 2 2 2 4. Να λύσετε την εξίσωση που ανάγεται σε δευτεροβάθμια. χ2 + 2χ - .Jx2 + 2χ + 3 - 3 = Ο ΛίJση Η εξίσωση χ2 + 2χ -.Jx2 + 2χ + 3 - 3 (Ι) ορίζεται για χ Ε IR ώστε χ2 + 2χ + 3 � Ο . Θέτουμε .Jx2 + 2χ + 3 = t � Ο τότε χ2 + 2χ + 3 = t2 ή χ2 + 2χ = t2 - 3 τότε η εξίσωση : ισοδύναμα γράφεται: t2 - 3 - t - 3 = Ο <::::> t2 - t - 6 = Ο <::::> t 1 = 3, t2 = -2 (Απορρίπτεται) έτσι από την .Jx2 + 2χ + 3 = t <=> .Jx2 + 2χ + 3 = 3 <=> χ2 + 2χ + 3 = 9

<::::> χ2 + 2χ - 6 = Ο <::::> . . . χ 1 = - Ι + 2.JΊ, χ2 = -1 - 2.J7 θα εξετάσουμε αν είναι δεκτές. Δηλαδή χ2 + 2χ + 3 = ( 2.JΊ - Ι )2 + 2 ( 2.J7 - 1 ) + 3 = 4 · 7 + 1 - 4.J7 + 4.J7 - 2 + 3 = 30 > Ο και χ2 + 2χ + 3 = ( -2.J7 - Ι )2 + 2 ( -2.J7 - Ι ) + 3 = 4 · 7 + Ι + 4.J7 - 4.J7 - 2 + 3 = 30 > Ο Επομένως οι ρίζες χ 1 και χ2 είναι δεκτές. 5. Να επιλύσετε την εξίσωση : χ( χ + l)(x2 + χ + 1) = 42 .\ίJση Έχουμε χ(χ + 1)(χ2 + χ + Ι) = 42 <::::> (χ2 + χ)(χ2 + χ + 1) = 42 Αν θέσουμε χ2 + χ = y, y Ε R θα έχουμε : y(y + 1) = 42 <::::> y2 + y - 42 = Ο <::::> (y + 7)(y - 6) = Ο <::::> y = -7 ή y = 6 . Τότε χ2 + χ = 6 ( 1 ) ή χ2 + χ = -7 (2) .

Η εξίσωση ( l )<=> χ2 + χ - 6<::::> (χ + 3)(χ - 2) = 0<::::> χ = 2 ή χ = -3 . Η εξίσωση (2) δεν έχει ρίζες στο R γιατί Δ = 1 - 4 · 7 = -26 < Ο Επομένως η δοθείσα εξίσωση έχει ρίζες χ = -3 , χ = 2 . 6. Να δείξετε ότι μια δευτεροβάθμια εξίσωση αχ2 + βχ + γ = Ο, α ::ι:. Ο, α, β, γ Ε IR δεν μπορεί να έχει πάνω από δύο ρίζες. ΛίJση Υποθέτουμε ότι η εξίσωση αχ2 + βχ + γ = Ο έχει τρεις διαφορετικές ρίζες (κ ::ι:. λ :;t: μ :;t: κ) τότε θα ισχύουν: ακ2 + βκ + γ = 0 ( 1 ), αλ2 + βλ + γ = Ο (2), αμ2 + βμ + γ = Ο (3) . Αν αφαιρέσουμε κατά μέλη τις ( 1 ,2) έχουμε : α ( κ2 - λ2 ) + β( κ - λ) = Ο και κ :;t: λ θα είναι κ - λ :;t: Ο τότε απο α(κ + λ)(κ - λ) + β(κ - λ) = Ο � α(κ + λ) + β = 0 (4) . Όμοια αν αφαιρέσουμε κατά μέλη τις (2,3) έχουμε α(λ2 - μ2 ) + β(λ - μ) = Ο και λ - μ :;t: Ο � α(λ + μ) + β = Ο (5) .

Από τις σχέσεις (4, 5) προκύπτει ότι α( κ + λ) + β = α(λ + μ) + β � α( κ - μ) = Ο Αυτό είναι άτοπο γιατί από την υπόθεση έχουμε α :;t: Ο και κ - μ :;t: Ο . Επομένως δεν μπορεί να υπάρχουν τρείς διαφορετικές ρίζες.

ΕΥΚΛΕΙΔΗΣ Β' 70 τ.2/23

Page 26: Ευκλειδης Β 70

Μαθηματικά για την Α ' Λυκείου

1 1 ) Αν μας ζητάνε το Ε Ι ΑΟΣ Ρ Ι ΖΩ Ν της Εξίσωσης Υπολογίζουμε ΜΟΝΟ το πρόσημο της Διακρίνουσας (δεν βρίσκουμε τις ρίζες) οπότε το είδος των ριζών προκύπτει σύμφωνα με την θεωρία και τις χρήσιμες προτάσεις που έχουμε αναφέρει στην παράγραφο. Π αραδείγματα Ι . Να δείξετε ότι η εξίσωση α( γ - β)χ2 + β( α - γ)χ + γ(β - α) = Ο με α, β, γ Ε JR και α( γ - β) :;t: Ο έχει ρίζες πραγματικές. Λίιση Έχουμε Δ = β2 (α - γ)2 - 4α(γ - β)γ(β - α) = β2 (α2 + γ2 - 2αγ) - 4αγ(γβ - γα - β2 + αβ)

= β2αz + βzγz - 2αγβz - 4αγzβ + 4αzγ2 + 4αγβz - 4αzγβ = βzαz + βzγz + 2αγβ2 - 4αγ2β - 4αzγβ + 4α2γ2

= β2 (α2 + γ2 + 2αγ) - 4αγβ(γ + α) + 4α2γ2 = β2 (α + γ)2 - 4αγ · β( γ + α) + 4α2γ2 = (β( α + γ) - 2αγ Υ :?: Ο

Επομένως η εξίσωση έχει πραγματικές ρίζες. 2. Ν α δείξετε ότι όταν α, β, γ Ε Q και β - γ - α :;t: Ο . Οι λύσεις της εξίσωσης: (β - γ - α ) χ2 - 2γχ + (α - β - γ) = Ο ανήκουν στο σύνολο των ρητών αριθμών.

ΛίJση Έχουμε Δ = (-2γ)2 - 4(β - γ - α)(α - β - γ) = 4γ2 - 4 [γ + (α - β)) [γ - (α - β)]

= 4γ2 - 4[ γ2 - (α - β)2] = 4γ2 - 4γ2 + 4(α - β)2 = [ 2 (

α - β)τ :?: ο

Επομένως οι ρίζες της εξίσωσης ανήκουν στο Q. Ειδικά αν α=β από την β - γ - α :;t: Ο έχουμε γ :;t: Ο και ' -2γ γ Δ=Ο τοτε ρ 1 = ρ

2 = - = - = -Ι .

2(β - γ - α) -γ

αz βz z 3 . Να βρείτε το είδος των ριζών της εξίσωσης: -- - - = γ (I) όταν α,β,γ είναι μήκη πλευρών χ + 1 χ τριγώνου. Λίιση Αν χ :;t: -1, 0 η εξίσωση ( Ι ) <=> α2χ - β2 (χ + 1) = γ2χ(χ + 1) <=> α2χ - β2χ - β2 = γ 2χ 2 + γ2 χ

<=> γ2 χ 2 + ( γ2 - α2 + β2 ) χ + β2 = Ο ,υπολογίζουμε την Διακρίνουσα Δ = ( γ2 - αz + βz )2 _ 4γ2β2 = ( γz - αz + βz γ - (2γβ)z = ( γz - α2 + βz _ 2γβ) ( γ2 - αz + βz + 2γβ ) = [

(γ - β)2 - α2] [

(γ + βγ - α2 ] τότε Δ = (γ - β

-α) (γ - β + α) (γ + β - α) (γ + β + α)

Επειδή α,β,γ είναι μήκη πλευρών τριγώνου από την τριγωνική ανισότητα θα ισχύουν: γ < β + α, β < γ + α, α < γ + β, α + β + γ > 0 έτσι γ - β - α < Ο, γ + α - β > Ο, γ + β - α > Ο, α + β + γ > Ο

επομένως Δ < Ο . Δηλαδή η εξίσωση δεν έχει ρίζες στο σύνολο R.

4 . Δίνεται η εξίσωση χ2 + χ - 2 l x l + κ - 1 = Ο (1) Να βρείτε τις τιμές του κ Ε JR ώστε η εξίσωση (1) να έχει 4 ρίζες πραγματικές και άνισες. Λύση Διακρίνουμε τις περιπτώσεις: i ) Α ν χ :?: Ο τότε η εξίσωση ( 1 ) ισοδύναμα γράφεται: χ 2 + χ - 2χ + κ - Ι = Ο <=> χ 2 - χ + κ - Ι = Ο για να έχει δύο ρίζες πραγματικές και άνισες, επειδή έχουμε τον περιορισμό χ :?: Ο πρέπει Δ > Ο και ρ 1 + ρ

2 > Ο και ρ 1 • ρ

2 > Ο

? 5 Τότε Δ = ( -1)- - 4(κ - Ι) = 5 - 4κ > Ο <=> κ < 4 , S = ρ 1 + ρ2

= 1 > Ο

Ρ = Ρ ι · ρ2

= κ -1 > Ο <=> κ > 1 .

Αν συναληθεύσουμε τις ανισότητες: προκύπτει Ι < κ < � . 4

i i ) Αν χ < Ο τότε η εξίσωση ( Ι ) ισοδύναμα γράφεται: ΕΥΚΛΕΙΔΗΣ Β' 70 τ.2/24

για κάθε κ E JR και

Page 27: Ευκλειδης Β 70

Μαθηματικά για την Α ' Λυκείου

χ 2 + χ + 2χ + κ - 1 = 0 <::> χ 2 + 3χ + κ - 1 = 0 για να έχει δύο ρίζες πραγματικές και άνισες, επειδή έχουμε τον περιορισμό ρ1 + ρ2 < Ο και ρ1 • ρ2 > Ο .

χ < Ο πρέπει Δ > Ο και

1 3 Τότε Δ = 32 - 4(κ - 1) = 1 3 - 4κ > Ο <::> κ < - , S = ρ1 + ρ2 = -3 < 0 για 4 Ρ = ρ1 • ρ2 = κ - 1 > Ο <=> κ > 1 .

κάθε κ ε !R. και

Αν συναληθεύσουμε τις ανισότητες: προκύπτει 1 < κ < � . Επομένως για να έχει η δοθείσα (4) ρίζες 4 πραγματικές και άνισες πρέπει να συναληθεύουν οι ανισότητες (2) και (3) . Τότε προκύπτει: 1 < κ < 2 . 4 5. Να βρείτε το πρόσημο των ριζών της εξίσωσης χ2 - (λ 2 + l)x + λ 2 = Ο όταν λ ε �.* . ΛίJση Από την εξίσωση έχουμε : Ρ = λ2 > 0 , Δ=[-{λ2 +1)Τ -4λ2 = λ4 + 1 + 2λ2 -4λ2 = λ4 + 1 - 2λ2 = (λ2 - 1)2 :=:: 0 και S = λ 2 + 1 > Ο . Επομένως αν λ ε IR.* - { ± 1} θα ισχύουν Ρ > Ο, Δ > Ο, S > Ο Οπότε η εξίσωση θα έχει δύο ρίζες θετικές. Αν λ = ±1 έχουμε Δ = Ο, Ρ > Ο, S > Ο . Δηλαδή η εξίσωση θα έχει δύο ίσες ρίζες θετικές. 1 1 1 ) Ασκήσεις που μας δ ίνουν iι ζητάνε Σ Χ Ε2� Ε Π Σ Ρ Ι ΖΩΝ και Σ Υ Ν Τ ΕΛ ΕΣΤΩΝ • Όταν μας δίνουν μια σχέση που περιέχει ρ1 , ρ2 μεταξύ των ριζών της αχ 2 + β χ + γ = Ο που είναι συμμετρική ( δηλαδή μια σχέση με ρ1 , ρ2 που αν αντικαταστήσουμε τα ρ1 , ρ2 αντίστοιχα με τα ρ2 , ρ1 η σχέση δεν μεταβάλλεται) και μας ζητάνε μια σχέση μεταξύ των συντελεστών ή αν μας δώσουν μια

συμμετρική παράσταση και μας ζητούν την τιμή της, τότε μετασχηματίζουμε πάντα την συμμετρική παράσταση ώστε να εκφράζεται από τα S, Ρ. χρησιμοποιώντας τους τύπους της παραγράφου (11) .

Π α ραδείγματα 1 . Δίνεται η εξίσωση : χ2 - 5χ - 1 = Ο με ρίζες ρ1 , ρ2 να βρείτε την τιμή της

Α = 2ρ: - 3ρ:ρ2 - 3ρ1ρ; + 2ρ� . ΛίJση

Α = 2 (ρ� + ρ� ) - 3ρ ιρ2 (ρι + ρ2 ) = 2 [( ρι + ρ2 )3 - 3ρ ιρ2 (ρι + ρ2 )] - 3ρ ιρ2 (ρι + ρ2 ) = 2 (ρ1 + ρ2 )3 - 9ρ1 ρ2 (ρ1 + ρ2 ) και S = 5, P = - 1 τότε Α = 2 · 53 - 9(-1) · 5 = 1 70 .

2 . α) Αν η εξίσωση αχ2 + βχ + y = Ο (1), α, β, γ ε Q, α :;t: Ο έχει ρίζα τον άρρητο αριθμό

ρ1 = κ + .Jλ, κ, λ Ε Q, λ :;t: ρ2 , ρ Ε Q να δείξετε ότι η άλλη ρίζα της θα είναι ο αριθμός ρ2 = κ - .Jλ . ΛίJση α) Αφού ο αριθμός ρ1 είναι ρίζα της εξίσωσης προκύπτει ότι : α ( κ + .Jλ )2 + β ( κ +.Jλ ) + γ = Ο <:::> α ( κ + λ + 2κ.Jλ) + βκ + β.jλ + γ = Ο <:::> ( ακ2 + αλ + β κ + γ ) + ( 2ακ + β) · .Jλ = Ο <:::> 2ακ + β = Ο ( 1 ) και 2ακ + β = Ο (2) . Για να είναι ο ρ2 λύση πρέπει να επαληθεύει την εξίσωση (Ι) έτσι αρ/ + βρ2 + γ = α (κ -·Jλγ + β (κ -.Jλ ) + γ = α (κ2 + λ - 2κν'λ) + βκ - βν'λ + γ = ( ακ2 + αλ + β κ + γ ) - ( 2ακ + β) · ν'λ = Ο - Ο · ν'λ = Ο (Από την ( 1 ),(2),) Άρα πράγματι την επαληθεύει.

ΕΥΚΛΕΙΔΗΣ Β' 70 τ.2/25

Page 28: Ευκλειδης Β 70

Μαθηματικά για την Α ' Λυκείου

3. Δίνεται η εξίσωση αχ2 + βχ + γ = Ο, α, β, γ :;t: Ο με ρίζες ρ1 , ρ2 •

Να δείξετε ότι δεν είναι δυνατόν να ισχύουν συγχρόνως οι σχέσεις IP ι l > 2 lil (1) και IP2 1 > 2 li l (2).

Λύση

'Εχουμε ρι + ρ2 = -Ι :;t: Ο, ρι · ρ2 = _l :;t: O άρα ρι , ρ2 :;t: O και ρι · ρ2 = -Ι τότε I ρι ' Pz = ι-r ι = ιr ι (3) . α α ρι + ρ2 β ρι + ρz β β Αν δεχθούμε ότι ισχύουν συγχρόνως οι σχέσεις ( 1 ) και (2) τότε λόγω της (3) θα έχουμε: lρ ι l > 2 ιr ι = 2 ρι · ρ2 τότε lρι l · l ρ ι + ρz l > 2 lρ ι l · l ρz l όμως lρ ι i > Ο επομένως lρι + ρz i > 2 1Pz l έτσι β ρι + ρz lρ ι i + Ι ρz ι � ι ρ ι + ρz i > 2 1Pz l δηλαδή lρι l > lρz l και lρz l > 2 11 1 = 2 ρι · ρ2 τότε lρz l · lρ ι + ρz l > 2 lρ ι l · l ρz l β ρι + ρz όμως lρz l > Ο επομένως lρι + ρz l > 2 lρι l έτσι lρ ι l + lρz l � lρ ι + Pz l > 2 lρ ι l δηλαδή lρz l > lρ ι l · Παρατηρούμε ότι από τις (4),(5) προκύπτει άτοπο. Επομένως δεν μπορούν να ισχύουν συγχρόνως οι σχέσεις ( 1 ) και (2). 4. Δίνεται η εξίσωση 3 χ2 - 2( κ + 1 )χ + κ - 1 = Ο (1) με ρίζες χ1 , χ2 να υπολογίσετε τις τιμές του κ Ε JR ώστε : 12χ 1χ/ + 4χ/ + 12χ2χ: + 4χ23 = 8 (2). Λύση

Υπολογίζουμε το άθροισμα S = Χ ι + χ2 = - -2(κ + 1) = 3. (κ + 1) . 3 3 Τότε 1 2χ ι χ/ + 4χ ι3 + 1 2χ2χ; + 4χ/ = 8 <=>

[ J [ 2 ]3 8 27 <=> 4 Χ ι3 + χ/ + 3χ ι χ/ + 3χ2χ; = 8 <=> (χι + χ2 )3 = 2 <=> 3(κ + 1) = 2 <=> 27 (κ + 1)3 = 2 <=> (κ + 1)3 = 4

}27 3 3Wl 3� 3�- 4 <::> κ + 1 = V4 <::> κ + 1 = V4 <::> κ + 1 = if4J <::> κ =-4- - 1 = 4 Αν η σχέση που περιέχει τα ρι , ρ2 δεν είναι συμμετρική ως προς ρι , ρ2 τότε εκτός από τις Ρι + ρ2 = _Ι , α Ρ ι , ρ2 = 1_ χρησιμοποιούμε μια τουλάχιστον από τις α ρ; + β ρ ι + γ = Ο (3) και α ρ; + β ρ2 + γ = Ο ( 4 ). α Π αράδειγμα

Δίνεται η εξίσωση αχ2 + βχ + γ = Ο , α, β, γ Ε JR και α · γ :;t: Ο . Αν ρ1 , ρ2 είναι οι ρίζες της να

υπολογίσετε την τιμή της παράστασης Α = { αρ1 + β )-3 + { αρ2 + β )-3 • Λύση Επειδή α γ :;t: Ο θα είναι ρ ι · ρ2 :;t: Ο . Επίσης έχουμε α ρ; + βρι + γ = Ο και α ρ; + βρ2 + γ = Ο ή ( αρι + β) ρι = -γ και (αρ2 + β)ρ2 = -γ ή αρι + β = _ _r_ και αρ2 + β = _ _1__ . ρι ρ2 Επομένως η παράσταση Α γράφεται : Α = 1 + 1 = 1 + 1 = -_e{_ ρ/

(αρ, + β)' (αρ, + β)' ( _ :.)' ( _ ;, )' γ' γ'

(ρι + ρz )3 - 3ρι ρz (ρι + ρz ) _ γ3 -

(-�J3 - 3� (-�J _ο______:__-:--_ο______:_ = γ3

ΕΥΚΛΕΙΔΗΣ Β' 70 τ.2/26

Page 29: Ευκλειδης Β 70

Μαθηματικά για την Α ' Λυκείου

ΓΕΩΜΕΤΡΙΑ Παραλληλόγραμμα - Τραπέζια

Μαυρογιανάκης Λεωνίδας

Άσκηση 1'1 Δίνεται τρίγωνο ΑΒΓ και Δ, Ε τα μέσα των πλευρών του ΑΒ και ΑΓ αντίστοιχα. Από τυχαίο σημείο Λ της ΔΕ φέρουμε παράλληλες προς τις ΑΒ και ΑΓ που τέμνουν την ΒΓ στα σημεία Θ και Ζ αντίστοιχα. Να δείξετε ότι το τετράπλευρο ΔΕΖΘ είναι παραλληλόγραμμο. Λύση

Ι "; τρόπος

Επειδή η ΔΕ ενώνει τα μέσα των πλευρών ΑΒ και ΑΓ θα είναι ΔΕ = 11 ΒΓ ( 1 ) 2 Το τετράπλευρο ΒΘΛΔ είναι παραλληλόγραμμο αφού οι απέναντι πλευρές του είναι παράλληλες επομένως ΔΛ = ΒΘ. (2) . Με τον ίδιο τρόπο το τετράπλευρο ΛΕΓΖ είναι παραλληλόγραμμο, επομένως ΑΕ = ΖΓ (3) Είναι όμως ΒΓ = 2·ΔΕ <::::> ΒΘ + ΘΖ + ΖΓ = 2 ·ΔΕ <::::> ΔΛ+ΘΖ+ΛΕ=ΔΕ +ΘΖ = 2 ·ΔΕ <::::> ΘΖ = ΔΕ (4) ΔΕ=//ΘΖ. Από τις σχέσεις ( 1 ) και (4) προκύπτει το ζητούμενο. 2"; τρόπος Φέρνουμε την ευθεία ΑΛ που τέμνει την ΒΓ στο σημείο Η. Στο τρίγωνο ΑΒΗ έχουμε ότι το σημείο Δ είναι μέσο της ΑΒ και το Λ μέσο της ΑΗ. Επειδή ΛΘ // ΑΒ θα είναι το Θ το μέσο της ΒΗ. Με τον ίδιο τρόπο έχουμε ότι το Ζ είναι μέσο του ΗΓ. . ΒΗ ΗΓ ΒΓ . Άρα ΘΖ = ΘΗ + ΗΖ = -+-= -= ΔΕ. Δηλαδη ΔΕ = I/ ΘΖ. 2 2 2 Άσκηση 2 '1 Δίνεται παραλληλόγραμμο ΑΒΓ Δ. Από το κέντρο του Ο φέρνουμε ΟΚ ..ι ΑΔ ( Κ σημείο της ΑΔ) και από την κορυφή Δ την ΔΗ ..L ΒΓ. (Η σημείο της ευθείας ΒΓ). Να δείξετε ότι:

i) ΟΚ =

ΔΗ ii) ΒΗ = 2ΚΔ

2

Λύση i) Από την κορυφή Β φέρνουμε την ΒΕ ..L ΑΔ. Στο τρίγωνο ΔΕΒ είναι ΟΚ // ΒΕ (κάθετες προς την ίδια ευθεία ΑΔ) και Ο μέσον της πλευράς του ΒΔ, άρα ΟΚ = _.!._ ΕΒ ( 1 )

2 Το τετράπλευρο ΕΒΗΔ είναι ορθογώνιο (προφανώς) άρα και ΒΕ = ΔΗ (2)

ΕΥΚΛΕΙΔΗΣ Β' 70 τ.2/27

Page 30: Ευκλειδης Β 70

Μαθηματικά για την Α ' Λυκείου

1 Από τις ( 1 ) και (2) έχουμε ότι ΟΚ = - ΔΗ.

2

·=:.τ -

/ /

/,/' /

/ Ι' /

I Γ-

i i ) Έχουμε ότι το Ο είναι μέσο της ΒΔ, ΟΚ I/ ΕΒ, άρα το σημείο Κ είναι το μέσον της ΕΔ, δηλαδή ΕΔ = 2ΚΔ και αφού ΒΗ = ΕΔ συνεπάγεται ότι ΒΗ = 2ΚΔ.

Ασκηση 31 1 Στην προέκταση της πλευράς ΑΒ τετραγώνου ΑΒΓ Δ παίρνουμε τυχαίο σημείο Μ και προεκτείνουμε την πλευρά ΒΓ προς το μέρος του Γ κατά τμήμα ΓΝ = ΑΜ. Από το Ν φέρνουμε παράλληλη προς την ΔΜ και από το Μ παράλληλη προς την ΔΝ. Α ν αυτές τέμνονται στο Λ, να δείξετε ότι το τετράπλευρο ΔΜΛΝ είναι τετράγωνο. ΛίJση

Ό (\ �I

i - -�ι �- -�-'':;·<::__ . .

Συγκρίνουμε τα τρίγωνα ΑΔΜ και ΓΔΝ. Είναι ορθογώνια, έχουν ΑΔ = ΔΓ και ΑΜ = ΓΝ, άρα είναι ίσα και επομένως ΔΝ = ΔΜ ( 1 ) Το ΔΜΛΝ είναι παραλληλόγραμμο από κατασκευή με δύο διαδοχικές πλευρές ίσες, άρα είναι ρόμβος

Λ Λ Είναι: Δ z = Μ ι (2) ως εντός εναλλάξ

Λ Λ Και Δ ι = ΑΔΜ (3) γωνίες των ίσων τριγώνων (απέναντι από τις ίσες πλευρές ΓΝ και ΑΜ)

Λ Λ Λ Λ Προσθέτοντας κατά μέλη τις (2) και (3) έχουμε: Δ ι + Δ z = Μ ι + ΑΔΜ = 90°. Επομένως ο ρόμβος

έχει μια γωνία ορθή άρα είναι τετράγωνο.

ί\σκη ση 4η Δίνεται παραλληλόγραμμο ΑΒΓ Δ και εξωτερικά αυτού κατασκευάζουμε τα τετράγωνα ΑΒΕΖ και ΒΓΗΘ. Να δείξετε ότι ΒΔ = ΕΘ και ΒΔ .l ΕΘ.

ΕΥΚΛΕΙΔΗΣ Β' 70 τ.2/28

Page 31: Ευκλειδης Β 70

Μαθηματικά για την Α ' Λυκείου

__ )

-- - ,

Λ Λ Λ Λ Από τις παράλληλες ΑΔ και ΒΓ έχουμε ότι: ΔΑΒ + ΑΒΓ = 2 ορθές, επίσης ΑΒΓ + ΕΒΘ =2 ορθές,

Λ Λ Λ επομένως οι γωνίες ΔΑΒ και ΕΒΘ είναι παραπληρωματικές της γωνίας ΑΒΓ άρα μεταξύ τους ίσες.

Λ Λ Συγκρίνουμε τα τρίγωνα ΑΔΒ και ΒΕΘ. Έχουν ΑΔ = ΒΘ, ΑΒ = ΒΕ και ΔΑΒ = ΕΒΘ . Συνεπώς είναι ίσα (και θα είναι ίσες και οι πλευρές τους), άρα ΒΔ = ΕΘ. Έστω Ι το σημείο τομής της ΔΒ με την ΕΘ.

Λ Λ Λ Λ Από την προηγούμενη ισότητα τριγώνων έχουμε ΑΒΔ = ΒΕΘ ( 1 ) εξ άλλου ΕΒΙ + ΑΒΔ = 90° (2).

Λ Λ Λ Από τις ( 1 ) και (2) έχουμε ότι ΒΕΘ + ΕΒΙ =90° (3) . Από την (3) συνεπάγεται ότι ΒΙΕ =90°, άρα ΔΒ.lΕΘ.

Αν Μ είναι το μέσο της πλευράς ΑΒ τριγώνου ΑΒΓ και Ν σημείο της προέκτασής της προς το μέρος

του Β τέτοιο ώστε ΜΝ = ΑΓ . Αν Ρ είναι το μέσο της ΒΓ, να δείξετε ότι η ευθεία ΝΡ είναι κάθετη 2

στην διχοτόμο της γωνίας Α. ΛiJση

ΑΓ Η ΜΡ ενώνει τα μέσα των πλευρών ΑΒ και ΒΓ, επομένως είναι ΜΡ = 11 - , άρα ΜΝ = ΜΡ ( 1 ) . Από τις

2

ΕΥΚΛΕΙΔΗΣ Β' 70 τ.2/29

Page 32: Ευκλειδης Β 70

Μαθηματικά για την Α ' Λυκείου Λ Λ

παράλληλες ΜΡ και ΑΓ έχουμε ΜΡΝ = ΑΕΝ (2) (εντός εκτός και επί τα αυτά μέρη γωνίες ίσες) . Λ Λ

Το τρίγωνο ΜΡΝ είναι ισοσκελές, επομένως ΜΡΝ = ΜΝΡ (3) . Από τις (2) και (3) έχουμε Λ Λ

ΑΝΕ = ΑΕΝ , δηλαδή το τρίγωνο ΑΝΕ είναι ισοσκελές, ΑΝ = ΑΕ και η διχοτόμος της γωνίας Α θα είναι και ύψος. Επομένως ΑΔ ..l ΝΡ.

Άσκηση 611 Έστω Ε είναι τυχαίο σημείο της διαγωνίου ΒΔ, ορθογωνίου ΑΒΓ Δ. Στην προέκταση της ΑΕ παίρνουμε σημείο Ζ τέτοιο ώστε ΕΖ = ΑΕ και σημειώνουμε με Θ το μέσο της ΕΖ. Αν Η είναι τυχαίο σημείο της ΓΖ και η ευθεία ΗΘ τέμνει την προέκταση της ΕΒ στο σημείο Κ, να δείξετε ότι το τετράπλευρο ΕΚΖΗ είναι παραλληλόγραμμο. Λύση

·-. .. _

-. .. _ ---.

---.

.-----·

--·· .--·

· -..

· - - . __ _

___ _ . . · _ _ _ ... - - --- - - -- --- ·

!;): __

· · -. . . ·-- - -- - - - . ..._

-- -Γ - __ :·.() �- -·

Φέρνουμε τη διαγώνιο ΑΓ και Ο το κέντρο του ορθογωνίου . Στο τρίγωνο ΑΓΖ το ευθύγραμμο τμήμα ΟΕ ενώνει τα μέσα των πλευρών ΑΓ και ΑΖ, επομένως ΟΕ 11 ΓΖ και κατά συνέπεια ΕΚ 11 ΗΖ ( 1 ) .

Λ Λ Συγκρίνουμε τα τρίγωνα ΚΕΘ και ΘΗΖ, αυτά έχουν Θ1 = Θz (κατακορυφήν) , ΕΘ = ΘΖ (υπόθεση) και

Λ Λ Ε1 = Ζ 1 (εντός εναλλάξ των παραλλήλων ΕΚ, ΗΖ), άρα είναι ίσα και συνεπώς ΕΚ = ΗΖ (2) Από τις σχέσεις ( 1 ) και (2) το τετράπλευρο ΕΚΖΗ είναι παραλληλόγραμμο.

Άσκηση 7'1 Από σημείο Ρ της υποτείνουσας ΒΓ ορθογωνίου και ισοσκελούς τριγώνου ΑΒΓ (ΑΒ = ΑΓ), φέρουμε ΡΚ ..l ΑΓ και ΡΛ ..l ΑΒ. Αν (ε) η μεσοκάθετος του ΚΛ, να δείξετε ότι αυτή περνά από το μέσο της ΒΓ. Λύση Το τετράπλευρο ΑΚΡΛ έχει τρεις γωνίες ορθές, άρα είναι ορθογώνιο και ΑΛ = ΚΡ ( 1 ) . Το τρίγωνο ΚΡΓ

Λ Λ είναι ορθογώνιο ( Κ = 90°) και Γ = 45° ( αφού ΑΒΓ τρίγωνο ορθογώνιο και ισοσκελές) . Άρα και το ΚΡΓ τρίγωνο είναι ισοσκελές. ΚΡ = ΚΓ (2) . Από τις σχέσεις ( 1 ) και (2) συνεπάγεται ΑΛ = ΚΓ (3)

ΕΥΚΛΕΙΔΗΣ Β' 70 τ.2/30

Page 33: Ευκλειδης Β 70

Μαθηματικά για την Α ' Λυκείου

ΒΓ Αν Μ είναι το μέσο της ΒΓ τότε ΑΜ = - � ΑΜ = ΒΜ = ΜΓ (4) .

2 Λ Λ

Είναι ΑΜΒ τρίγωνο ισοσκελές, Β = 45°, άρα και ΛΑΜ = 45°, συνεπώς έχουμε : Λ Λ

ΛΑΜ = ΚΓΜ (5) . Από (3), (4) και (5) τα τρίγωνα ΛΑΜ και ΚΓΜ είναι ίσα, επομένως ΜΑ = ΜΚ. Το Μ δηλαδή ισαπέχει από τα άκρα του ΛΚ, άρα βρίσκεται στην μεσοκάθετο (ε) .

Άσκηση 8'1 Δίνεται τετράγωνο ΑΒΓ Δ. Α ν Κ, Λ, Μ, Ν τα μέσα των πλευρών ΑΒ, ΒΓ, Γ Δ, ΔΑ αντίστοιχα, να δείξετε ότι τα σημεία Ε, Ζ, Η, Θ που τέμνουν οι ΚΔ και ΒΜ τις ΑΛ και ΝΓ αντίστοιχα, είναι κορυφές τετραγώνου.

ΛίJση

(1. · · - - - - - . . • _ _

··ιt.ι._

,/

, l.,.ρ

�\ .. / · . . _____ ./

·-- - - . . __

_ _ .........

_

/

- !'i-.:: :-,....

.. · - ,,--

,/ ----φ '

· · · · · ·.· .. · .. :.Ρ. ._ ;- _

------

Το τετράπλευρο ΚΒΜΔ είναι παραλληλόγραμμο αφού ΚΒ = I/ ΔΜ, επομένως ΚΔ // ΒΜ και ΕΘ// ΖΗ ( 1 ) Παρόμοια αποδεικνύεται ότι ΑΛ // ΝΓ και συνεπώς ΕΖ // ΘΗ (2) . Από τις ( 1 ) και (2) το ΕΖΗΘ είναι παραλληλόγραμμο. Στο τρίγωνο ΑΒΖ, είναι Κ μέσον της ΑΒ και ΚΕ // ΒΖ, άρα το Ε είναι μέσο της ΑΖ. Για τον ίδιο λόγο στο τρίγωνο ΒΗΓ θα είναι Ζ μέσο της ΒΗ.

Λ Λ Τα τρίγωνα ΑΒΖ και ΒΗΓ είναι ίσα γιατί προφανώς τα τρίγωνα ΑΒΛ και ΒΓΜ είναι ίσα, άρα Α ι = Β ι

Λ Λ Λ Λ Λ Λ (3) και Α ι = Μ ι (4) όμως B z = Μ ι (ως εντός εναλλάξ) και Α ι = Γ ι (εντός - εκτός και επί τα αυτά)

Λ Λ και τελικά Β 2 = Γ ι (5) . Δηλαδή τα τρίγωνα ΑΒΖ και ΒΗΓ έχουν ΑΒ = ΑΓ και τις προσκείμενες σ ' αυτές τις πλευρές γωνίες ίσες. Επομένως από την ισότητα αυτή έχουμε ΑΖ = ΒΗ τότε ΕΖ = ΖΗ (6), δηλαδή το παραλληλόγραμμο ΕΖΗΘ έχει δύο διαδοχικές πλευρές ίσες, άρα είναι ρόμβος.

Λ Λ Τώρα στο τρίγωνο ΑΚΕ έχουμε Κι = B z (7) (ως εντός εκτός και επί τα αυτά ) και προσθέτοντας κατά μέλη τις ισότητες (3) και (7) έχουμε:

Λ Λ Λ Λ Λ Λ Α ι + Κ ι = Β ι + B z = 90°, άρα ΑΕΚ = 90° τότε ΘΕΖ = 1 ορθή . Από την τελευταία σχέση έχουμε ότι ΕΖΗΘ είναι τετράγωνο. Άσκη ση 9'1 Σε τραπέζιο ΑΒΓΔ (ΑΒ // ΓΔ) είναι ΓΔ = 2ΑΒ. Αν η διάμεσος ΜΝ τέμνει την διαγώνιο ΑΓ στο Κ, να δείξετε ότι η ευθεία ΒΚ περνά από το μέσο της Γ Δ. ΛίJση Στο τρίγωνο ΑΔΓ, είναι Μ μέσο της ΑΔ και ΜΚ /I ΔΓ, άρα το σημείο Κ είναι το μέσο της ΑΓ. Στο τρίγωνο ΒΛΓ είναι το σημείο Ν το μέσο της ΒΓ και ΝΚ//ΛΓ, άρα το σημείο Κ είναι το μέσο της ΒΛ. Επομένως στο τετράπλευρο ΑΒΓ Λ οι διαγώνιοι διχοτομούνται, άρα είναι παραλληλόγραμμο και ΑΒ=ΛΓ. Από την υπόθεση είναι ΑΒ = Υ2 ΓΔ, επομένως ΛΓ = Υ2 ΓΔ, δηλαδή το Λ είναι μέσο της ΓΔ.

ΕΥΚΛΕΙΔΗΣ Β' 70 τ.2/31

Page 34: Ευκλειδης Β 70

Μαθηματικά για την Α ' Λυκείου

· . . _ _

Άσκη ση l () ' l Σε τραπέζιο ΑΒΓΔ (ΑΒ // ΓΔ), ΑΒ < ΓΔ η μεσοκάθετος της πλευράς ΒΓ τέμνει την ΓΔ στο Ε και την προέκταση της ΑΒ στο σημείο Ζ. Να δείξετε ότι το τετράπλευρο ΒΕΓΖ είναι ρόμβος. ΛίJση

@--- - - - - -- -· - - - -- �,"'·-_ _, - - - -- - - - - - - -.-

Λ Λ Αφού το σημείο Ε είναι σημείο της μεσοκαθέτου του ΒΓ είναι ΕΒ = ΕΓ και Β 1 = Γ 1 . Είναι όμως

Λ Λ Λ Λ Γ 1 = Β 2 ως εντός εναλλάξ, άρα και Β 1 = Β2 . Επομένως στο τρίγωνο ΒΕΖ το τμήμα ΒΜ είναι ύψος και διχοτόμος επομένως το τρίγωνο είναι ισοσκελές, δηλαδή ΒΕ = ΒΖ. Είναι επιπλέον ΒΖ // ΕΓ, άρα στο τετράπλευρο ΒΕΓΖ οι δύο απέναντι πλευρές του είναι ίσες και παράλληλες, συνεπώς είναι παραλληλόγραμμο, με δύο διαδοχικές πλευρές ίσες άρα είναι τελικά ρόμβος. Άσκη ση 1 1 η Δίνεται τετράγωνο ΑΒΓΔ και Μ το μέσον της πλευράς ΑΔ. Αν η ΓΜ τέμνει την ΒΔ στο Η, να δείξετε ότι:

i) η ευθεία ΑΗ περνάει από το μέσο της ΔΓ. ii) ΒΔ = 3ΔΗ

ΛίJση

i ) Φέρνουμε την διαγώνιο ΑΓ και σημειώνουμε με Ο το κέντρο του τετραγώνου το οποίο είναι και μέσο της ΑΓ. Στο τρίγωνο ΑΔΓ οι ΓΜ και ΔΟ είναι διάμεσοι του, άρα το σημείο Η είναι το βαρύκεντρο του τριγώνου, συνεπώς η ΑΝ είναι η τρίτη διάμεσος του τριγώνου , δηλαδή το σημείο Ν είναι το μέσο της πλευράς Γ Δ.

i i ) Αφού το σημείο Η είναι το βαρύκεντρο του τριγώνου ΑΔΓ, θα είναι ΗΟ = Yz ΔΗ ή ΔΟ = 3/2 ΔΗ <=> 2ΔΟ = 3ΔΗ <=> ΒΔ = 3ΔΗ.

ΕΥΚΛΕΙΔΗΣ Β ' 70 τ.2/32

Page 35: Ευκλειδης Β 70

J181ΙΙJΙ8-ιιιι• ,.. Ι'φ• W 1'�1'1 �•• Αιι••Ι•ιι

Άλγεβρα Β ' Λυκείου Πολυώνυμα - Πολυωνυμικές Εξισώσεις

Γκίνης Παναγιώτης - Χριστιάς Σπύρος

Στο άρθρο που ακολουθεί παρουσιάζονται κάποιες βασικές γνώσεις από την θεωρία των πολυωνύμων

και των πολυωνυμικών εξισώσεων, ερωτήσεις σωστού - λάθους που αφορούν στην προαναφερόμενη θεω­

ρία, και γενικές ασκήσεις πάνω στην ύλη αυτής της ενότητας.

Στόχος αυτής της δημοσίευσης είναι ο μαθητής να κατανοήσει τη θεωρία και να εξοικειωθεί με τις βα­

σικές εφαρμογές των πολυωνύμων και των πολυωνυμικών εξισώσεων.

Α. ΒΑΣ Ι ΚΕΣ Γ\ΩΣ Ε Ι Σ Α ΠΌ τ l l Θ F: Ω Ρ i Α ι . Κάθε παράσταση της μορφής Ρ( χ) = αν χ ν + αν-1 χ ν- 1 + . . . + α1 χ + αο με ν Ε Ν * και

αν , αν_1 , . . . α1 , α0 Ε � , Ονομάζεται ΠΟλυώνυμο με μεταβλητή Χ. α) Οι αριθμοί αν , αν_ 1 , . . . α 1 , α0 ονομάζονται συντελεστές του πολυωνύμου ενώ ο αριθμός α0 ονο­

μάζεται σταθερός όρος του πολυωνύμου. β) Εάν Ρ( χ) = α0 , τότε το Ρ(χ) αποτελεί ένα σταθερό πολυώνυμο. Εάν Ρ( χ) = α0 = Ο , τότε το Ρ(χ)

είναι το μηδενικό πολυώνυμο. γ) Εάν αν * Ο , τότε το ν αποτελεί το βαθμό του πολυωνύμου ή ισοδύναμα λέμε ότι το πολυώνυμο

Ρ(χ) είναι νιοστού βαθμού. Συμβολικά βαθμΡ(χ)=ν. i) Εάν Ρ( χ) = α0 με α0 * Ο τότε το πολυώνυμο Ρ(χ) είναι μηδενικού βαθμού. ii) Για το μηδενικό πολυώνυμο δεν ορίζεται βαθμός.

2 . Έστω Ρ(χ), Q(x) δύο μη μηδενικά πολυώνυμα. Τότε :

α) Εάν Ρ( χ) + Q(x) είναι μη μηδενικό πολυώνυμο θα έχουμε: Βαθμ{Ρ(χ)+Q(χ)} ::; rnax {Βαθμ Ρ(χ), ΒαθμQ(χ)} .

β) Βαθμ{ Ρ( χ) · Q(x) }=Βαθμ{Ρ(χ)} + Βαθμ{Q(χ)} . 3. α) Με την αντικατάσταση χ=ρ Ε ΙR παίρνουμε τον αριθμό Ρ(ρ) = ανρν + αν_1 ρ

ν- 1 + . . . + α1ρ + α0 ο

οποίος ονομάζεται Αριθμητική τιμή του πολυωνύμου Ρ( χ) για χ=ρ. Εάν Ρ(ρ) = Ο τότε ο αριθ­

μός ρ ονομάζεται ρίζα του πολυωνύμου Ρ( χ) . β) Ισχύει Ρ( Ο) = α0 και P(l) = α1 + α2 + . . . + αν_ 1 + αν . γ) Δύο πολυώνυμα Ρ(χ), Q(x) είναι ίσα όταν και μόνο οι συντελεστές των ομοβάθμιων όρων είναι

αντίστοιχα ίσοι, ισοδύναμα όταν και μόνο Ρ(ρ )=Q(ρ) για κάθε ρ Ε IR . δ) Το Ρ(χ) είναι το μηδενικό πολυώνυμο όταν και μόνο όλοι οι συντελεστές του είναι Ο, ισοδύναμα

όταν και μόνο Ρ(ρ )=Ο για κάθε ρ Ε IR . 4. α) Εάν το πολυώνυμο Ρ(χ) είναι νιοστού βαθμού, θα έχει το πολύ ν το πλήθος διαφορετικές πραγ­

ματικές ρίζες. β) Εάν το πολυώνυμο Ρ(χ) έχει περισσότερες από ν το πλήθος διαφορετικές πραγματικές ρίζες, τότε

Ρ( χ) = Ο για κάθε χ Ε � , δηλαδή το Ρ( χ) είναι το μηδενικό πολυώνυμο. 5 . Ταυτότητα Διαίρεσης Π ολυωνίφων : Για κάθε ζεύγος πολυωνύμων Δ(χ) και δ(χ) όπου δ(χ) είναι

πολυώνυμο διαφορετικό του μηδενικού πολυωνύμου, υπάρχουν μοναδικά πολυώνυμα π(χ) και υ(χ)

ΕΥΚΛΕΙΔΗΣ Β' 70 τ.2/33

Page 36: Ευκλειδης Β 70

Μαθηματικά για την Β ' Λυκείου

τέτοια ώστε : Δ( χ) = δ(χ)π(χ) + υ( χ)*

όπου υ( χ) μηδενικό πολυώνυμο ή βαθμ{υ(χ)}<βαθμ{δ(χ)} . α) Τα πολυώνυμα Δ(χ), δ(χ), π(χ) και υ(χ) ονομάζονται αντίστοιχα διαιρετέος, διαιρέτης, πηλίκο

και υπόλοιπο της διαίρεσης. β) Μπορούμε να έχουμε βαθμ{δ(χ)}>βαθμ{Δ(χ)} . Τότε π(χ)=Ο και υ(χ)=Δ(χ) . γ) Εάν υ( χ) μηδενικό πολυώνυμο τότε λέμε ότι το δ(χ) διαιρεί το Δ(χ) ή ότι το δ(χ) είναι παράγο-

ντας του Δ(χ) ή ότι το Δ(χ) διαιρείται από το δ(χ) . δ) Εάν υ( χ) μηδενικό πολυώνυμο τότε οι ρίζες του πολυωνύμου δ(χ) είναι και ρίζες του πολυωνύ-

μου Δ(χ) . Το αντίστροφο δεν ισχύει. ε) Διαίρεση πολυωνίJμου Ρ(χ ) μΕ πολυώνυμο πρώτου βαθμοίJ :

i) Το υπόλοιπο της διαίρεσης του πολυωνύμου Ρ(χ) με πολυώνυμο της μορφής (χ-ρ) είναι στα­θερά και ισχύει ότι υ( χ) = c = Ρ(ρ) .

ii) Το (χ-ρ) είναι παράγοντας του πολυωνύμου Ρ( χ) εάν και μόνο εάν Ρ(ρ) = Ο .

iii) Το υπόλοιπο της διαίρεσης του πολυωνύμου Ρ(χ) με πολυώνυμο της μορφής (χ+ρ) είναι ο αριθμός υ = Ρ( -ρ) .

iv) Το υπόλοιπο της διαίρεσης του πολυωνύμου Ρ(χ) με πολυώνυμο της μορφής (αχ+β) (όπου

α "# Ο ), είναι ο αριθμός υ = Ρ(-�) . α

στ) Έστω πολυώνυμο Ρ( χ) . Τότε ισχύουν οι ακόλουθες ισοδυναμίες: Το πολυώνυμο Ρ(χ) διαιρείται με πολυώνυμο της μορφής (χ-ρ) <=> Το πολυώνυμο (χ-ρ) διαιρεί το πολυώνυμο Ρ(χ) <=> Το πολυώνυμο (χ-ρ) είναι παράγοντας του πολυωνύμου Ρ(χ) <=> Η διαίρε­ση του Ρ(χ) με το (χ-ρ) είναι τέλεια <=> Το υπόλοιπο της διαίρεσης του Ρ(χ) με το (χ-ρ) είναι το μηδενικό πολυώνυμο <=> Ρ(ρ) = Ο <=> Το ρ είναι ρίζα του Ρ( χ) <=> Ρ( χ) = (χ - ρ )Κ( χ) όπου Κ( χ), πολυώνυμο βαθμού κατά ένα μικρότερου του βαθμού του Ρ(χ) .

ζ) Έστω πολυώνυμο Ρ(χ) . Τότε ισχύουν οι ακόλουθες ισοδυναμίες: Το πολυώνυμο Ρ( χ) διαιρείται με το πολυώνυμο Q(x) <=> Το πολυώνυμο Q(x) διαιρεί το πολυώ­νυμο Ρ( χ) <=> Η διαίρεση του Ρ( χ) με το Q(x) είναι τέλεια <=> Το υπόλοιπο της διαίρεσης του Ρ( χ) με το Q(x) είναι το μηδενικό πολυώνυμο <=> Ρ( χ) = Q(x)K(x) όπου Κ(χ) πολυώνυμο, του οποί­

ου ο βαθμός καθορίζεται από τη σχέση : Βαθμ{Κ(χ)}=Βαθμ{Ρ(χ)}-Βαθμ{Q(χ)} .

6. Έστω το πολυώνυμο Ρ( χ) = αν χ ν + αν-ι χ ν- ι + . . . + α1 χ + α0 με αν "# Ο και ρ1 , ρ2 , . . . ρν οι ρίζες του. Τό­

τε ισχύει: Ρ( χ) = αν (χ - ρι )(χ - ρz ) . . . (χ - ρ ν ) ·

7. Π ολυωνυμ ικές Εξισώσεις : Είναι οι εξισώσεις της μορφής αν χ ν + αν_1 χ ν- ι + . . . + α1 χ + α0 = Ο με

αν "# Ο . Ισχύει το ακόλουθο Θειόρή μα : Εάν αν , αν-ι , . . . α 1 , α0 ακέραιοι και ο ακέραιος ρ "# Ο είναι

ρίζα της εξίσωσης τότε ο ρ διαιρεί το α0 • (το αντίστροφο δεν ισχύει) .

Για την επίλυση : των πολυωνυμικών εξισώσεων χρησιμοποιούμε την παραπάνω πρόταση, σύμφωνα με την οποία, εντοπίζουμε ακέραια ρίζα της εξίσωσης ανάμεσα στους ακέραιους διαιρέτες του α0 και στη συνέχεια παραγοντοποιούμε το Α' μέλος της εξίσωσης (με σχήμα Horner ή με κάποιον άλλο τρόπο) . α) Εάν α1 + α2 + . . . + αν-ι + αν = Ο τότε ο ρ=l είναι ρίζα της εξίσωσης.

β) Εάν αν , αν_1 , . . . α 1 , α0 ομόσημοι, τότε αναζητούμε μόνο αρνητική ρίζα.

γ) Εάν οι συντελεστές περιττής τάξης α1 , α3 , α5 , . . . είναι αρνητικοί, τότε η εξίσωση δεν έχει αρνη­

τική ρίζα. δ) Εάν α0 = Ο τότε το μηδέν είναι ρίζα της εξίσωσης.

ε) ο αριθμός ρ αποτελεί ρίζα της εξίσωσης Ρ( χ) = αν χ ν + αν-Ι χ ν-Ι + . . . + α\ χ + ασ = ο με πολλαπλό­

τητα μ, εάν το πολυώνυμο Ρ( χ) γράφεται ως: Ρ( χ) = (χ - ρ )μ Π( χ) με Π(ρ) "# Ο .

* Για να μην εκλαμβάνεται μια τέτοια σχέση ως εξίσωση, καλό είναι να γράφουμε Δ( χ) = δ( χ )π( χ) + υ( χ) , για κά­

θε χ ε IR , απευθυνόμενοι στις αριθμητικές τιμές τους. Σε παλαιότερα σχολικά βιβλία αυτό αποδιδόταν με το σύμ­βολο Δ( χ) = δ(χ)π(χ) + υ( χ) που υποδηλώνει ταύτιση .

ΕΥΚΛΕΙΔΗΣ Β' 70 τ.2/34

Page 37: Ευκλειδης Β 70

Μαθηματικά για την Β ' Λυκείου

Β . Ε ΙJΩ'fΉ Σ Ε Ι Σ Σ Ω Σ Τ Ο Υ Λ Λ Θ Ο Υ Σ

1. Κάθε σταθερό πολυώνυμο είναι μηδενικού βαθμού. 2. Σε μία διαίρεση πολυωνύμων ο βαθμός του διαιρετέου είναι πάντοτε μεγαλύτερος ή ίσος του βαθμού

του διαιρέτη . 3. Δύο μη μηδενικά πολυώνυμα είναι ίσα εάν και μόνο εάν έχουν τις ίδιες ρίζες. 4 'Ε λ , ξ' ν ν- Ι Ο . στω η πο υωνυμικη ε ισωση ανχ + αν_1 χ + . . . + α1 χ + α0 = .

α1 + α2 + . . . + αν-ι + αν = Ο , τότε η εξίσωση έχει ακέραια ρίζα. Εάν ισχύει

5. Έστω η πολυωνυμική εξίσωση αν χ ν+ αν_] χ

ν- Ι + . . . + αι χ + ασ = ο με αν ' αν-Ι , . . . αι ' ασ ακέραιους. Εάν ο ακέραιος ρ =ι. Ο , είναι διαιρέτης του α0 , τότε ο ρ είναι ρίζα της εξίσωσης.

6. Για οποιαδήποτε πολυώνυμα Ρ(χ), Q(x) με βαθμούς μ και ν αντιστοίχως. Ισχύει: Βαθμ{Ρ(χ)Q(χ)}=μν.

7. Εάν ένα πολυώνυμο είναι το μηδενικό πολυώνυμο, τότε έχει βαθμό μηδέν. 8. Έστω Ρ( χ) = αχ2 + βχ + γ . Εάν οι ρ1=-2 , ρ2=1 και ρ3=4 είναι ρίζες του, τότε το Ρ( χ) είναι το μηδε­

νικό πολυώνυμο. 9. Το πηλίκο της διαίρεσης ενός πολυωνύμου έκτου βαθμού με ένα πολυώνυμο δευτέρου βαθμού, είναι

ένα πολυώνυμο τετάρτου βαθμού. 10. Για οποιοδήποτε πολυώνυμο Q(x) που είναι παράγοντας του πολυωνύμου Ρ(χ) οι ρίζες του πολυω­

νύμου Ρ(χ) είναι και ρίζες του πολυωνύμου Q(x). 11 . Εάν το πολυώνυμο Ρ(χ) διαιρείται με το πολυώνυμο Q(x), τότε θα διαιρείται και με κάθε παράγοντα

του πολυωνύμου Q(x). 12. Εάν γνωρίζουμε ότι το πολυώνυμο Q(x) διαιρεί το πολυώνυμο Ρ(χ), τότε το πολυώνυμο Q(x) διαιρεί

και το πολυώνυμο Ρ( χ) + Q(x) .

. \ Π Α :\ Τ Η Σ Ε Ι Σ Στ Ι Σ Ε ΡΩ Τ Η � Ε Ω 2 �ΩΣΤ Ο Υ - Λ Α Θ Ο Υ Σ

I : I : I : I : I : I ,: I : I : I : I � I � I : ι Γ. Γ Ε !\ Ι ΚΑ Θ Ε Μ Α Τ Α Π Ο Λ Υ Ω Ν Υ Μ Ω i\ Κ Λ Ι Π ΟΛ Υ Ω Ν Υ Μ Ι Κ Ω Ν Ε Ξ Ι �Ω � [ Ω Ν

Θ Ε ΙVΙ Α Ι " : Έστω το πολυώνυμο Ρ(χ), με σταθερό όρο τον αριθμό -1, για το οποίο ισχύει: Ρ(χ + 1) - Ρ(χ) = 4χ + 3 για κάθε χ Ε 9t . α) Να βρεθεί το πολυώνυμο Ρ(χ). β) Να λυθεί η ανίσωση : (2χ - l)P(x) :::;; Ο ΛίJση : α) Εάν το πολυώνυμο Ρ( χ) είναι σταθερό πολυώνυμο, τότε παίρνουμε ότι Ρ( χ + 1) - Ρ( χ) = Ο που είναι

άτοπο λόγω της υπόθεσης. Έστω ότι το πολυώνυμο Ρ( χ) είναι πρώτου βαθμού. Τότε το Ρ( χ) θα έ­χει τη μορφή Ρ( χ) = κχ + λ με κ,λ ε 9t . 'Ετσι παίρνουμε ότι Ρ( χ + 1) - Ρ( χ) = κ( χ + 1) + λ - κχ - λ =κ που είναι πάλι άτοπο λόγω της υπόθεσης. Εάν το πολυώνυμο Ρ(χ) είναι βαθμού μεγαλύτερου ή ίσου του 3, τότε η διαφορά Ρ( χ + 1) - Ρ( χ) θα είναι ένα πολυώνυμο δευτέρου βαθμού που αποτελεί επίσης άτοπο λόγω της υπόθεσης. Άρα το πολυώνυμο Ρ(χ) θα πρέπει να είναι δευτέρου βαθμού και έτσι Ρ(χ) = κχ 2 + λχ + μ . Όμως γνωρίζουμε ότι μ=-1 . Άρα Ρ(χ) = κχ 2 + λχ - 1 . Τότε θα έχουμε: Ρ( χ + 1) - Ρ( χ) = κ( χ + 1)2 + λ( χ + 1) - 1 - κχ2 - λχ + 1 = 2κχ + (κ + λ) . Επομένως, λόγω υπόθεσης: 2κχ + (κ + λ) = 4χ + 3 για κάθε χ Ε 9t <::::> 2κ=4 και κ+λ=3<::::>(κ, λ)=(2, 1) . Άρα το ζητούμενο πολυώ­νυμο είναι το Ρ(χ) = 2χ2 + χ - 1 .

β) Η προς λύση ανίσωση γράφεται: (2x - 1)(2x2 + x - 1) :s;; O ( 1 ) και ( 1 )<::::> (2x - 1)2 (x + 1) :s;; O 1 <::> χ ε (-οο, - 1] υ {-} . 2

ΕΥΚΛΕΙΔΗΣ Β' 70 τ.21/35

Page 38: Ευκλειδης Β 70

Μαθηματικά για την Β ' Λυκείου

Ιi�V ιι: :νΙ Α 2 " : Δίνεται το πολυώνυμο Ρ( χ) = (2α + β)χ3 + (α + β)χ2 + (α2 - 5α - 3)χ + β2 + 5 με α , β Ε 9t . α) Να βρεθούν τα α, β ώστε να έχει ρίζα τον αριθμό 1 . β) Να λυθεί η εξίσωση Ρ(χ)=Ο.

α) Πρέπει και αρκεί Ρ(1)=0 . Αλλά Ρ(1)=0<::> 2α+β+α+β+α2 -5α-3+β2 +5 = 0<::>α2 -2α+β2 +2β+2= 0<=> α2 - 2α + 1 + β2 + +2β + 1 = 0 <::> (α - 1)2 + (β + 1)2 = 0 <::> α = 1 και β = - 1 .

β) Για τις τιμές των α και β που βρήκαμε από το πρώτο ερώτημα, το πολυώνυμο Ρ(χ) γράφεται: Ρ( χ) = χ3 - 7χ + 6 . Γνωρίζουμε ότι το χ=1 είναι ρίζα του πολυωνύμου. Κάνοντας σχήμα Horner για το πολυώνυμο Ρ(χ) με ρ=1 , θα πάρουμε ότι το πηλίκο της διαίρεσης του Ρ(χ) με το (χ-1) είναι το πο­λυώνυμο π(χ)=χ2+χ-6 . Έτσι έχουμε : Ρ( χ) = Ο <::::> (χ - 1)(χ 2 + χ - 6) = Ο <::::> χ = 1 ή (χ2 + χ - 6) = Ο . <=> χ= 1 ή χ = -3 ή χ = 2 .

Θ [ �ιΊ! /•. Δίνονται τα πολυώνυμα: Q1 (x) = χ2 - (κ + λ)χ + κλ , Q2 (x) = χ2 - (κ + μ)χ + κμ και Q3 (χ) = χ2 - (λ + μ)χ + λμ όπου κ, λ, μ Ε 9t με κ :;e λ :;e μ :;e κ . Να λυθεί η εξίσωση

μ Q. (x)

+ λ Q2 (x)

+ κ Q3 (x) = χ2 .

Qι (μ) Qz (λ) Q3 (κ)

Το πολυώνυμο Q1 (χ) = χ2 - (κ + λ)χ + κλ παραγοντοποιείται και παίρνει τη μορφή : Q1 (χ) = (χ - κ)(χ - λ) . Όμοια Q2 (χ) = (χ - κ)(χ - μ) και Q3 (χ) = (χ - λ)(χ - μ) . Θέτουμε Q(x) = μ Q1 (x) + λ Q2 (x) + κ Q3 (x) _ Τότε έχουμε: Q(μ) = μ Q1 (μ) = μ , Q(λ) = λ Q2 (λ) = λ και Qι (μ) Qz (λ) QJ (κ) Qι (μ) Qz (λ) Q(κ) = κ Q3 (κ) = κ . Δηλαδή, το πολυώνυμο Ρ( χ) = Q(x ) - χ , που είναι πολυώνυμο της μορφής QJ (κ) αχ2+βχ+γ έχει τρεις διαφορετικές πραγματικές ρίζες μ,λ,κ. Άρα το πολυώνυμο Ρ(χ) είναι το μηδενικό πολυώνυμο. Έτσι Q( χ) = χ για κάθε χ Ε JR . Επομένως η δοθείσα εξίσωση γράφεται: χ = χ 2 ( 1 ) και ( 1 ) <::::> χ 2 - χ = 0 <=> χ (χ - 1 ) = 0 <=> χ Ε { 0, 1} . Θ Ε Μ Α 4" : Δ, λ , Ρ( ) ι4 13 ινονται τα πο υωνυμα χ = α14χ + α13Χ + ... + α 1 χ + α0 τα οποία ισχύει ότι Ρ(χ) = Q(x) για κάθε χ Ε 9t . α) Να βρεθεί το α0 • β) Ν α υπολογισθεί το άθροισμα: α1 + α2 + ... + α14 .

και Q(x) = (χ2 - 1)7 + (χ2 - χ - 1)7 , για

γ) Ν α υπολογισθούν τα αθροίσματα: α2 + α4 + ..• + α14 και α1 + α3 + .. . + α13 . ΛiJ nT α) Έχουμε Ρ(Ο) = α0 • Όμως P(x) = Q(x) για κάθε χ Ε 9{ . Άρα P(O) = Q(0) = (- 1)7 + (- 1) 7 = -2 . Έτσι

α0 = -2 . β) Η αριθμητική τιμή του πολυωνύμου Ρ( χ) για χ=1 είναι το Ρ(1) = α1 4 + α1 3 + . . . + α1 + α0 • Όμως

P(l) =Q(1) => <χι4 +<χι3 + . . . +<χι +<χσ =(-1)7 =><χι4 +<χι3 + ... +<χι +<χο =-1 => α,4 -ΉΧ13 + . . . -tσ, -2=-1=> =><χι4 +<χι3 + . . . +<Χι = 1 . γ) Η αριθμητική τιμή του πολυωνύμου Ρ(χ) για χ=-1 είναι το

Ρ(-1) =<χι4 -<Χι3 +<χι2 -• • • -<χι +<χσ = (<χι4 +<χι2 + . . . +<χσ)-(<Χι3 +<χι ι + . . . <Χι ) . Όμως Ρ( -1) = Q( -1) = [( -1)2 γ = 1 . => (α1 4 + α1 2 + . . . + α0 ) - (α1 3 + α1 1 + . . . α1 ) = 1 => (α1 4 + α1 2 + . . . + α2 ) - (α1 3 + α1 1 + . . . α1 ) = 3 (αφού αο=-2) . Από το ερώτημα β) έχουμε α1 4 + α1 3 + . . . + α1 = 1 => (α1 4 + α1 2 + . . . α2 ) + (α1 3 + α1 1 + . . . + α1 ) = 1 . Εάν θέ­σουμε Κ= α2 + α4 + . . . + α1 4 και Λ= α1 + α3 + . . . + α1 3 παίρνουμε το σύστημα: Κ-Λ=3 και Κ+Λ=1 . Η

λύση του συστήματος είναι Κ=2 και Λ=-1 . Άρα α2 + α4 + . . . + α1 4 = 2 και α1 + α3 + . . . + α1 3 = -1 . ΕΥΚΛΕΙΔΗΣ Β' 70 τ.2/36

Page 39: Ευκλειδης Β 70

Μαθηματικά για την Β · Λυκείου

Πολυώνυμο Ρ(χ) με σταθερό όρο το -1 , διαιρείται με το (χ+α) και δίνει πηλίκο (χ2+3χ+4), ενώ όταν διαιρείται με το (χ+β) δίνει πηλίκο (χ2+4χ+2). Να βρεθούν τα α, β και στη συνέχεια το πολυώνυμο Ρ( χ).

Αφού ο σταθερός όρος του Ρ(χ) είναι ίσος με -1 θα έχουμε ότι Ρ(Ο)=-1 . Η ταυτότητα της διαίρεσης του Ρ(χ) με το (χ+α) θα είναι : Ρ(χ)=(χ2+3χ+4)(χ+α)+υ1 . Τότε Ρ(Ο)=4α+υ1 • Άρα 4α+υ1=-1 . ( 1 ) Επίσης η ταυ­τότητα της διαίρεσης του Ρ(χ) με το (χ+β) θα είναι: Ρ(χ)=(χ2+4χ+2)(χ+β)+υ2 • Έτσι Ρ(Ο)=2β+υ2 • Επομέ­νως παίρνουμε ότι 2β+υ2=-1 . (2) Το πολυώνυμο Ρ(χ) θα έχει τη μορφή : Ρ(χ)=κχ3+λχ2+μχ-1 . Από την ταυτότητα της διαίρεσης του Ρ(χ) με το (χ+α) και από την (1) παίρνουμε ότι: Ρ(χ)=χ3+(α+3)χ2+(3α+4)χ-1 . Άρα πρέπει να έχουμε : α+3=λ και 3α+4=μ. Από την ταυτότητα της διαίρεσης του Ρ(χ) με το (χ+β) και από την (2) παίρνουμε ότι: Ρ(χ)=χ3+(4+β)χ2+(4β+2)χ-1 . Άρα πρέπει να έχουμε : 4+β=λ και 4β+2=μ. Από τις τέσσερις τελευταίες εξισώσεις προκύπτει το σύστημα: α+3=4+β και 3α+4=4β+2 από όπου βρίσκουμε α=6 και β=5 . Έτσι, αντικαθιστώντας αυτές τις τιμές των α και β σε μία από τις δύο ταυτότητες διαίρεσης, προκύπτει ότι το πολυώνυμο Ρ(χ) είναι το Ρ(χ)=χ3+9χ2+22χ-1 .

Ν α βρεθεί πολυώνυμο Ρ( χ) 1 ου βαθμού το οποίο να ικανοποιεί τη σχέση : Ρ [ Ρ2 (χ) J - 2Ρ(χ + 1) = 27χ2 + 12χ - 5 (1) για κάθε χ Ε JR.

Έστω Ρ(χ) = αχ + β, α :;ι Ο. Έχουμε Ρ2 (χ) = (αχ + β)2 = α2χ2 + 2αβχ + β2 και Ρ( χ + l ) = α(χ + 1) + β = αχ + α + β. Επομένως

( 1 ) <=> Ρ [Ρ2 (χ)] - 2Ρ(χ + 1) = 27χ2 + 1 2χ - 5 <::::> Ρ(α2χ2 + 2αβχ + β2 ) - 2αχ - 2α - 2β = 27χ2 + 1 2χ - 5

<=> α(α2χ2 + 2αβχ + β2 ) - 2αχ - 2α - 2β = 27χ2 + 12χ - 5 <=> α3χ2 + 2α2βχ + αβ2 - 2αχ - 2α- 2β = 27χ2 + 1 2χ - 5 <::::> α3χ2 + (2α2β - 2α)χ + αβ2 - 2α - 2β = 27χ2 + 12χ - 5 {α3 = 27

α= 3 Για να ισχύει η ( 1 ) για κάθε χ Ε JR πρέπει και αρκεί: 2α2β -2α = -7 δηλαδή { . Άρα Ρ ( χ ) = 3 χ + I .

β = 1 αβ2 -2α-2β = 1 8

α) Δίνεται το πολυώνυμο Ρ ( χ) = (3χ3 - 2χ2 + 5χ - 7)2008 + (6χ5 - 7χ4 + 2χ3 + 8χ - 8)2009 • Να βρεθεί το άθροισμα των συντελεστών του πολυωνύμου.

β) Έστω τα τυχαία πολυώνυμα Ρ(χ) και Q(x) καθένα από τα οποία έχει άθροισμα συντελεστών ίσο με 2. Να βρεθεί το άθροισμα των συντελεστών του γινομένου τους.

α) Έστω Ρ( χ) = αν χ ν + αν-Ι χ ν- Ι + . . . + αι χ + αο , ν Ε Ν και αί Ε IR, ί=Ο, 1 ,2, . . . ν, ν Ε Ν. Τότε

αν + αν-ι + . . . + α ι + αο = Ρ(1) = (3 - 2 + 5 - 7)2oos + ( 6 - 7 + 2 + 8 - 8)2009 = ( -1 )2oos + 1 2009 = 1 + 1 = 2 . β) Είναι P(l) = 2 και Q(l) = 2 . Άρα εάν R(x) = Ρ( χ ) · Q(x) τότε R(l) = P(l) · Q(l) = 2 · 2 = 4 .

Δίνεται πολυώνυμο Ρ(χ) το οποίο διαιρούμενο με το (χ-1) δίνει υπόλοιπο 14 ενώ έχει το (χ-3) παρά­γοντα. Να βρεθεί το υπόλοιπο της διαίρεσης του Ρ(χ) δια του Q(x) = χ2 - 4χ + 3.

Το πολυώνυμο Q(x) παραγοντοποιημένο γράφεται στη μορφή : Q(x) = (χ - l)(x - 3). Αφού το Q(x) είναι πολυώνυμο δευτέρου βαθμού το υπόλοιπο της διαίρεσης θα είναι πολυώνυμο της μορφής υ(χ) = αχ + β, α,β Ε IR. Άρα Ρ(χ) = (χ - 1)(χ - 3)π(χ) + υ(χ), για κάθε x E IR όπου π(χ) το πηλίκο της

P(l) = 14 � 14 = α+ β διαίρεσης. Δηλαδή Ρ(χ) = (χ - 1)(χ - 3)π(χ) + αχ + β για κάθε χ E IR Έτσι έχουμε:

Ρ(3) = ο � ο = 3α+ β Άρα α = -7, β=2 1 . Οπότε: υ( χ ) = -7χ + 2 1 .

r · . Γ' . : ' �· , I ,

Έστω τα πολυώνυμα Ρ(χ) και Q(x) για τα οποία ισχύει η σχέση

ΕΥΚΛΕΙΔΗΣ Β' 70 τ.21/37

Page 40: Ευκλειδης Β 70

Μαθηματικά για την Β " Λυκείου

Q(x) = [Ρ(2χ2 - 3χ - 4) τ + 2χ2 - 3χ + α, για κάθε χ Ε JR, όπου α Ε JR. Εάν το υπόλοιπο της διαίρε­σης Ρ( χ) : (χ - 1) είναι 3 να προσδιοριστεί η τιμή της παραμέτρου α αν είναι γνωστό ότι το υπόλοιπο της διαίρεσης Q(x) : (χ + 1) είναι επίσης 3. ΛίJση : Αφού το υπόλοιπο της διαίρεσης Ρ( χ) : (χ - 1) είναι 3 τότε P(l) = 3 . Επίσης έχουμε Q( - 1) = 3 . Άρα παίρνουμε ότι: Q( -1) = [ Ρ(2( -1)2 - 3( - 1) - 4) ] 2 + 2( - 1)2 - 3( - 1) + α => 3 = [P(l) ]2 + 2 + 3 + α => 3 = 9 + 2 + 3 + α => α = -1 1 . Θ Ε Μ Α 1 0" : Δίνεται το πολυώνυμο Ρ ( χ) = χ3 + κχ2 - λχ + μ, κ,λ,μ Ε JR. Εάν είναι γνωστό ότι το πολυώνυμο Q(x) = χ2 - 2χ + 1 διαιρεί το Ρ( χ) , τότε να δείξετε ότι κ+μ=-4. ΛίJση : Είναι Q(x) = (χ - 1)2 • Αφού το Q(x) διαιρεί το Ρ(χ) τότε με σχήμα Horner έχουμε:

I : Ι 1� Ι κ�! Ι κ�;!:�� Ι ι Ι Το υπόλοιπο της διαίρεσης είναι Ο . Άρα λ+μ+l=κ (1) . Επίσης, το υπόλοιπο της διαίρεσης του πηλίκου

διαί ε , ε (χ-1) είναι Ο. Χρησιμοποιώντας πάλι σχήμα Horner έχουμε : 1 κ-λ+1 1

κ+2 1 κ+2 2κ-λ+3

Άρα 2κ-λ+3=0 (2). Προσθέτοντας τις (1) και (2) κατά μέλη παίρνουμε: 2κ+μ+4=κ δηλαδή κ+μ=-4, που είναι το ζητούμενο. Θ Ε Μ Λ 1 1 " : Δίνεται πολυώνυμο Ρ(χ) για το οποίο ισχύουν ο ι σχέσεις: Ρ(-2)=0 και Ρ(2χ+2)= Ρ(χ2 - 4) για κάθε χ Ε JR. Εξετάστε εάν το Ρ( χ) έχει παράγοντα το πολυώνυμο χ2 + 2χ. ΛίJση : Έχουμε Ρ(2χ + 2) = Ρ(χ2 - 4) για κάθε x E JR Για χ = -2 έχουμε Ρ(-4+2)=Ρ(4-4) => Ρ(-2) = Ρ(0) = 0 από την υπόθεση . Έστω υ( χ) = αχ + β το υπόλοιπο της διαίρεσης του Ρ(χ) με το χ2 + 2χ. Εάν π(χ) το

πηλίκο τότε έχουμε : Ρ(χ) = π(χ)(χ2 + 2χ) +αχ +β για κάθε χ Ε "ΙR , οπότε: => => Ρ(Ο) =Ο } Ο= β } α=Ο} Ρ(-2) =0 0=-2α+β β =Ο

Επομένως το υ ( χ) είναι μηδενικό πολυώνυμο, οπότε το χ2 + 2χ είναι παράγοντας του Ρ(χ). Θ Ε Μ Α R 2 " : α) Να βρεθεί το λ Ε JR ώστε το Ρ(χ) = χ 5 + 3χ3 - 5χ2 + 4χ - 3 να έχει ακέραια ρίζα τον αριθμό 2λ-1 . β) Για την τιμή του αριθμού λ από το α) ερώτημα να λυθεί η aνίσωση Q(x) � Ο με

Q(x) = χ4 - αχ3 - βχ2 + 5χ - 2 εάν γνωρίζουμε ότι ο αριθμός λ είναι ρίζα του πολυωνύμου Q(x) πολλαπλότητας τουλάχιστον 2.

ΛίJση : α) Πιθανές ακέραιες ρίζες του πολυωνύμου Ρ(χ) είναι οι ακέραιοι διαιρέτες του -3 , δηλαδή οι ακέραιοι

P(l) = Ο Ρ(- 1) = -1 6

± 1, ± 3 . Έχουμε Μοναδική ακέραια ρίζα είναι ο αριθμός 1 . Ρ(3) = 288 Ρ(-3) = -222

Επομένως πρέπει και αρκεί 2λ - 1 = 1, δηλαδή λ = 1 .

β) Ο αριθμός λ= 1 είναι ρίζα του Q(x) πολλαπλότητας τουλάχιστον 2, άρα Q( 1 )=0 και π( l )=Ο, όπου π(χ) το πηλίκο της διαίρεσης Q(x) : (χ - 1) . Κάνοντας την διαίρεση με σχήμα Horner έχουμε :

ΕΥΚΛΕΙΔΗΣ Β. 70 τ.2/38

Page 41: Ευκλειδης Β 70

Μαθηματικά για την Β ' Λυκείου

-α -β 5 -2 1 -α 1 -α-β 6-α-β

1 -α 1 -α-β 6-α-β 4-α-β και συνεχίζοντας το σχήμα Homer για το π_11λίκο, πάλι με διαΙQέ'[l' το (χ-11 θα έχουμε:

1 1 -α 1 -α-β 6-α-β 1 1 2-α 3-2α-β

1 2-α 3-2α-β 9-3α-2β_

Εξισώνοντας τα υπόλοιπα των διαιρέσεων με Ο παίρνουμε: δηλαδή {α + β = 4 α = 1} 3α + 2β = 9 β=3

Άρα το πολυώνυμο είναι Q(x) = χ4 - χ3 - 3χ 2 + 5χ - 2 . Για να λυθεί η aνίσωση Q(x) :::; Ο βρίσκουμε πρώτα εάν το Q(x) έχει και άλλες ρίζες. Από το δεύτερο σχήμα Homer βρίσκουμε ότι το πηλίκο της διαίρεσης Q(x) : (χ - 1)2 είναι π1 (χ) = χ2 + χ - 2 με ρίζες τους αριθμούς 1 και -2 .

Άρα Q ( χ ) :::; Ο <=> (χ - 1 ) 3 (χ + 2) :::; Ο <=> (χ - 1 ) ( χ + 2) :::; Ο <=> χ ε [-2, 1] . Θ Ε Ι\ Ι Α 1 3 " : Να λυθεί η εξίσωση : χ2 - 3χ + .Jx2 - 3χ - 6 = 8 (1). λιΊση : Για να ορίζεται η (I) πρέπει και αρκεί:

( Ι) <=> <:::> x2 - 3x - 6 = y2 (2) .Jx2 - 3x - 6 = y } y ;;::: O } ( 1 ) χ 2 - 3x + .Jx2 - 3χ - 6 = 8 y2 + 6 + y = S (3)

(Π). Τότε :

Αλλά (3 ) <=> y2 + y - 2 = 0 <:::> y = 1 ή y = -2 <=> y = 1, λόγω της (Ι) . Τότε: ( 2) <:::> χ2 - 3χ - 6 = 1 <:::> χ2 - 3χ - 7 = Ο <:::> χ 3 ± Jfi . Τελικά: ( Ι ) <:::> χ = 3 ± Jfi . Τιμές δεκτές (γιατί;) 2 2

, Δ · ζ · cττ) 3 ± m δ · λ θ · Π ρ οσοzη : εν χρεια εται να αντικαταστησουμε στην το χ με 2 για να ουμε αν επα η ευε-ται (πράγμα χρονοβόρο) διότι αυτό είναι βέβαιο λόγω της (2). Φυσικά ούτε χρειάζεται να επιλύσουμε την (Π).

Θ Ε \Ι Λ i 4" Να λυθεί η εξίσωση : λ - 2χ = .J4x2 + 4 (Ι) για τις διάφορες τιμές του λε IR . ΛίJση : Η (I) ορίζεται για κάθε χ ε IR αφού 4χ2 +4 ;;::: 4>0 για κάθε χ ε IR .

λ - 2χ ;;::: ο } χ :::; 2: } χ :::; 2: } ( 1 ) Άρα (I) <=> 2 2 <=> 2 <=> 2 . (λ - 2χ ) = 4Χ + 4 λ2 - 4λχ + 4χ2 = 4χ2 + 4 λ2 - 4 = 4λχ (2) ..

Εάν λ = Ο τότε ( 2) <=> Οχ= - 4, αδύνατη . Ε ' λ ο ' (2) λ2 - 4 ' αν -:t- τοτε <=> χ=-- οποτε: 4λ

λ2 - 4 λ λ2 - 4 λ2 - 4 λ2 - 4 - 2λ2 -λ2 -4 ( 1) <=> --:::;-<=>--:::; λ<=>---λ:::; Ο<=> :::; Ο<=>-- :::; Ο<=> (λ2 + 4)2λ ;;::: Ο<:::> λ>Ο α 4λ 2 2λ 2λ 2λ 2λ φού λ 7:- Ο και λ2 + 4 > Ο για κάθε λ ε IR .

ΕΥΚΛΕΙΔΗΣ Β' 70 τ.21/39

Page 42: Ευκλειδης Β 70

Μαθηματικά για την Β ' Λυκείου

Κατσούλης Γιώργος

Στόχος του άρθρου είναι να βοηθήσει τους μαθητές στην επίλυση ασκήσεων που αναφέρονται στις με­τρικές σχέσεις και τα εμβαδά. Αρκετές ασκήσεις συνδυάζουν και τα δυο κεφάλαια και αναλύονται σε επιμέ­ρους ερωτήματα, όπου χρειάζεται βέβαια, υπάρχουν οι απαραίτητες παρατηρήσεις και επισημάνσεις.

Άσικηση ! η

Δίνεται τρίγωνο ΑΒΓ με β=8, γ=5 και Α=60°. Να υπολογίσετε α) την πλευρά α β) το εμβαδόν (ΑΒΓ) γ) το ύψος ΑΔ=υa δ) το ΑΗ, όπου Η το ορθόκεντρο του τριγώνου. Λ1Jση α) Από το νόμο των συνημίτονων έχουμε: α2=β2+/-2βγσυνΑ=82+52-2 · 8 · 5συν60° = =64+25-2· 8 · 5 ..!.. =49. Άρα α=7 2

1 1 1 J3 Λ � β) (ΑΒΓ)=-βyημΑ=-8 · 5ημ60" =-8 · 5- = 1νν3 . 2 2 2 2 Α

/ ·."> I '· , Ε

1/ ')<:>, __ _ ί / , '

ί " / " Η

I / ι ι /

Β;:_ ___ Δ - · - - - - - - Γ

. 1 Λ � 1 2oJ3 γ) Εχουμε (ΑΒΓ)=-αυα <::> 1νν3 =-7υα <::>υα =-- . 2 2 7 δ) Έστω ΒΕ=υβ. Στο τρίγωνο ΑΒΕ είναι Α =60°, Ε =90° οπότε Β 1 =30°. Άρα ΑΕ = ΑΒ = � . 2 2 Το τετράπλευρο ΔΗΕΓ είναι εγγράψιμο ( Δ + Ε = 90° + 90° = 1 80° ), οπότε ΑΗ.ΑΔ=ΑΕ.ΑΓ

�8 <::> ΑΗ = ΑΕ · ΑΓ = -2-=�=_2__ = 7J3

ΑΔ 20J3 20J3 J3 3 7 7

Σz6λ� .n · Όταν γνωρίζουμε δυο πλευρές και την πε­ριεχόμενη γωνία ενός τριγώνου, υπολογίζουμε εύ­κολα την τρίτη πλευρά και το εμβαδόν του.

Δίνεται τρίγωνο ΑΒΓ με α=34, β=20, γ=18. Να υπολογίσετε α) την προβολή της πλευράς γ πάνω στην β β) το εμβαδόν (ΑΒΓ) γ) το γινόμενο Rρ όπου R,ρ οι ακτίνες του περι­γεγραμμένου και του εγγεγραμμένου κύκλου του τριγώνου αντίστοιχα

α) Αρχικά βρίσκουμε το είδος του τριγώνου. Η μεγαλύτερη πλευρά είναι α=34 και α2= 1 1 56. Επίσης β2+γ2=202+ 1 82=400+ 324=724.

/.

' , Δ

/� ' Α

/ / -� " ., "

" �,� " '

"/ Β�.

··,

·--- "'� ·--------- ,,

- --------- ...... , ., ------- - - -::_�

Γ Άρα α2>β2+γ2<::>Α>90°. Φέρνουμε το ύψος ΒΔ. Ζητάμε το ΑΔ. Από τη γενίκευση του Πυθαγορεί­ου θεωρήματος είναι: α2=β2+γ2+2β.ΑΔ(Α>90°)<::>342=202+ 1 82+2 .20.ΑΔ <::> 1 1 56=400+324+40ΑΔ<::> ΑΔ = 432 = 54 .

β) Από τον τύπο του Ήρωνα έχουμε : 40 5

Ε = �τ ( τ - α) ( τ - β) ( τ - γ) Είναι τ = α + β + γ = 1 8 + 20 + 34 = 36 . 2 2 Άρα (ΑΒΓ) = .J36 · 2 · 1 6 · 1 8 = 1 44 ) 'Ε (ΑΒΓ) αβγ 144 34 · 1 8 · 20 R 85 γ χουμε =-<=> = <::> =- . 4R 4R 4 Επίσης ( ΑΒΓ) = τ · ρ <::> 1 44 = 36 · ρ <::> ρ = 4 . Άρα

85 R · ρ = -4 = 85 . 4 Σ;:6λ ω : Όταν γνωρίζουμε τις πλευρές ενός τριγώνου για να βρούμε την προβολή μιας πλευράς πάνω σε άλλη αρχικά βρίσκουμε το είδος του τριγώνου.

Στο σχήμα είναι ΔΓ=ΟΑ=R, ΟΜ=7 Δ M(O,R)=40.

\ \

Δ

/� I \ Χ I

και

ΕΥΚΛΕΙΔΗΣ Β' 70 τ.2/40

Page 43: Ευκλειδης Β 70

Μαθηματικά για την Β ' Λυκείου

α) Δείξτε ότι R=3 β) Υπολογίστε το ΜΓ=χ γ) Υπολογίστε τα εμβαδά (ΟΜΓ) και (ΑΜΓ) Λύση α) Έχουμε Δ M(O,R)=40�0M2 -R2=40�

�R2=72 --40�R2=9�R=3 β) Έχουμε ΜΓ·ΜΔ=ΔΜ(Ο,R)�ΜΓ(ΜΓ +Γ Δ)=40� �χ( χ+ 3)=40�χ2+ 3χ--40=0�χ=5ή χ=--8 . Άρα ΜΓ=5 (χ>Ο) γ) Το τρίγωνο ΟΓΔ είναι ισόπλευρο, οπότε f 1 =60°. Άρα Μ f Ο= 1 20° . Επομένως (ΟΜΓ) = _!_ΟΓ · ΜΓημ120° = _!_3 · 5 · .J3 = 1 5.J3 . 2 2 2 4 Επίσης τα τρίγωνα ΟΜΓ και ΑΜΓ έχουν κοινό . . Γ Ά ( ΑΜΓ) ΑΜ 4 . υψος απο το . ρα, ( ΟΜΓ)

= ΟΜ = Ί , οποτε

(AMΓ) = i (OMΓ) = i . 1 5.J3 = 1 5.J3 . 7 7 4 7 Σzόi.ιο : Για να βρούμε το εμβαδόν ενός τριγώνου, αρκεί να το συγκρίνουμε με άλλο τρίγωνο, με γνωστό εμβαδόν και να υπολογίσουμε το λόγο των εμβαδών τους.

�\. σκηση ..ι η Σε ορθογώνιο τρίγωνο ΑΒΓ (Α =90°) είναι 5 3 α = -γ και ΑΔ = U = -γ 4 α 5

α) Να υπολογίσετε το λόγο ..!!. γ

β) Α ν ΑΕ = .!. β και η προέκταση του ΒΕ τέμνει 3

τον περιγεγραμμένο κύκλο του τριγώνου στο Η

λ , λ ' ΒΕ να υπο ογισετε το ογο ΕΗ

.

Λύση

α) Ισχύει ΑΔ·ΒΓ=ΑΒ·ΑΓ � �γ�γ = γ . β � β = �γ � � = � . 5 4 4 γ 4

Α Η

Β

�ΒΕ2 = γ2 + γ2 � ΒΕ2 = 1 7γ2 � ΒΕ = γJli ( 1 ) 16 16 4 Οι χορδές ΒΗ και ΑΓ τέμνονται στο Ε, οπότε

( Ι ) γJli 1 2 ΒΕ·ΕΗ=ΑΕ·ΕΓ �--ΕΗ = -β-β � 4 3 3 γJli ΕΗ = �β2 � γJli ΕΗ = � ·!!_γ2 � 4 9 4 9 1 6 JliEH = l � ΕΗ =_γ_ = γJli (2) 2 2Jli 34

, ΒΕ 34 1 7 Επομενως - =- =- . ΕΗ 4 2 Άσκηση 5'1

Δίνεται τρίγωνο ΑΒΓ με AB=l6, ΑΓ=12, ΒΓ=20 α) Δείξτε ότι το τρίγωνο είναι ορθογώνιο β) Υπολογίστε το ύψος ΑΔ γ) Αν Ε το μέσο του ΑΔ, υπολογίστε το (ΑΒΕ) δ) Αν ΓΖ 1. ΒΕ , υπολογίστε το γινόμενο ΒΕ·ΒΖ Λύση α) ΑΒ2+ΑΓ2=1 62+ 1 22=256+ 144=400=202=ΒΓ2 Άρα Α =90°

β) Ισχύει

Α

� Β Δ Γ

ΑΒ·ΑΓ=ΑΔ·ΒΓ�ΑΔ ΑΒ·ΑΓ _ 16 · 12 _ 16 · 3 _ 48 ( 1 ) ΒΓ -2ο --5- -S , 1 l ΑΔ ΑΔ · ΒΔ γ) Εχουμε (ΑΒΕ) =-ΑΕ · ΒΔ =--ΒΔ = (2)

Επίσης 2 2 2 4

ΑΒ2 = ΒΓ · ΒΔ � ΒΔ = ΑΒ2 = 1 62 = 256 = 64 (3) ΒΓ 20 20 5 48 64

Από ( 1 ), (2), (3) είναι (ΑΒΕ) = 5 . 5 = 768 4 25 δ) Το τετράπλευρο ΔΕΖΓ είναι εγγράψιμο (Δ + Ζ =90° +90° = 1 80°), οπότε Β Ε-ΒΖ= ΒΔ· ΒΓ=ΑΒ2= 1 62=256.

Άσκηση 6'1 Δ Λ

Δίνεται τρίγωνο Α Β Γ ( Α <90°) εγγεγραμμέ-νο σε κύκλο (0, R) με ΑΒ=5, ΑΓ=21 και ΒΓ=19. α) Δείξτε ότι Α =60° β) Υπολογίστε την ακτίνα R

Δ Λ γ) Αν ΒΔ=υp, υπολογίστε το ΟΔ

β) Στο τρίγωνο Α Β Ε ( Α =90°) έχουμε Λύση

ΒΕ2=ΑΒ2+ΑΕ2�ΒΕ2= γ2 +�β2�ΒΕ2 =γ2 +� ·J__γ2 α) Από το νόμο των συνημίτονων έχουμε 9 9 16

ΕΥΚΛΕΙΔΗΣ Β' 70 τ.21/41

Page 44: Ευκλειδης Β 70

Μαθηματικά για την Β ' Λυκείου

α2 = β2 + γ2 - 2βγσυνΑ <::::> β2 + γ2 - α2 2 12 + 52 - 1 92

συνΑ = = 2βγ 2 . 2 1 · 5

= 44 1 + 25 - 36 1 = 1 05 = .!_ . Ά α: Α = 60ο •

2 1 0 2 1 0 2 ρ

Γ

β) 'Εχουμε : {ΑΒΓ) = .!_β · γημΑ = .!_2 1 · 5ημ60° = 2 2

= .!.. 2 1 · 5 �

= 1 05� ( 1 )

2 2 4

Επίσης: {ΑΒΓ) = αβγ � 1 05� = 1 9 · 2 1 · 5 <=>

4R 4 4R

<::::> � = !2_ <::::> R = ..!.2_ = 1 9�

. R � 3

γ) Επειδή Δ εσωτερικό σημείο του κύκλου (0, R) ισχύει ΔΑ·ΔΓ=R2-ΟΔ2 (2) Στο τρίγωνο ΑΒΔ είναι Δ =90° Α =60° οπότε

Β =30° Άρα ΑΔ = ΑΒ = � ( 3) Επομένως 2 2

5 37 ΔΓ = ΑΓ - ΑΔ = 2 1 - - = - {4) . 2 2

Από (2), (3), (4) και το β) ερώτημα έχουμε:

� . 37 = (..!.2_)2 - ΟΔ2 <::::> 1 85 = 36 1 _ 0Δ2 <=> 2 2 � 4 3

<=> ΟΔ2 = 36 1 _ 1 85 = 889 . Άρα ΟΔ = {889 . 3 4 1 2 'J12

�zόλιο : Είναι β2 > α2 + γ2 , οπότε Β > 90° λ. σκη ση 7'1 Θεωρούμε ορθογώνιο τρίγωνο ΑΒΓ

( Α =90°), Δ το μέσο της ΑΒ και ΔΕ .l ΒΓ . Α ν

(mΕ} =�(ΔΕΓ} aποδείξτε ότι: 5

ΛίJ ση

α2 .J3 α) 2γ = .J3α β) (ΑΒΓ) = -- .

8

α) Το τετράπλευρο ΑΔΕΓ είναι εγγράψιμο (Α+Ε=90°+90°= 1 80°) οπότε

ΒΔ·ΒΑ=ΒΕ·ΒΓ <::::> lγ = ΒΕ · α ( 1 ) 2

Τα τρίγωνα ΒΔΕ και ΔΕΓ έχουν κοινό ύψος ΔΕ

. {ΒΔΕ) ΒΕ vπoe . 3 ΒΕ ΒΕ οποτε = - <::::> - = - <::::> ---{ΔΕΓ) ΕΓ 5 ΕΓ ΒΕ + ΕΓ

3 ΒΕ 3 3 = - <=> - = - <=> ΒΕ = -α (2) 5 + 3 ΒΓ 8 8

Β

Από ( 1 ) και (2) είναι γ2 3α2 2 3α2 α� - = - <::::> γ = - <::::> γ = - <::::> 2γ = α� 2 8 4 2

β) Επειδή Α =90° είναι α2=β2+γ2<::::> (α) 3α2 α2 α β2 = α2 _ γ2 <::::> β2 = α2 - - <::::> β2 = - <=> β = -

4 4 2

Άρα {ΑΒΓ) = .!.. βγ = .!.. . � . �α = α2 �

2 2 2 2 8

Σημε ίωση : Αφού β = � , είναι Β = 30° . 2

λ. σκη ση S'l Από το μέσο Μ, χορδής ΑΒ, κύκλου (0, R)

φέρνουμε τυχαία ευθεία που τέμνει τον κύκλο στα Γ και Δ. Ν α αποδείξετε ότι: α) ΑΒ2=4ΜΓ.ΜΔ β) ΑΓ2+ΒΓ2+ΒΔ2+ΑΔ2=2ΓΔ2 γ) Αν επιπλέον ισχύει ότι ΜΔ=ΑΒ τότε ΑΔ·ΔΒ=4ΑΓ·ΒΓ Λ ίJ ση α) Οι χορδές ΑΒ και Γ Δ τέμνονται στο Μ οπότε

ΜΑ·ΜΒ=ΜΓ·ΜΔ

<::::> ΑΒ · ΑΒ = ΜΓ · ΜΔ <::::> ΑΒ2 = 4ΜΓ · ΜΔ 2 2

Α

β) Στο τρίγωνο ΑΒΓ είναι � + Br =2ΙΜ2 + Air ( 1 ) 2

Στο τρίγωνο ΑΒΔ είναι Β& +Μ =2ΔΜ2 + Air (2) 2

Με πρόσθεση κατά μέλη έχουμε :

ΕΥΚΛΕΙΔΗΣ Β' 70 τ.2/42

Page 45: Ευκλειδης Β 70

Μαθηματικά για την Β ' Λυκείου

ΑΓ2 + ΒΓ2 + ΒΔ2 + ΑΔ2 = 2ΓΜ2 + 2ΔΜ2 + ΑΒ2 (α) = 2ΓΜ2 + 2ΔΜ2 + 4ΜΓ · ΜΔ =

= 2 (ΓΜ2 +ΔΜ2 + 2ΜΓ · ΜΔ) = 2(ΜΓ + ΜΔ)2 = 2ΓΔ2

, (ΑΒΓ) ΑΓ · ΒΓ γ) Ε χουμε ( ΑΒΔ)

= ΑΔ . ΔΒ

(3)

(διότι AfB + ΑΔΒ = 1 80° )

Επίσης �ΑΒΓj =

ΓΗ (διότι ΑΒ κοινή βάση) ΑΒΔ ΔΚ

Αλλά ΓΗ = ΜΓ (διότι Μ r Η � Δ Μ Κ ), οπότε ΔΚ ΜΔ

(ΑΒΓ) ΜΓ (4) Από (3), (4) προκύπτει ότι (ΑΒΔ) ΜΔ ΑΓ · ΒΓ = ΜΓ = ΜΓ = _!_ <::::> ΑΔ · ΔΕ = 4ΑΓ · ΒΓ ΑΔ · ΔΒ ΜΔ 4ΜΓ 4 '

διότι από ΑΒ2=4ΜΓ.ΜΔ έχουμε ΜΔ=4ΜΓ (αφού ΜΔ=ΑΒ)

\ -'C κηση 9 '1 Σε οξυγώνιο τρίγωνο ΑΒΓ ισχύει

α2=2ΑΗΑΔ, όπου ΑΔ ύψος και Η το ορθόκε-

ντρο. Δείξτε ότι: α) μ.. = aJ3 β) 2ΑΗ2=ΒΗ2+ΓΗ2 2

. \ {Ιση α) Έστω ΒΕ=υβ. Το τετράπλευρο ΔΗΕΓ είναι εγ­γράψιμο ( Δ + Ε =90°+90°= 1 80°) οπότε ΑΗ-ΑΔ=ΑΕ-ΑΓ ( 1 ) Επίσης Α<90°, οπότε α2=β2+γ2-2ΑΓ·ΑΕ<::::>α2=β2+γ2-2ΑΗ·ΑΔ<::::> <::::>α2=β2+γ2 -α2<::::>β2+γ2=2α2 (2)

α2 (2) α2 Αλλά β2 + γ2 = 2μ� + l<::::> 2α2 = 2μ� + 2 <=>

<::::> 4α2 = 4μ� + α2 <::::> 4μ� = 3α2 <::::> μα = αJ3 2

Α

Β Δ Γ β) Πρέπει 2ΑΗ2=ΒΗ2+ΓΗ2<::::>2ΑΗ2= =(ΒΔ 2+ΔΗ2)+(Γ Δ 2+ΔΗ2) <::::>2ΑΗ2 -2ΔΗ2= =ΒΔ2+ΓΔ2<::::>2ΑΗ2 -2ΔΗ2=(γ2-ΑΔ2)+(β2-ΑΔ2) <::::> ( 2 ) <::::>2ΑΗ2-2ΔΗ2+2ΑΔ2=β2+γ2 <=> ΑΗ2-ΔΗ2+ΑΔ2=α2

(ΑΗ+ΔΗ)(ΑΗ-ΔΗ)+ΑΔ2=α2<::::>ΑΔ(ΑΗ-ΔΗ)+ΑΔ2=α2<::::> ΑΔ(ΑΗ-ΔΗ+ΑΔ)=α2<::::>ΑΔ(2ΑΗ)=α2<::::>α2=2ΑΗΑΔ που ισχύει από υπόθεση

ί\ σ κη σ η 1 () '1 Ένα πολεμικό πλοίο, ξεκινώντας από μια

αρχική θέση Α, παίρνει εντολή να κινηθεί 9 μί­λια νότια, στη συνέχεια 5 μίλια δυτικά και τέλος ξανά 3 μίλια νότια. Πόσο απέχει από την αρχική του θέση ; Λύση Έστω Β, Γ οι ενδιάμεσες θέσεις και Δ η τελική θέ­ση του πλοίου . Θέλουμε να υπολογίσουμε την α­πόσταση ΑΔ=ΑΚ +ΚΔ. Έστω ΚΓ=χ, τότε ΒΚ=5-χ. Τα τρίγωνα ΑΒΚ και ΚΓΔ είναι όμοια,

, ΑΒ ΒΚ ΑΚ 9 5 - χ ΑΚ οποτε - = - = - <=> - = -- = - . ΓΔ ΚΓ ΚΔ 3 χ ΚΔ

Α

9

Β --, 5-χ I ο... _ _ _ _ _ _

Μ

Άρα S - x = 3 και ΑΚ=3ΚΔ ( 1 ) χ

5 - χ 5 Είναι -- = 3 <::::> 5 - χ = 3χ <::::> 4χ = 5 <::::> χ = - . χ 4

Στο τρίγωνο ΚΓ Δ {Γ = 90° ) είναι

ΚΔ2 = Γ Δ2 + ΚΓ2 <=> ΚΔ2 = 32 + χ 2 = 9 + 25 = 1 69 . 1 6 1 6

Άρα ΚΔ = � (2) . Από ( 1 ), (2) είναι ΑΚ = 3 9 4 4

' 1 3 39 52 Άρα ΑΔ=ΑΚ+ΚΔ = - + - = - = 1 3 . 4 4 4

Άρα το πλοίο απέχει 1 3 μίλια από την αρχική του θέση .

2"ς τρόπος : Προεκτείνουμε την ΑΒ και φέρνουμε παράλληλη από το Δ προς τη ΒΓ. Στο τρίγωνο ΑΜΔ είναι ΑΜ=9+3= 1 2, ΜΔ=ΒΓ=5 , οπότε ΑΔ2=ΑΜ2+ΜΔ2= 1 22+52= 1 69 . Άρα ΑΔ= 1 3

ΕΥΚΛΕΙΔΗΣ Β' 70 τ.21/43

Page 46: Ευκλειδης Β 70

Μαθηματικά για την Β ' Λυκείου

Εύρεση πρωτευόντων στοιχείων τριγώνου από δευτερεύοντα στοιχεία του.

Επιμέλεια: Κώστας Βακαλόπουλος

Ε Ι ΣΑ ΓΩ Γ Η

Με αφορμή την άσκηση 2 (σελ. 65) του σχολικού βιβλίου της κατεύθυνσης των Μαθηματικών της Β ·

Λυκείου μια ομάδα μαθητών αποτελούμενη από τους : Χρήστο Τριανταφύλλου, Νατάσα Λέκκου, Μαρία

Φρογουδάκη έψαξαν και βρήκαν στην βιβλιογραφία τους ασκήσεις στις οποίες ζητούνται τα κύρια στοιχεία

ενός τριγώνου και συγκεκριμένα οι εξισώσεις των πλευρών και οι κορυφές του, όταν γνωρίζουμε δευτερεύ­

οντα στοιχεία του, όπως τα ύψη, οι διάμεσοι και οι διχοτόμοι. Οι μαθητές τις έλυσαν, τις έκαναν μια «πα­

ρέα ασκήσεων» εμείς τις επεξεργαστήκαμε και σας τις παρουσιάζουμε για την ωφέλεια όλων.

Άσκηση J 'l Να βρεθούν οι εξισώσεις των πλευρών τριγώνου ΑΒΓ αν είναι Α(ι,3) και χ - 2y+ ι =Ο , y - ι = Ο οι εξισώσεις των διαμέσων του. ΛίJση

Α •

-----

Έστω ΑΒΓ το τρίγωνό μας. Επειδή οι συντεταγμέ­νες του Α δεν επαληθεύουν τις εξισώσεις των δια­μέσων, οι εξισώσεις που δίνονται, αυτές αντιστοι­χούν στις άλλες διαμέσους , έστω ΒΔ: χ - 2y + 1 =

Ο και ΓΕ: y - 1 = Ο. Έστω B(xz, Yz) και Γ(χ3 , YJ) . (Προφανώς αν βρούμε τις συντεταγμένες των Β, Γ

. . . τελειώσαμε !)

181 Το Β ανήκει στην ΒΔ άρα: χ2 - 2yz + 1= Ο ( 1 )

Αν Ε το μέσο της ΑΒ είναι ε ( 1 +2χ2 , 3 +/2 )

και ανήκει στην ΓΕ. Άρα επαληθεύει την εξίσωσή

της, δηλαδή : 3 + Υ2 - 1 = Ο (2) 2

Λύνοντας το σύστημα των ( 1 ), (2) έχουμε: Yz = - 1 , Xz = - 3 , άρα Β(- 3, - 1 ) . ι:8J Το Γ ανήκει στην ΓΕ άρα: y3 - 1 = Ο (3)

Αν Δ το μέσο της ΑΓ είναι Ε ( 1 +2χ3 , 3 +/3 ) και

ανήκει στην ΒΔ. Άρα επαληθεύει την εξίσωσή της,

δηλαδή : 1 + χ3 - 2 · 3 + y3 + 1 = 0 (4) 2 2

Λύνοντας το σύστημα των (3), (4) έχουμε:

Υ3 = 1 , χ3 = 5, άρα Γ(5 , 1 ) . Από τα σημεία A( l , 3), Β(-3 , - 1 ), Γ(5 , 1 ) προσδιο­ρίζουμε τις εξισώσεις των πλευρών του τριγώνου : ΑΒ : χ - y + 2 = 0 , ΒΓ: x - 4y - 1 = 0 και ΑΓ: χ + 2y - 7 = Ο . (Αν αρχικά είχαμε υποθέσει για τις

διαμέσους ότι ΓΕ: χ - 2y + Ι = Ο και ΒΔ: y - Ι = Ο τα σημεία θα ήταν: Γ(-3, -1), Β(5, 1) και οι εξισώ­σεις των πλευρών ΑΓ: χ - y + 2 = Ο , ΒΓ:

x - 4y - 1 = 0 και ΑΒ: x + 2y - 7 = 0) .

Άσκηση 2 ' � Να βρεθούν οι εξισώσεις των πλευρών του τρι­γώνου ΑΒΓ αν είναι γνωστή η κορυφή του Γ(S,-2) και οι εξισώσεις της διχοτόμου του ΑΔ: 3x+y - ι2 = Ο και της διαμέσου του ΑΜ: l lx + 6y - 58 = Ο Λύση Έστω το τρίγωνό μας ΑΒΓ. Για την κορυφή Α τα

πράγματα είναι εύκολα! Λύνουμε το σύστημα των εξισώσεων του ΑΜ και ΑΔ και την προσδιορίζου­με : { 3x + y - 1 2 = 0 }

1 1χ + 6y - 58 = Ο � . . . Α(2, 6)

ΕΥΚΛΕΙΔΗΣ Β' 70 τ.2/44

Page 47: Ευκλειδης Β 70

Μαθηματικά για την Β ' Λυκείου

Θεωρούμε το συμμετρικό του Γ ως προς την δι­

χοτόμο ΑΔ. Το σημείο αυτό Γ' θα είναι σημείο

της ΑΒ (γιατί;) Το Γ' προσδιορίζεται κατά τα

γνωστά (βλέπε εφαρμογή 2 σελ. 63, σχολικού βι­

βλίου) . . .. Γ'(2, - 4)

Έτσι, μέχρι στιγμής μπορούμε να βρούμε τις εξι­σώσεις των πλευρών ΑΓ (από τα Α, Γ) και της ΑΒ (από τα Α, Γ). Πράγματι:

. . . . ΑΓ : 4χ +3y - 26 = Ο . . . . ΑΒ : χ = 2

(Συγνώμη, αλλά υποθέτουμε ότι ξέρετε να βρίσκετε

την εξίσωση ευθείας όταν γνωρίζετε δύο σημεία

τους.) Στην συνέχεια εργαζόμαστε όπως στην προηγούμενη άσκηση : Έστω Β(χ2, y2) . Το Β ανή­κει στην ΑΒ, άρα χ2 = 2 ( 1 ) . Επίσης, αν μέσο Μ

της ΒΓ τότε θα είναι Μ ( χ 2 2+ 8 , y2 +

2

(-2) ) και

επειδή ανήκει στην ΑΜ επαληθεύει την εξίσωσή χ + 8 y - 2

της. Άρα: 1 1 · 2 + 6 · 2 - 58 = Ο (2) 2 2

Λύνοντας το σύστημα των ( 1 ), (2) έχουμε : χ2 = 2, Υ2 = 3 , άρα Β(2, 3)

Από τα σημεία Β και Γ προσδιορίζουμε την εξί­σωση της ΒΓ: 5χ + 6y - 28 = Ο .

Να βρεθούν οι εξισώσεις των πλευρών του τρι­γώνου ΑΒΓ αν είναι Α(4, -1) και x-y-1 = Ο, χ - 1 = Ο οι εξισώσεις των διχοτόμων δύο γω­νιών του (εσωτερικές ή εξωτερικές).

Έστω ΑΒΓ το τρίγωνό μας. Επειδή το Α δεν επαλη­θεύει τις εξισώσεις τους, οι διχοτόμοι που δίνονται είναι των γωνιών Β και Γ. (Στο σχήμα έχουμε υποθέ­

σει ότι οι διχοτόμοι είναι εσωτερικές, ομοίως αν ήταν

εξωτερικές) Έστω: ΒΔ:χ-y-1 =Ο και ΓΕ:χ-1 =Ο

� Υπολογίζουμε κατά τα γνωστά το συμμετρικό του Α ως προς την ΒΔ: χ - y-1=0 . . . . . Α1 (-0, 3 )

[,(] Υπολογίζουμε επίσης το συμμετρικό του Α ως προς την ΓΕ : χ - Ι = 0 . . . . . Α2 (-2, - 1) .

Από τα Α 1 , Α2 βρίσκουμε την εξίσωση της ΒΓ: 2χ - y + 3 = Ο

! Τώρα τα πράγματα είναι εύκολα:

Λύνοντας το σύστημα · {ΒΔ · χ - y - 1 = Ο }

ΒΓ : 2χ - y + 3 = Ο

προσδιορίζουμε το Β(-4, -5) και το σύστημα

{ΓΕ : χ - 1 = 0 } Br . 2 _ προσδιορίζουμε το Γ( l , 5)

. x -y+3 - 0

Από τα σημεία Α και Β προσδιορίζουμε την εξί­σωση της . . . ΑΒ : 2x+y-7=0 και από τα Α και Γ προσδιορίζουμε την εξίσωση της . . . ΑΓ:χ-2y-6=0

Δίνεται τρίγωνο ΑΒΓ με Α(3, -1) και ορθόκε­ντρο Η( 4, -1 ). Η εξίσωση της μεσοκαθέτου της πλευράς ΑΒ είναι: x+4y=16. Να βρεθούν οι εξι­σώσεις των πλευρών του τριγώνου.

·:\ ".-, ,. .. : Αν ε η μεσοκάθετος της ΑΒ έχουμε :

λ ε · λ ΑΒ = - 1 <::::> λ ε = 4 .

ΕΥΚΛΕΙΔΗΣ Β' 70 τ.21/45

Page 48: Ευκλειδης Β 70

Μαθηματικά για την Β ' Λυκείου

Άρα η εξίσωση της ΑΒ είναι: y + 1 = 4(χ - 3) <:::> 4χ - y - 1 3 = Ο

λ ΙJ - 1 1 ,, ' I

1 ·- -

• - - -�J Ε�χ 2 · "'2 Ι _i

- - Ε � ,)

Αφού το Β ανήκει στην ΑΒ θα ισχύει: ; 4χ2 - y2 - 1 3 = ο ( 1 ) .

_ - �

Όμως το Μ είναι μέσο της ΑΒ οπότε ( 3 + X z Yz - 1 ) Μ ,-- και 2 2

αφού ανήκει στην ε, θα

ισχύει: 3 + χ 2 + 4 . Υ 2 - 1

- 1 6 = Ο (2) 2 2

Από την λύση του συστήματος των ( 1 ) και (2) προκύπτει ότι χ2 = 5 και y2 = 7, άρα Β(5 ,7)

7 + 1 Άρα ΒΕ: y + 1 = -- (χ - 4) <:::> 8χ - y - 33 = Ο 5 - 4

1 Επίσης, λΑr = - - αφού ΑΓ j_ ΒΕ και

8 1

ΑΓ: y + 1 = = - - (χ - 3) <:::> χ + 8y + 5 = Ο 8

ΑΔ: y + 1 = Ο ·( χ - 3) <:::> y = - 1 Αφού ΑΔ j_ ΒΓ και ΑΔ: y = - 1 , η ΒΓ έχει εξίσω­ση : χ = 5 .

λσκηση ::;ι ι Να βρεθούν οι εξισώσεις των πλευρών του τρι­γώνου ΑΒΓ, αν είναι Β(2, -2) και χ - 3y + 2 = Ο, χ + y + 2 = Ο οι εξισώσεις μιας εσωτερικής δι­χοτόμου του και ενός ύψος του αντιστοίχως, που άγονται από διαφορετική κορυφή. ΛίJση Επειδή οι συντεταγμένες του σημείου Β δεν επαλη-

θεύουν τις παραπάνω εξισώσεις η διχοτόμος και το ύψος που δίνονται δεν είναι από την κορυφή Β.

-� Β

.ι. _c - - - - - ·:C' -B i 2 - 2 1 - - �

Γ

Έστω η (εσωτερική) διχοτόμος ΑΔ: χ - 3y + 2 = Ο και το ύψος ΓΖ: χ + y + 2 = Ο του τριγώνου ΑΒΓ. Εί­ναι ΓΖUΒ, άρα: λrτλΑΒ=---1 <::>--1 -λΑΒ=---1 <:::>λΑΒ= 1 . Άρα η εξίσωση της ΑΒ είναι:

y + 2 = χ - 2 <:::> χ - y - 4 = Ο.

Οι συντεταγμένες του σημείου Α θα είναι η λύση του συστήματος των εξισώσεων των ευθειών ΑΒ { χ - y -4 = ο } {χ = 7} και ΑΔ: <:::> άρα Α(7,3)

x -3y + 2 = 0 y = 3

Στη συνέχεια εργαζόμαστε κατά τα γνωστά για να βρούμε το συμμετρικό σημείο Β ' του Β ως προς την διχοτόμο ΑΔ οπότε Β '(Ο, 4) .

Επειδή Β ' είναι σημείο της ΑΓ (γιατί;) η εξίσωση της ΑΓ θα είναι η εξίσωση της ΑΒ ' .

4 - 3 Έτσι: ΑΓ: y - 4 = -- (χ - Ο ) <:::> χ + 7y - 28 . 0 - 7

Λύνοντας το σύστημα των εξισώσεων των ευθειών ΑΓ και ΓΖ προσδιορίζουμε το σημείο

Γ: {χ + 7y - 28 = Ο}<=> {χ = -7}

άρα Γ(-7,5) . x + y + 2 = 0 y = 5

Τέλος από τα σημεία Β και Γ προσδιορίζουμε την 5 + 2 εξίσωση της ΒΓ: y+2=-- (x-2)<:::>7x+9y+4=0. 7 - 9

(Αντίστοιχα αποτελέσματα θα έχουμε αν υποθέσου­

με ότι η διχοτόμος είναι ΑΔ : χ + y + 2 = Ο και το

ύψος ΓΖ: χ - 3y +2 =0) ΕΥΚΛΕΙΔΗΣ Β' 70 τ.2/46

Page 49: Ευκλειδης Β 70

Μ81ΙΙJΙ8ΊΙ8. ,.. q• r ��111 �- Αιι••Ι•ιι

Στατιστική Αποστόλης Κακαβάς

Άσκηση J 'l Οι μαθητές μιας τάξης ξόδεψαν σε μια μέρα κατά μέσο όρο 2 ευρώ στο κυλικείο του σχολείου. Εάν

ν

ο συντελεστής μεταβολής είναι 30% και Σ t � = 87, 2 να βρείτε τον αριθμό των μαθητών της τάξης. i = l

Λί>ση ν 2 { ( ν )2 } ( ν )2 S 8 1 ν � ti Σ ti � ti

Είναι CV = = <::::> 0,3 = - <::::> 8 = 0,6 ευρώ. 82 = - · Σ t; - '=1 <::::> 82 = � - '=1 2 <::::> χ 2 ν i=t ν ν ν

Σ ti � t i � ti 87 2 82=� - .!.::!._ <::::> 82=� - χ2 <::::> 0,36=-'- - 4<::::>0,36ν=87, 2--4ν<::::>4,36ν=87,2<::::>ν=20 μαθητές.

ν 2 [ ν ]2 ν

2

ν ν ν ν

Άσκηση 2 '1 Στο παρακάτω ιστόγραμμα αθροι­στικών σχετικών συχνοτήτων φαί-νεται ο χρόνος σε min που χρειά­στηκαν 40 μαθητές της Γ ' Λυκείου για να λύσουν μια άσκηση. i) Να γίνει εκτίμηση της διαμέσου του δείγματος. ii) Να βρεθεί το ποσοστό των μαθη­τών που χρειάστηκαν τουλάχιστον 7 min για να λύσουν την άσκηση. iii) Να βρεθεί πόσο χρόνο το πολύ χρειάστηκε το 10% των πιο γρήγο­ρων μαθητών για να λύσουν την άσκηση, καθώς και πόσοι είναι οι μαθητές αυτοί. Λί>ση

090 κ ---·----F!S _ _ _ _ _ _ _ _ _ _ _ _ _ _ _ _ _ _ _ _ _ _ _ _ _ _ _ Η _ �;;<z,_..... ,

ο � / '

Β Γ ... /' ,---____,/ / 0,60 0,50 - - - - - - - - - - - - - - - · - - - - - ;y/

Δ / ,Ε

χ δ 4

.-r-·..-· ο

7 χ

10 min

i) Από το σημείο του κατακόρυφου άξονα με τεταγμένη 0,50 φέρω παράλληλη στον οριζόντιο άξονα που τέμνει το πολύγωνο αθροιστικών σχετικών συχνοτήτων στο σημείο Ε και από το σημείο Ε φέρω παράλ­ληλη στον κατακόρυφο άξονα που τέμνει τον οριζόντιο στο σημείο με τετμημένη δ (διάμεσος) Στα όμοια τρίγωνα ΑΔΕ και ΑΒΓ έχουμε:

ΔΕ =

ΑΔ <::::> δ - 2 =

0,5 - 0,2 <::::> δ - 2 =

0,3 <::::> δ - 2 = � <::::> 4(δ _ 2) = 6 <::::> δ = 3 ,5 min

ΒΓ ΑΒ 4 - 2 0,6 - 0,2 2 0,4 2 4 ii) Από το σημείο του οριζόντιου άξονα με τετμημένη 7 φέρω παράλληλη στον κατακόρυφο άξονα που τέμνει το πολύγωνο αθροιστικών σχετικών συχνοτήτων στο σημείο Ζ και από το σημείο Ζ φέρω παράλ­ληλη στον οριζόντιο άξονα που τέμνει τον κατακόρυφο στο σημείο με τεταγμένη Fκ.

ΕΥΚΛΕΙΔΗΣ Β' 70 τ.2/47

Page 50: Ευκλειδης Β 70

Μαθηματικά για την Γ ' Λυκείου

Στα όμοια τρίγωνα ΘΗΖ και ΘΙΚ έχουμε: ΗΖ ΗΘ 7 - 6 F - Ο 75 1 F - Ο 75

- = - <::::> -- = Κ ' <::::> - = Κ ' <::::> 2(Fκ - 0,75) = 0, 1 5 <=>Fκ = 0,825 <::::> Fκ = 82,5 % ΙΚ ΘΙ 8 - 6 0,9 - 0,75 2 0,1 5

Άρα το ποσοστό των μαθητών που χρειάστηκαν τουλάχιστον 7min να λύσουν την άσκηση είναι: 1 00% -82,5% = 1 7,5% ii i) Από το σημείο του κατακόρυφου άξονα με τεταγμένη 0, 1 φέρω παράλληλη στον οριζόντιο άξονα, που τέμνει το πολύγωνο των αθροιστικών συχνοτήτων στο σημείο Λ και από το σημείο Λ φέρω παράλ­ληλη στον κατακόρυφο άξονα που τέμνει τον οριζόντιο άξονα στο σημείο με τετμημένη χ

ΜΛ ΟΜ χ 0,1 χ 1 . Στα όμοια τρίγωνα ΟΜΛ και ΟΝΑ έχουμε: -- = -- <=> - = - <=> - = - <=> χ = 1 mιn ΝΑ ΟΝ 2 0,2 2 2 ν . ν .

Άρα έκαναν το πολύ 1 min fi = _ι <=> Ο, 1 = _ι <=> νί = 4 μαθητές. ν 40

Άσκηση 3 '1 Σ' ένα δείγμα 200 στοιχείων, αφαιρούμε σε κάποια απ' αυτά τα στοιχεία τον αριθμό ιο και στα υ­πόλοιπα στοιχεία προσθέτουμε τον αριθμό 6. Η μέση τιμή του νέου δείγματος είναι μεγαλύτερη κα­τά 2 μονάδες του παλαιού. Να βρείτε σε πόσα στοιχεία του δείγματος προσθέσαμε τον αριθμό 6 και σε πόσα στοιχεία αφαιρέσαμε τον αριθμό ι Ο. ,\ �Jση

200 Σ t . Η αρχική μέση τιμή είναι χ = i= Ι ι

200 Έστω χ τα στοιχεία στα οποία αφαιρέσαμε τον αριθμό 1 Ο, τότε τα στοιχεία που προσθέσαμε τον αριθμό 6

200 Σ t . - 1 0χ + 6(2ΟΟ - χ) είναι 200 - χ. Η νέα μέση τιμή είναι χ' = .1..:i=�' _ι ______ _ 200

WO WO Σ t - - 1 0χ + 6(200 - χ) Σ t . 2οο

Ισχύει: χ' = χ +2 <=> i = Ι ι = i = Ι ι +2 <=>Σ t i 200 200 i = l

200

- ι οχ + 6(2οο - χ) = Σ t i + 400 <=> i =l

-l Ox + 6(200 - χ) = 400 <=> χ = 50 Άρα σε 50 στοιχεία αφαιρούμε τον αριθμό 1 Ο και στα υπόλοιπα 1 50 προσθέτουμε τον αριθμό 6 . 7Ι. σ κη ση 4 1 1 Αν οι τιμές χ1 , χ2, • • • ,χν έχουν μέση τιμή χ , τυπική απόκλιση Sx και συντελεστή μεταβολής CVx και οι μέσες τιμές αυτές μειωθούν κατά ι 0% και μετά αυξηθούν κατά μια ποσότητα β, τότε να βρε­θούν: α) Η νέα μέση τιμή y η νέα τυπική απόκλιση Sy ως συνάρτηση των χ , sx και β. β) Μετά απ' αυτές τις μεταβολές ο CVy μειώνεται κατά 20% σε σχέση με τον CVx και αν δίνεται ότι χ = ι 000, να βρεθεί η ποσότητα β. ΛiJση α) Οι νέες τιμές Yi , όπου ί= 1 ,2, . . . ,ν είναι: Yi = 0,9xi + β οπότε y= 0,9 χ + β και Sy = 0,9 · sx

s Y sx Ο, 9 · s x 0 8 s x 0,9 0 8 1 β) CV = Ο 8 ·CV <=> - = Ο 8 . - <::::> = . - <=> = . - <=> Υ ' χ Υ ' χ ο, 9χ + β ' χ 0,9χ + β '

χ

0 9 1 ' = 0,8 ο -- <=> 900 = 0,8(900 + β) <=> 900 = 720 + 0,8 β <=> β = 225 .

0,9 ο 1 000 + β 1 000

Λσκ:η ση 5'� α) Να βρεθεί η τυπική απόκλιση 5 διαδοχικών ακεραίων. β) Να βρεθούν οι 5 μικρότεροι διαδοχικοί ακέραιοι ώστε το δείγμα τους να είναι ομοιογενές. Λύση α) Έστω κ - 2, κ - 1 , κ, κ + 1 , κ + 2, κ ε Ζ οι 5 διαδοχικοί ακέραιοι, τότε

ΕΥΚΛΕΙΔΗΣ Β' 70 τ.2/48

Page 51: Ευκλειδης Β 70

Μαθηματικά για την Γ Λυκείου

κ - 2 + κ - 1 + κ + κ + 1 + κ + 2 5κ χ = = -= κ 5 5 2 (κ - 2 -χ/ + (κ - 1 - χ)2 + (κ - χ)2 + (κ + 1 - χ)2 + (κ + 2 - χ)2 s = � 5 2 (κ - 2 - κ)2 + (κ - 1 - κ)2 + (κ - κ)2 + (κ + 1 - κ)2 + (κ + 2 - κ)2 s = � 5 s2 = 22 + 12 + 02 + 12 + 22 � s2 = 2 � s = J2 5 β) Για να είναι το δείγμα ομοιογενές πρέπει και αρκεί CV � 1 0% � � � 1 0% � J2 � 0, 1 � 0, 1κ � χ κ J2 � κ � 1 0 J2 , κ ε Ζ , άρα κ ε Α = { 1 5 , 1 6, 1 7 , 1 8 , . . . } άρα κmίη = 1 5 οπότε οι 5 μικρότεροι διαδοχικοί ακέραιοι ώστε το δείγμα να είναι ομοιογενές είναι 1 3 , 1 4, 1 5 , 1 6, 1 7 . Άσκηση 6η α) Να συμπληρωθεί ο παρακάτω πίνακας β) Να βρεθεί η s �

Χϊ νί Χ 1 - 1000

Υϊ = 250

νiyi 2 ν1 y 1 750 10 1000 30 1250 25 1500 25 1750 4 2000 3 2250 3

Σύνολα -Λύση α)

Χϊ νί Υϊ χ 1 - 1000

νiyi 2 250

ν1 y 1 750 10 - 1 - 1 0 1 0 1000 30 ο ο ο 1250 25 1 25 25 1500 25 2 50 1 00 1750 4 3 1 2 36 2000 3 4 1 2 48 2250 3 5 1 5 75

7 Σύνολα 1 00 - Σν . y . = 1 04

i = l 1 1 7 2 Σ ν . y. =294 . ι ι 1 = 1

Σ ν . y . 2 2 1 7 2 i = l 1 1 1 1 04 1 ! ( 7 )2 ) β) s = - Σ ν; Υ; - = -{294 --} = - (294 - 1 08, 1 6) = 1 ,8584

Υ ν i =Ι ν 1 00 1 00 1 00

χ i - 1 000 . ' 2 2 2 έχουμε y; = 250

1= 1 ,2, . . . , 7� 250 y; = χ; - 1 000 � χ; = 250y; + 1 000 αρα s x = 250 sY

s � = 62500· 1 ,8584 τότε s � = 1 1 6 . 1 50 Π αρατή ρηση : Αν οι τιμές χ; μιας μεταβλητής είναι αρκετά μεγάλες και μπορούμε να εκφράσουμε κά­ποιες άλλες τιμές y; ως συνάρτηση των τιμών χ; με μια συνάρτηση της μορφής y; = αχ; + β όπου οι τιμές

ΕΥΚΛΕΙΔΗΣ Β' 70 τ.2/49

Page 52: Ευκλειδης Β 70

Μαθηματικά για την Γ Λυκείου

Yi είναι αρκετά μικρές, τότε είναι ευκολότερο να βρούμε πρώτα τα μέτρα θέσης και διασποράς των τιμών ' ' ξ λ ' ' θ ' ' λ ' 1 β ' Yi κανοντας πρα εις με πο υ μικροτερους αρι μους και μετα υνοντας ως προς Xi = - y . - - , οπου

α 1 α i= 1 ,2 , . . . , κ α =F Ο μπορούμε να βρούμε τα μέτρα θέσης και διασποράς των τιμών xi κάνοντας μόνο μια φορά πράξεις με μεγάλους αριθμούς σε κάθε περίπτωση .

Άσκηση 7'1 Σε μια κανονική κατανομή το 99,35% των τιμών βρίσκονται στο διάστημα (13, 33) όπου τα άκρα του διαστήματος είναι συνάρτηση των χ και s και διαφέρουν κατά πολλαπλάσιο του s. α) Να βρείτε σε κάθε περίπτωση (δύο περιπτώσεις) τα χ και s. β) Να εξετάσετε σε κάθε περίπτωση αν είναι τα δείγματα ομοιογενή. ΛίJση Για την καμπύλη συχνοτήτων σε μια κανονική κατανομή ισχύει: Έχουμε ότι 34% +34% + 1 3 ,5% + 1 3 ,5% + 2,35% = 97,35% Άρα το διάστημα ( 1 3 , 33) είναι το ( x-3s, χ +2s) ή το ( x-2s, χ +3s) Πρέπει και αρκεί να ισχύει: {χ - 3s = 1 3} {x - 2s = 1 3}

χ + 2s = 33 (Σ ι ) ή

χ + 3s = 33 (Σz) { χ - 3s = 1 3 } {- 5s = -20} { s = 4 } _:ι;· -j.:. .χ -2::; Χ - s }" .ι +s .ι +2 s .ι +3s

(Σ ι) <::::> <::::> <::::> - x - 2s = -33 x - 3s = 1 3 χ = 25 :._ 68% ­

·- 95% ---{ x - 2s = 1 3 } {- 5s = -20} { s = 4 } ---- 99 7 '):;. ___ __.,

(Σz) <::::> <::::> <::::> - χ - 3s = -33 χ - 2s = 1 3 χ = 2 1

β) Αν χ = 2 5 και s = 4 , τότε CVι = ; = 2� = Ο, 1 6 = 1 6% > 1 0% συνεπώς δεν είναι ομοιογενές το

πρώτο δείγμα. Αν χ = 2 1 και s = 4, τότε CV2 = ; = ;1 = 0, 1 904 = 1 9,04% > 1 0% συνεπώς δεν είναι

ομοιογενές ούτε το δεύτερο δείγμα.

Άσκηση 8'1 Σε ένα κυκλικό διάγραμμα παρουσιάζονται οι προτιμήσεις των μαθητών της Α ' Λυκείου για ποια κατεύθυνση ενδιαφέρονται να ακολουθήσουν. Η επίκεντρη γωνία που αντιστοιχεί στην τεχνολογι­κή κατεύθυνση είναι 5χ +2y μοίρες, για την θετική είναι 2χ + 2y μοίρες και για την θεωρητική x+4y=108°. Να υπολογίσετε το ποσοστό των μαθητών σε κάθε κατεύθυνση.

Λύση Έχουμε αι = 5χ + 2y ( 1 ) , α2 = 2χ + 2y (2) και α3 = χ + 4y = 1 08° (3)

( 3 ) ( 1 ),( 2 )

Ισχύει αι + αz + α3 = 360° <:::> αι + α 2 = 252° <:::> 7χ +4y = 252° ' { χ + 4y = 1 08 } {- χ - 4y = - 108} { 6χ = 1 44 } {χ = 24} Άρα

7x + 4y = 252 <::::>

7x + 4y = 252 <::::> x + 4y = 1 08 <::::>

y = 2 1 Άρα αι = 1 62° , α2 = 90° και α3 = 1 08° α ι = 360·fι <::::> fι = �:� = 0,45 άρα 45 % των μαθητών της Α ' λυκείου προτιμάει τεχνολογική κατεύ-

θυνση α2 = 360 f2 <::::> f2 = 90 = 0,25 άρα 25% προτιμάει θετική κατεύθυνση και 360

α3 = 360 ·f3 <::::> f3 =

1 08 = 0,3 άρα 30 % προτιμάει θεωρητική κατεύθυνση 360

ΕΥΚΛΕΙΔΗΣ Β' 70 τ.2/50

Page 53: Ευκλειδης Β 70

Μαθηματικά για την Γ Λυκείου

Γενικές Ασκήσεις Αργυράκης Δημήτρης - Μπρίνος Παναγιώτης

I . Δίνεται συνάρτηση f : R � R. Α ν η f είναι περιττή, συνεχής στο Χ0 = -1 fi(x) - 2 - ημ(χ + 1) και lim = 1 , τότε:

χ� -ι χ + 1 α) Να βρεθεί η τιμή της f στο Χ0 = - 1. β) Να αποδειχθεί ότι η f είναι συνεχής στο χ1 = 1 . γ) Να βρεθεί το όριο lim f(x) - 1

. χ� ι + χ - 1

ΛίJση : f(χ) -2-ημ(χ+ 1)

α) Για χ =ι:. -1 , θεωρούμε τη συνάρτηση g(x) , οπότε f(χ) =(χ + 1) g(χ) +2+ημ(χ+1) ( 1 ) χ + 1 Από υπόθεση είναι 1im g(x) = 1 , άρα 1im f(x) = lim [( χ + 1 ) g ( x ) + 2 + ημ(χ + 1 )] = 2. Επειδή η f είναι

χ � - 1 χ�- 1 χ �- 1

συνεχής στο Χ0 = - 1 , θα είναι f ( - 1 ) = lim f(x) , επομένως f ( -1 ) = 2 . χ �-1

f περιπή β) Αρκεί να δείξουμε ότι f ( 1 ) = 1im f(x) . Πράγματι έχουμε f ( 1 ) = f [- (-1 )] = - f (- 1 ) = -2 . Και αν

x �l

θέσουμε x=-t τότε f(x)=f(-t)=-f(t) οπότε, limf ( χ ) = - lim f( t ) = -2 . Επομένως f ( 1 ) = lim f(x) , οπότε η χ�1 t---+- 1 x �l

f είναι συνεχής στο χ 1 = 1 . γ) Επειδή lim(f(x) - 1 ) = -3 και lim -1- = +οο έχουμε lim f(x) - 1 = - οο.

x �l χ �ι+ χ - 1 χ �ι+ χ - 1 { αχ2 + βχ + 1 χ :;t: 1 2 . Να βρεθούν τα α , β , γ ε R , ώστε η συνάρτηση f, με f(x) = lx - 1 1 ' , να είναι

γ , χ = 1 συνεχής στο Χ0 = 1 .

Λύση : Αν η f είναι συνεχής στο Χ0 = 1 , τότε limf(x) = f( 1 ) = γ ( 1 ) .

x�l

αχ2 + βχ + 1 Για χ =ι:. 1 έχουμε: f(x) = I I � Ιχ - 1 1 f(χ) = αχ2 + βχ + 1 χ - 1 � lim( l x - 1 l f(x)) = lim(αx2 + βχ + 1 ) � Ο · γ = α + β + 1 � β = - α - 1

x �l x �l

αχ2 - (α + l )χ + 1 Αντικαθιστώντας την τιμή του β έχουμε: f(x) =

I I =

χ - 1 = αχ

2 - αχ - χ + l = αχ (χ - 1 ) - (χ - 1) = (χ - l ) (αχ - 1 ) Αλλά l x - 1 1 l x - 1 1 l x - 1 1

·

(χ - l ) (αχ - 1) (χ - 1) (αχ - 1) lim f(x) = lim = α - 1 και lim f(x) = lim = -α + 1 , οπότε χ �ι· χ �ι· χ - 1 χ �ι- χ �ι- -χ + 1 ( l )� α - 1 = -α + 1 = γ � α = 1 , γ = Ο . Άρα β=-α- 1 =-2 { l x - 1 1 , χ =ι:. 1 Επαλήθευση : Για α = 1 , β = -2 και γ = Ο έχουμε f(x) =

0 ' χ = 1 Είναι limf(x) = lim lx - 1 I = O = f( l ) , οπότε η f είναι συνεχής στο Χ0 = 1 όταν και μόνο

x�l x�l

α = 1 , β = -2 και γ = Ο . ΕΥΚΛΕΙΔΗΣ Β' 70 τ.2/51

Page 54: Ευκλειδης Β 70

Μαθηματικά για την Γ Λυκείου

3 . Δίνεται συνάρτηση f : R � R με 2 f(x) = χ + ημ(f(χ) - χ) , για κάθε χ Ε R. α) Να αποδειχθεί ότι I f(x) I � 2 1 χ I , για κάθε χ Ε R. β) Να αποδειχθεί ότι η f είναι συνεχής στο Ο.

Λύση : α) Για κάθε χ Ε R είναι: / 2f(x)/ = / χ + ημ(f(χ) - χ)/ � / χ / + /ημ(f(χ) - χ) / � / χ / + / f(x) - χ / � /χ / + / f(x) / + / χ / Άρα 2 /f(x) / � 2 / χ / + / f(x) / => / f(x) / � 2 / χ / , για κάθε χ ε R. ( 1 ) β ) Για χ = Ο από τη σχέση ( 1 ) έχουμε /f(O) / � Ο, οπότε f(O) = Ο (2) . Για κάθε xeR είναι / f(x) / � 2 / χ / , οπότε : - 2 / χ / � f(x) � 2 / χ / . Είναι lim ( - 2 / χ / ) = 0= lim(2 / x / ) , οπότε από κριτήριο παρεμβολής έχουμε lim f(x) = O (3 ) . Χ-->0 Χ-->0 Χ-->0 Από (2) και (3) έχουμε lim f(x) = f(O) . Άρα η f είναι συνεχής στο Ο. χ-->0

4. 'Εστω συνάρτηση f : R � R με f(xy) = f(x) + f(y) για κάθε χ , y Ε R και Iim f(x) = 2010 . χ--> 1 χ - 1

α) Να αποδειχθεί ότι η f είναι συνεχής στο Χ0 = 1 . β) Να βρεθεί το lim f(x) - f(3) . Χ-->3 Χ - 3 γ) Να αποδειχθεί ότι η f είναι συνεχής στο Χ0 = 3 . Λύση :

α) Για x :;t: 1 , θεωρούμε τη συνάρτηση g(x) = f(x) , οπότε f( x ) = (x- 1 )g( x ) ( 1 ) χ - 1 Από υπόθεση είναι limg(x) = 20 1 0, επομένως lim f ( x ) = lim[( x- 1)g(x )] = O (2). Χ-->1 Χ-->1 Χ-->1 Για x = y= 1 από τη δοθείσα σχέση έχουμε f(l) = f(l) + f(1), οπότε f(l) = O (3 ) . Από (2) και (3 ) έχουμε lim f(x) = f( 1 ) . Άρα η f είναι συνεχής στο 1 . χ-->1 β) Αν θέσουμε χ = 3t , τότε f(x) - f(3)

_ f(3t) - f(3) _ f(3) + f(t) - f(3) = f(t)

χ - 3 -

3t - 3 - 3 ( t - 1 ) 3 ( t - 1 ) '

lim f(x) - f(3) = _!_ lim f ( t ) = _!_ · 20 1 Ο = 670 . χ-->3 χ - 3 3 t-->1 t - 1 3

οπότε

γ) Αρκεί να δείξουμε ότι limf(x) = f(3) 'Εστω g ( x ) = f ( x ) - f (3) . Τότε f ( x ) = ( x - 3 ) g ( x ) + f (3 ) χ � χ - 3

οπότε limf( χ ) = Ο · 670 + f (3) = f (3 ) . χ�3

5. Δίνεται συνάρτηση f : R � R με f 2 (χ) � 2xf(x) + η μ 2χ, για κάθε χ Ε R .

α) Να αποδειχθεί ότι η f είναι συνεχής στο Χ0 = Ο . β) Να αποδειχθεί ότι lim f(�) = Ο .

x�+oc Χ

Λύση : α) Έχουμε f2 (x) � 2xf(x) + ημ2χ => f2 (x) - 2xf(x) + χ2 � χ2 + ημ2χ =>

(f(x) - x)z � χ2 + ημzχ => /f(χ) - χ / � �χz +ημzχ => -�χz + ημ2χ � f(x) - χ � �χz + ημ2χ =>

=> χ - �χ2 + ημ2χ � f(χ) � χ + �χ2 + ημ2χ ( 1 ) για κάθε x e R.

Είναι lim (x - �x2 + ημ2x ) = O = lim (x + �x2 + ημ2x ) , επομένως από κριτήριο παρεμβολής έχουμε χ�Ο χ � Ο

limf(x) = Ο . Επίσης για χ = Ο από την ( Ι ) έχουμε Ο� f ( O) � O , άρα f ( O)=O . χ � ο

Επομένως lim f(x) = f ( O) , οπότε η f είναι συνεχής στο χ0 = 0 . χ � Ο

ΕΥΚΛΕΙΔΗΣ Β' 70 τ.2/52

Page 55: Ευκλειδης Β 70

Μαθηματικά για την Γ Λυκείου

οπότε από κριτήριο παρεμβολής έχουμε lim f(�) = Ο . (Είναι lim ημχ = Ο)

χ---++οο Χ χ�+οο Χ

6 . α) Να βρείτε το lim e1 x , για τις διάφορες τιμές του χ Ε R . t---+-00

, , , . e1 x • συνχ + ημχ

β) Να αποδειξετε οτι η συναρτηση f(x) = lιm 1 , χ Ε R είναι συνεχής. t --+ - oo χ + e χ

Λύση :

α) Είναι e1 x = ( e ' Υ . Διακρίνουμε περιπτώσεις:

• Αν e x > 1 , δηλαδή χ > Ο , τότε 1im (e x ) 1 = Ο t ---t-00

• Αν e ' = 1 , δηλαδή χ = Ο , τότε lim (e x ) 1 = 1 t--+-oo

• Αν e ' < 1, δηλαδή χ < Ο , τότε 1im (e x ) 1 = +οο t--+-00 { Ο αν χ > 0

Επομένως lim e1 ' = lim (e x ) 1 = 1 � αν χ = Ο t--+-'X t--+-::C + οο , αν χ < Ο

β) Κατ' αρχάς βρίσκουμε τον τύπο της συνάρτησης f. Διακρίνουμε περιπτώσεις :

ll·m e1 x · συνχ + ημχ __

Ο · συνχ + ημχ __ η μχ

• Α ν χ > Ο , τότε I-+ -χ Χ + et x Χ + ο Χ

ο ,

1. e1 x · συνχ + ημχ Ι · Ι + Ο __ 1 • Α ν χ = , τοτε ιm -------''---

�--+ -χ χ + e1 x Ο + Ι • Αν χ < Ο , τότε

1 t x ( + -t x ) συνχ + -· ημχ . e ι χ · συνχ + ημχ . e συνχ e · η μχ . eι χ συνχ+ Ο · ημχ Ιιm = Ιιm = lιm = = συνχ

t --+ -"' χ + eι χ t--+ -oo eι χ (χe-ι χ + Ι ) t--+ -oo χ -1-+ Ι χ · 0 + 1

et x

Άρα είναι f(x) =

η μχ χ

συνχ ,

αν χ > Ο

αν χ = Ο = { η�χ ' αν

αν χ < Ο συνχ , αν

χ > Ο

χ � Ο

Η συνάρτηση f είναι συνεχής στο (Ο , +οο ) ως πηλίκο συνεχών συναρτήσεων, στο (- οο , Ο ) ως βασική

συνάρτηση , και στο Χ0=0, αφού lim ημχ = lim συνχ = 1 = f(O) . Επομένως η f είναι συνεχής συνάρτηση .

χ--+0+ χ χ--+0-

7 . Έστω συνάρτηση f : R � R, η οποία για κάθε x,yER ικανοποιεί τη σχέση f(x)-f(y)� 2009lx-� (1).

ΕΥΚΛΕΙΔΗΣ Β' 70 τ.2/53

Page 56: Ευκλειδης Β 70

Μαθηματικά για την Γ Λυκείου

Να αποδείξετε ότι: α) l f(x) - f(y) l � 2009 l x - y l για κάθε x , y Ε R . β) Η f είναι συνεχής. γ) Η εξίσω­ση f(x) = c x , για κάθε c Ε R με c > 2009 έχει το πολύ μια πραγματική ρίζα. Λί>ση : α) Η σχέση ( 1 ) ισχύει για κάθε χ , y Ε R, οπότε θα ισχύει και η σχέση : f(y) - f(x) � 2009 l y - x l , άρα lf (χ) - f (y)l � 2009 l x - Yl β) Για χ , Χ0 Ε R, έχουμε: l f (χ) - f (x0 ) 1 � 2009 l x - Χ0 1 => -2009 l x - Χ0 1 � f (χ) - f (x0 ) � 2009 l x - Χ0 1 Είναι lim ( -2009 1 χ - xo l ) = lim ( 2009 1 χ - xo l ) = Ο , οπότε από το κριτήριο παρεμβολής έχουμε ότι και

Χ �Χ 0 Χ � Χ0

lim [f(x) - f (x0 )] = 0 => lim f(x) = f (x0 ) . Επομένως η f είναι συνεχής συνάρτηση . Χ �Χ ο Χ --+ Χο

Ρι "'Pz Όμως l f(ρ 1 ) - f(p2 ) 1 � 2009 l p1 - ρ2 1 , άρα c l ρ1 - ρ2 1 � 2009 l ρ 1 - ρ2 1 � c � 2009, το οποίο είναι άτοπο,

αφού από υπόθεση είναι c > 2009 . Άρα η εξίσωση f(x) = cx , για κάθε c Ε R με c > 2009 έχει το πολύ μια πραγματική ρίζα. 8. Δίνεται η συνάρτηση f(x) = ημχ - 3χ - 1 , χ Ε R .

α) Να αποδειχθεί ότι η f είναι γνησίως φθίνουσα στο [ ; , π] .

β) Να βρεθεί το r ([ � , π]) και να αποδειχθεί ότι η εξίσωση f(x) = - 2π έχει ακριβώς μια λύση στο

[ π ] , . f(x) + 3π + l - , π . γ) Ν α βρεθει το lιm . 2 Χ-+Π Χ - Π

Λύση :

α) Για χ 1 , χ2 Ε [i , π] με χ 1 < χ2 έχουμε -3χ 1 > -3χ2 � -3x 1 - l > -3χ2 - 1 ( 1 ) .

Επίσης η συνάρτηση η μχ είναι γνησίως φθίνουσα στο [ i , π] , επομένως ημχ 1 > ημχ2 (2). Από ( 1 ) και (2) προκύπτει με πρόσθεση κατά μέλη ημχ 1 - 3χ 1 - 1 > ημχ2 - 3χ 2 - 1, δηλαδή f(x 1 ) > f(x2 ) , άρα η f είναι γνησίως φθίνουσα στο [ i , π] . β) Η f είναι γνησίως φθίνουσα και συνεχής στο [� , π] . οπότε έχουμε r([� , π]=[f(π) , {�)}[ -3π-1 , -� J . Αλλά: -2 π Ε f([� , π]) . οπότε υπάρχει τουλάχιστον ένα Χ0 Ε [i , π] με f(χ0 ) = -2π και λόγω της μονοτονίας της f το Χ0 είναι μοναδικό . ) 'Ε f(χ) + 3π + 1 η μχ - 3χ - 1 + 3π + 1 ημχ - 3(χ - π)

γ χουμε = = =

χ - π χ - π χ - π = ( ημχ - 3) = (- ημ (π - χ ) - 3J π-�=1 (- η μ t - 3) . Άρα lim f(x) + 3π + 1 = lim (- ημ t - 3)= -4 . χ - π π - χ t χ-+π χ - π t-+0 t

9. Δίνεται συνεχής συνάρτηση f : R � R με f(O) =Ο και lim f(x) = +οο . Να δειχθεί ότι υπάρχει του-•-++«>

ΕΥΚΛΕΙΔΗΣ Β' 70 τ.2/54

Page 57: Ευκλειδης Β 70

Μαθηματικά για την Γ Λυκείου

- ι λάχιστον ένα Χ0 > Ο τέτοιο, ώστε να ισχύει f(x0 ) = 2 '· + Χ0 - Χ0συν- .

Λύση :

Θέτουμε g(x) = f(x) - 2-χ - χ + χ συν_!_ , με χ > Ο . χ

χ ο

Για κάθε χ > ο ισχύει l χ συν� l = Ι χ ι lσυν � I � l x 1 · 1 = χ ' οπότε -χ � χ συν � � χ ( 1 ) . Είναι

lirη (-χ) = Ο = lirη χ , οπότε από κριτήριο παρεμβολής έχουμε lirη_ (χσυν_!_) =0. χ�ο χ�ο χ�ο χ Επίσης η f είναι συνεχής στο Ο, οπότε lim f(x) = f ( Ο) = Ο .

Χ ---+0+

Άρα lim g(x) = lim (f(x) - 2-χ - χ + χ συν_!_) = -1 < Ο , οπότε υπάρχει διάστημα της μορφής (Ο, α) τέ-χ -+ο+ χ -+ο+ χ

τοιο, ώστε g(x) < O για κάθε χ ε (Ο, α) , και επομένως g(κ) < Ο για κάποιο κ ε (Ο, α) . Η συνάρτηση g

γράφεται g(x) = f(x) - Tx + χ(συν_!_ - ι) . Αν θέσουμε u = _!_ , τότε: lim [χ(συν_!_ - 1)] = lim σuvu -1 Ο . Χ Χ x-++ao Χ u-+0+ U

Επίσης lim 2-χ = Ο , οπότε limg(x) = lim[f(x) -Tx + x(σuv.!. -1)] =-too , άρα υπάρχει διάστημα της μορ-χ �+οc Χ--++<χ::ι X-+t«J χ

φής (β, + οο ) τέτοιο, ώστε g(x) > O για κάθε χ ε (β, + οο ) , επομένως g(λ) > Ο για κάποιο λε (β, + οο ) .

Έχουμε: • Η g είναι συνεχής στο [κ , λ] ως αποτέλεσμα πράξεων μεταξύ συνεχών συναρτήσεων και • g(κ) g(λ) < Ο . Σύμφωνα με το θεώρημα Bolzano, θα υπάρχει ένα τουλάχιστον Χ

0 ε (κ , λ) ς (Ο , + οο) , τέτοιο ώστε

g(xo )= O <=> f(x 0 )= Γχ· + Χ0- Χ

0 συν -1 . χ ο

1 Ο. Έστω f , g δύο συναρτήσεις συνεχείς στο [ Ο , ι ] . Α ν f ο g = g ο f , η f είναι γνησίως φθίνουσα στο [ Ο , ι ] , Ο� f(χ) � ι και Ο� g(χ) � ι για κάθε χ ε [ Ο , ι ] , να αποδείξετε ότι υπάρχει ξ ε [Ο , ι] τέ­τοιο, ώστε f(ξ) =ξ και g(ξ) = ξ .

Λύση : Θεωρούμε συνάρτηση h(x) = f(x) - x , χ ε [ Ο , 1 ] και παρατηρούμε ότι: • Η h είναι συνεχής στο [Ο , 1] ως διαφορά συνεχών συναρτήσεων και • h(O) h(1) = f(O) - ( f(l) - 1 ) � Ο .

'-ν-' '-----ν----' �ο ,;ο Διακρίνουμε περιπτώσεις � Α ν h(O) h(l) < Ο , τότε σύμφωνα με το Θεώρημα Bolzano, θα υπάρχει ένα τουλάχιστον ξ ε (Ο , 1 ) τέ-

τοιο, ώστε h(ξ) = Ο <::::> f(ξ) =ξ. � Εξάλλου h(O) h(l) = O ::::> h(O) = O ή h(l) = O ::::> f(O) = O ή f(l) = l => ξ = Ο ή ξ = 1 . Άρα και στις δύο περιπτώσεις θα υπάρχει ένα τουλάχιστον ξ ε [ Ο , 1 ] τέτοιο, ώστε f(ξ)=ξ . Με τη μέθο­δο της απαγωγής σε άτοπο θα αποδείξουμε ότι για το ίδιο ξ ε [ Ο , 1] ισχύει και g(ξ) =ξ . Έστω ότι: g(ξ) >ξ ( 1 ) . Τότε

f ,!. fog�gof f (ξ)�ξ ( 1 )=>f (g(ξ) )<f(ξ) => ( fog ) (ξ) <f(ξ) => (gof ) (ξ) <f(ξ) => g ( f(ξ) )<f(ξ) => g (ξ )< ξ , πράγμα άτοπο. Έστω ότι: g(ξ) <ξ (2). Τότε

ΕΥΚΛΕΙΔΗΣ Β' 70 τ.2/55

Page 58: Ευκλειδης Β 70

Μαθηματικά για την Γ Λυκείου f .j. fog�gof Γ (ξ)�ξ

(2 )� f (g(ξ) ) >f(ξ) � (fog ) (ξ) >f(ξ) � (gof ) (ξ) >f(ξ) � g (f(ξ) ) >f(ξ) � g (ξ )> ξ , πράγμα άτοπο. Επομένως υπάρχει ξ Ε [ Ο , 1 ] τέτοιο, ώστε f(ξ) =ξ και g(ξ) =ξ .

1 1 . Έστω συνεχής συνάρτηση f : R � R με f ( -1) = 1 , η οποία για κάθε χ Ε R ικανοποιεί τη σχέ­ση f(f(x)) + χ6 f(x) = Ο (1). α) Να βρεθούν οι τιμές f (l ) και f (O) .

[f(l) + l] x4 + f(l ) x2 - 3χ + 2 β) Να υπολογιστεί το όριο lim

( ) 3 2 H-ao f Ο Χ + 4χ - x + l

Λί>ση : α) Για χ = -1 από την ( 1 ) έχουμε : f(f( - 1 )) + ( - 1 )6 f( - 1 ) = Ο � f( l ) + 1 = Ο � f( l ) = - 1 Είναι f ( 1 ) = - 1 < Ο < 1 = f ( - 1 ) και η f είναι συνεχής στο [ -1 , 1 ] , επομένως από θεώρημα ενδιαμέσων τι­μών θα υπάρχει τουλάχιστον ένα Χ 0 Ε ( -1 , 1) τέτοιο, ώστε f(x0 ) = Ο . Για χ = Χ0 από την ( 1 ) έχουμε:

f(f(X0 )) + Χ06 f(X0 ) = Ο � f(O) + Χ0 6 · Ο = Ο � f(O) = Ο . β) Για χ ε ( -οο , Ο) έχουμε

lim [f( 1 ) + 1 ] x4 + f(1 ) x2 - 3x + 2 = lim Ο · χ4 +(-1) χ2 - 3χ + 2

= lim - χ2 - 3χ + 2 = lim - χ

2 = - _!_ Χ -Η<> f (O) x3 + 4x2 - x + 1 Χ -ΗΟ Ο · χ3 + 4χ2 - χ + 1 x --+-ao 4x2 - x + l x --+-ao 4χ

2 4

1 2. Έστω συνεχής συνάρτηση f : R � R , με f (Ο) = 2 η οποία για κάθε χ Ε R ικανοποιεί τη σχέση

f(f(x)) + 4 f(x) = 6 - χ4 (1) . α) Να βρεθούν οι τιμές f (2 ) και f ( - 2) . β) Να βρεθούν οι τιμές r ( - .J2)

και r (.J2) . γ) Αν lim χ4 + 4f(x) - S

= - 4 να βρεθεί το lim f(f(x)) . χ---+ 1 χ - 1 χ--+1

Λύση :

α) Για χ = Ο από την ( 1 ) έχουμε: f ( f ( 0)) +4f (O) = 6 � f (2 ) + 4 · 2 = 6 � f (2 ) = -2 . Για χ = 2 από την

( 1 ) έχουμε: f ( f (2 ) )+4f (2 ) = 6 - 24 � f ( -2) + 4 · (-2) = 6 - 1 6 � f ( -2) = -2 .

β) Είναι f (-2) = -2 < 0 < 2 = f ( O) και η f είναι συνεχής στο [-2 , 0] , επομένως από θεώρημα ενδιαμέ­σων τιμών θα υπάρχει τουλάχιστον ένα Χ0 Ε (-2 , Ο) έτοιο, ώστε f(x0 ) = Ο . Για χ = Χ0 από την ( 1 ) έ­χουμε : f(f(X0 )) + 4f(x0 ) = 6 - χ: � f ( O) + 4 · Ο = 6 - χ: � 2 = 6 - χ: � χ: = 4 � �xo =fi ή X0 =-J2 �x0 =-J2 , αφού Χ0 Ε (-2 , 0 ) . Άρα f (-J2 )= 0 . Είναι f ( 2) = -2 < 0 < 2 = f (O)

και η f είναι συνεχής στο [Ο , 2] , επομένως από θεώρημα ενδιαμέσων τιμών θα υπάρχει τουλάχιστον ένα Χ ι Ε (Ο , 2) τέτοιο, ώστε f(χ ι ) = Ο . Για χ = Χ ι από την ( 1 ) έχουμε:

f(f(x ι )) + 4f(x ι ) = 6 - x: � f (0) + 4 · 0= 6 - x: � 2= 6 - χ: � χ: = 4 � Χι = .fi ή Χι = -.fi� xι = .Ji ,

αφού Χ ι Ε ( Ο , 2 ) . Άρα f ( ν'2)= ο . χ4 + 4f(x) - 5 ( x- 1)g(x) - x4 + 5

γ) Για χ * 1 , θεωρούμε τη συνάρτηση g(x) = , οπότε f( χ) = (2) χ - 1 4

Από υπόθεση είναι u = f (x)

1im f( f(x) ) = lim f( u ) = 1 . χ --+ 1 u--+1

lim g(x)= -4, χ --+ 1

οπότε ( χ - 1 ) g ( x ) - χ4 + 5 lim f ( χ ) = lim = 1 ,

χ --+1 χ ---+1 4 και

1 3 . Έστω συνεχής συνάρτηση f : [α , Ρ] � R. Υποθέτουμε ότι για κάθε χ Ε [α, β] υπάρχει

y Ε [α , β) , ώστε l f(y) l � ! ι r<x> l · Να αποδειχθεί ότι υπάρχει ξ Ε [α , p] , ώστε f (ξ) = ο . 2

ΕΥΚΛΕΙΔΗΣ Β' 70 τ.2/56

Page 59: Ευκλειδης Β 70

Μαθηματικά για την Γ ' Λυκείου

Λύση: Η f είναι συνεχής στο [α , β] , οπότε από θεώρημα μέγιστης και ελάχιστης τιμής υπάρχουν χ ι , χ2 ε [α , β] , τέτοια ώστε μ = f(χ ι ) :::; f(x) :::; f(x 2 ) = Μ, για κάθε χ ε [α , β ] . Έστω ότι δεν υπάρχει ξ ε [α , β] , ώστε f (ξ ) = Ο, δηλαδή f(x) * Ο για κάθε χ ε [α , β] , τότε επειδή η f είναι συνεχής στο [α , β] , θα διατηρεί σταθερό πρόσημο στο διάστημα αυτό. Δηλαδή f(x) > Ο, για κάθε χ ε [α , β] ή f(x) < Ο, για κάθε χ ε [α , β ] . Έστω τώρα ότι η f(x) > Ο, για κάθε χ ε [α , β ] . Από υπόθε-

ση για κάθε χ ε [α , β] υπάρχει y ε [α , β] , ώστε \ f(y) \ :::; _!_ \ f(x) \ , δηλαδή f(y) :::; _!_ f(x) ( 1 ) , αφού δε-2 2

χτήκαμε ότι f(x) > Ο για κάθε χ ε [α , β ] . Για χ = Χ ι , από την ( 1 ) προκύπτει ότι υπάρχει y ε [α , β] , ώ-

στε f(y) :::; _!_ f(χ ι ) και _!_ f(χι ) < f(χι ) , αφού f(χ ι ) > Ο , άρα f(y) < f(χ ι ) , άτοπο, αφού f(χ ι ) είναι η ελά-2 2

χιστη τιμή της f. Ομοίως, αν υποθέσουμε ότι f(x) < Ο, για κάθε χ ε [α , β ] . Από υπόθεση για κάθε

χ ε [α , β] υπάρχει y ε [α , β ] , ώστε \ f (y) \ :::; _!_ \ f(x) \ , ή - f(y) :::; - _!_ f(x), ή f(y) � _!_f(x) (2) , αφού δε-2 2 2

χτήκαμε f(x) < Ο, για κάθε χ ε [α , β] . Για χ = χ2 , από την (2) προκύπτει ότι υπάρχει y ε [ α , β] , ώστε

f(y) � _!_f(x0 ) και _!_ f(x0 ) > f(x2 ) , αφού f(x 2 ) < 0 , άρα f(y) > f(x2 ) , άτοπο, αφού f(x2) είναι η μέγιστη 2 - 2 -

τιμή της f. Καταλήξαμε σε άτοπο, γιατί υποθέσαμε ότι δεν υπάρχει ξ ε [α , β] , ώστε f (ξ ) = Ο. Άρα υ­πάρχει ξ ε [α , β] , ώστε f(ξ) = Ο. 1 4 . Θεωρούμε συνεχή συνάρτηση f : R ---:) R και τον αριθμό z = x + i f(x), χ ε R. Αν ισχύει

f ( i) f ( 32α ) < Ο , α>Ο και για κάθε χ ε (Ο , 2α) ισχύει Ι z - α Ι :::; α , Ι 2z - α Ι � α , l 2z - 3α Ι � α , τότε

να αποδειχτεί ότι f (α ) = Ο . ΛίJση : Από τη σχέση Ι z - α Ι :::; α προκύπτει ότι η εικόνα M(z) του μιγαδικού z ανήκει στο κυκλικό δίσκο κέ-

ντρου Α (α , Ο) και ακτίνας α. Από τη σχέση \ 2z - α\ � α ή ισοδύναμα Jz -�� � � προκύπτει ότι η εικό-

να M(z) του μιγαδικού z είναι εξωτερικό σημείο ή σημείο του κύκλου κέντρου Β(� , Ο) και ακτίνας � . Από τη σχέση \2z - 3α\ � α ή ισοδύναμα Jz - 3

2α ι � � προκύπτει ότι η εικόνα M(z) του μιγαδικού z είναι

Υ ξ ' ' ' ' ' λ ' r ( 3α

ο) ε ωτερικο σημειο η σημειο του κυκ ου κεντρου l , και

ακτίνας � . Η f είναι συνεχής στο [ � , 32α ] και f( �) . f( 3;) < Ο ,

---,.�f------+-"'--'---'--+--� χ επομένως από θεώρημα Bolzano υπάρχει τουλάχιστον ένα

χ,ε(� ,�) έτσι, ώστε f(x0 ) = 0 . Για χ = Χ0 , έχουμε

z = x0 + if(x0 ) = xo ε R. Από το παραπάνω σχήμα, προκύπτει ότι zεR, αν και μόνο αν z = Ο ή z = α ή z = 2α . Από τις τιμές αυτές

μόνο η z = α ανήκει στο ( � , 32α ) , άρα f (α) = Ο.

ΕΥΚΛΕΙΔΗΣ Β' 70 τ.2/57

Page 60: Ευκλειδης Β 70

Μαθηματικά για την Γ Λυκείου

Με αφορμή μια άσκηση του σχολικού Βιβλίου Γ.Σ. Τασσόπουλος, Καθηγητής Βαρβακείου Λυκείου

Στην Άσκηση 6 σελίδα 1 48 του Σχολικού Βιβλίου Θετικής και Τεχνολογικής Κατεύθυνσης, ζητείται να βρεθεί συνάρτηση f τέτοια ώστε να ισχύει (3 περιπτώσεις) : ί) (fog)(x)=x2+2x+2, αν g(x)=x+ 1 ii) (fog)(x)= .,fl;;J , αν g(x)= -χ2

iii) (gοf)(χ)=ΙσυνχΙ , αν g(x)= .J1 + χ 2 Οι συγγραφείς με την έκφραση <<να βρεθεί συ­

νάρτηση f>> εννοούν <<να βρεθεί έστω μια< Ι ) συνάρ­τηση f>> .

Έτσι στο μεν ερώτημα (iii) απαντούν (ορθώς) βρίσκοντας δυο μόνο συναρτήσεις και όχι όλες, ενώ στο ερώτημα (ίί) αποφαίνονται (εσφαλμένα) ότι: η συνάρτηση f (δηλαδή η μοναδική συνάρτη­ση f) είναι η f ( χ ) = � με χ�Ο, πράγμα το ο-ποίο όπως θα δούμε παρακάτω δεν είναι σωστό.

Στην περίπτωση μάλιστα που θα είχαμε ( fog ) ( χ ) = .J1 - χ2 και g(x)= -χ2 με την ίδια λογι­

κή θα έβρισκαν f ( χ ) = � με χ�Ο. Όμως είναι προφανές ότι η f δεν μπορεί τότε να

οριστεί στο (-οο,Ο] π.χ. για χ= -2, αλλά πρέπει να ι­σχύει και χ2::-1 . Κάπου λοιπόν ελλοχεύει ένα λάθος.

Πράγματι το κακό ξεκινάει από το γεγονός ότι ακόμη και στο ερώτημα (ί) που είναι απλούστερο, δε γίνεται καθόλου λόγος για το πεδίο ορισμού της f.

Ας ξεκινήσουμε λοιπόν απ' αυτό για να φανεί τι ακριβώς απουσιάζει από τη Λύση του Σχολικού Βιβλίου. i) Έχουμε Arog= JR και Ag= JR . Άρα για το σύνο­

λο A '= {xεAg/g(x) εAr} ξέρουμε ότι Α '= JR

Ισχυριζόμαστε ότι AF JR . Π ράγματι : Αν ήταν A# JR , τότε θεωρούμε τα yi ε JR με Yi �Ar ο­πότε Α '= {χε JR /χ+ 1:;tyi }= {xε JR /x:;tyi-1 }:;t: JR , πράγμα άτοπο, αφού A '= JR . Επομένως AF JR και f(g(x))=(x+1 )2+ 1=g2(x)+ 1 , για κάθε χ ε JR .

1 Κατά τη γνώμη μας κακώς χρησιμοποιείται μια τέτοια έκφραση, αφού θα μπορούσαμε να είμαστε σαφέστατοι λέγοντας <<να βρεθεί τουλάχιστον μια συνάρτηση 6> . Στη Γεωμετρία π.χ. όταν λέμε να βρεθεί ή να κατα­σκευαστεί τρίγωνο με κάποιες ιδιότητες δεν εννοούμε τουλάχιστον ένα τρίγωνο, αλλά όλα τα τρίγωνα που έ­χουν αυτές τις ιδιότητες.

Άρα f( ω )=ω2+ 1 , για κάθε ω ε JR . Θα μπορού­σαμε να θέσουμε χ+ 1 =ω, δηλαδή χ=ω-1 στον τύπο της fog αν ήταν πολυπλοκότερος.

ίί) Έχουμε προφανώς Ag= JR και Αισg= JR , οπότε για το σύνολο A '= {x εAg/g(x) εAr} ξέρουμε ότι A '= JR και f (g ( x ) ) = �1 - g ( x ) , για κάθε

χεΑg= JR με g(x) εAh όπου h: (-οο,Ο]- JR :

h ( χ ) = � , δηλαδή Ah=( -οο, Ο] • Αν λοιπόν θέλουμε η fνα είναι μονόκλαδη συ­

νάρτηση, τότε θα έχουμε Ar � Ah δηλαδή AF(-oo, α] ή AF(-oo, α) με α�1 . Έστω Αι=(-οο, α] . Αν α<Ο, τότε Α '= {χε JR /-χ2�α}= {χε JR /χ22::-α}= = ( -oo, -�Ju[ �, +οο) :;t: JR, πράγμα άτοπο

αφού A '= JR . Αν Ο�α� 1 , τότε Α '= {χε JR /-x2�α}= JR . Ανά­λογα εργαζόμαστε αν AF(-oo, α) με Ο<α�1 (2) . Υπάρχουν λοιπόν άπειρες μονόκλαδες συ­ναρτήσεις της μορφής f ( χ ) = � με AF(-oo, α] και Ο�α�1ή AF(-oo, α) και Ο<α�1 με ευρύτερο Αrτο ( -οο, 1 ] και όχι μόνο η f: ( -οο, Ο]- JR : f ( χ ) = � που βρίσκει το σχολικό βιβλίο .

• Αν θέλαμε πολύκλαδη συνάρτηση f τότε θα εί-

( ) {-Jι - χ , χ ε (-οο,α)

χαμε : f χ = με Ο � α � 1 . Φ (χ ) , χ > α

Η φ( χ) είναι τυχαία συνάρτηση μονοκλάδη ή πολύκλαδη π. χ. Φ ( χ ) = 2χ + 3, χ > α ή

Φ ( χ ) = {2χ + 3, χ ε ( α, 2 ) χ + 5, χ � 2

• Ας θεωρήσουμε τώρα και το δυσκολότερο πα­ράδειγμα που προαναφέραμε, όπου :

( fog ) ( x ) = .J1 - χ2 και g(x)= -χ2

Έχουμε τώρα Arog=[-1 , 1 ] και Ag= JR .

Άρα για το σύνολο A '= {xεAg/g(x) εAr} ξέ­ρουμε ότι Α '=[-1 , 1 ] . Εξάλλου

2 Αν α=ο, τότε αποκλείεται να ισχύει AF(-oo, α)=(-οο,Ο), διότι θα είχαμε: Α '= {χε JR /-x2<0}= 1R * , πράγμα άτο­πο αφού Α '= JR .

ΕΥΚΛΕΙΔΗΣ Β' 70 τ.2/58

Page 61: Ευκλειδης Β 70

Μαθηματικά για την Γ Λυκείου

f (g ( χ )) = �1 + g ( χ ) για κάθε χεΑg= IR με

g(x) εAh όπου h : [-1 ,+οο)-+ IR : h ( χ) = .J1 + χ , δηλαδή Ah=[-1 ,+oo) .

• Αν λοιπόν θέλουμε η fνα είναι μονόκλαδη συ­νάντηση τότε θα έχουμε Ar ς Ah , δηλαδή AF[-1 , α] ή AF[-1 , α) (3) . Έστω AF[-1 , α] . Αν α<Ο, τότε Α'={χε IR /-1::=:-χ2:::;α}= = {χ ε IR /-α:::;χ2:::; 1 } = {χε IR /χ2:::; 1 και χ22:-α} = = {χ ε ΙR /χ ε [-1 , 1 ] και χε(��}-{ k-too)} :ι!:[-1 , 1 ] . Καταλήγουμε λοιπόν σε άτοπο. (Διακρίνουμε τις περιπτώσεις: α=-1 και -1 <α<Ο) Αν α2:0, τότε Α'= { χε IR /-1:::;-χ2:::;α}= = {χ ε JR /-α:::;x2:::; 1 }= {xεR/x2:::; 1 }=[-1 , 1 ] . Ανά­λογα εργαζόμαστε αν ΑF[-1 ,α) με α>Ο (4). Υπάρχουν λοιπόν άπειρες μονόκλαδες συναρ­τήσεις της μορφής f (χ) = Jl+;. με AF[-1 , α] και Ο:::;α:::; 1ή AF[-1 , α) και Ο<α:::; 1 με ευρύτερο Αrτο [-1 ,+οο).

• Αν θέλαμε πολύκλαδη συνάρτηση f τότε θα εί-

χαμε: f ( x ) = με α ;::: Ο {JΙ+;., χ ε [ -1 , α]

Φ ( χ ) , χ > α Όπου φ(χ) τυχαία συνάρτηση μονόκλαδη ή πολύκλαδη.

iii) Το ερώτημα (iii) βέβαια είναι πολυσυζητημέ­νο, επειδή ακριβώς μπορεί να οδηγήσει σε ένα ανεπαίσθητο λογικό λάθος κατά το οποίο θεω­ρείται (κακώς) η πρόταση : (Για κάθε χ ε Ω, ισχύει p(x) ή q(x)) ισοδύναμη

με την πρόταση : (Για κάθε χ ε Ω, ισχύει p(x) ή για κάθε χ ε Ω, ισχύει q(x))

Για παράδειγμα αν Ω= { 1 ,2 } τότε η πρόταση : (Για κάθε χ ε Ω, ισχύει χ-1 ή χ-2=0) είναι αληθής, ενώ η πρόταση (Για κάθε χ ε Ω, ισχύει χ-1 =0 ή για κάθε χ ε Ω, ισχύει χ-2=0) είναι προφανώς ψευδής.

Ακόμη σαφέστερα: Όταν λέμε ότι κάθε κάτοι­κος μιας πολυκατοικίας κάθεται στον πρώτο ή στο δεύτερο όροφο, προφανώς δεν εννοούμε ότι κάθε κάτοικος κάθεται στον πρώτο ή κάθε κάτοικος κά­θεται στο δεύτερο όροφο, δηλαδή ότι ο ένας όρο-

3 Αποκλείεται να ισχύει: ( - 1 ) � Ar διότι τότε 1 ,- 1 �Α ', πράγμα άτοπο, αφού Α'=[-1 , 1 ]

4 Αν α=Ο, τότε αποκλείεται να ισχύει ΑF[-1 ,α)=[-1 ,0),

διότι θα είχαμε: A'= {x ε R/-1::;-x2<0}=[-1 ,0)U(0, 1 ] , πράγμα άτοπο αφού Α'=[-1 , 1 ] .

φος είναι ακατοίκητος. Προσέχοντας όσα προανα­φέραμε θα έχουμε για το ερώτημα (i i i) :

(gof) ( χ ) = iσυνχl � �1 - f2 ( χ ) = iσυνχl �

� 1 - f2 (χ ) = συν2χ � f2 (χ ) = 1 - συν2χ �

� f2 ( χ ) = η μ2χ δηλαδή: Για κάθε χε IR , ισχύει :Qχ)=ημχ ή :Qχ)=-ημχ. Έτσι όμως δεν βρήκαμε την f, αφού π.χ. δεν γνω-

ρίζουμε την τιμή f( �) . Πιθανόν να είναι

f (�) =� ή f (�) = -� .

Δυο βέβαια συναρτήσεις όπως λέέι το Σχολικό βιβλίο, είναι η f1(χ)=ημχ με Α� = IR και I

η f2(χ)=-ημχ, Ar = IR . 2

Υπάρχουν όμως άπειρες συναρτήσεις που ικα­νοποιούν το ερώτημα (iii), με μορφή : {η μχ, χ ε Α

f ( x ) = -η μχ, χ ε IR - Α, όπου Α ς IR

Ειδικά αν Α= IR , τότε f-=f1, ενώ αν Α=0 τότε f-=f2. Με το άρθρο αυτό θέλαμε να γίνει κατανοητό

ότι μερικά προβλήματα, που φαίνονται απλά εκ πρώτης όψεως, κρύβουν πολλές φορές δυσκολίες ή παγίδες.

Τα μαθηματικά σήμερα ΑΝΑΛΥΙΗ

ΓΡΑΜ. ΑΛΓΕΒΡΑ ΑΝΑΛ. ΓΕΩΜΗΡΙΑ

μ ε ru·�·.:χ:αη Τ')<; Γφμ. ·Ε κt.<Χ)ης Ath�� � Grie:;.el • Pσste�

σρ. ίΙ'ΚΡ. Υπ. Παιδε ίας 6ιiC�J.' Γ 7 'C.0

3 τόμοι έγχρωμοι πολυτελείς Για το Γυμνάσιο - Λύκειο - Φοιτητές - Καθηγητές Με εγ�,·ιj η σ η το ��ύp::.ς των �:σ;•uφοiω·.' ε.7τισΊηuόιιι:υν της Γε ρμαι·:a ς

Μι rον πρωr•)Η.: r rο �<.:ι (ωντα •/6 r;>:Jno ��δ<:ϊJ !Ι\Ολiος 1ων tH:) μοv r t: ρ •,· ι � ι ·ι j JOOημo r ι κ(iJv 0Ηι..Ιρι ι;χ�· . i.ξΩοψ�Jλ)(f.ι i'/0 �: ι..ιρ:ιJ ! r Ο ι'ι<6 επίπt:t-ο διf:<ωκαλiοζ κ ω γνι� 1σης

αξία: 90 € - προσφορά μέ)(ρι 30 Μαρτίου 45 € r::1f1 ο CJljντι;ο-..φος ται μο&ψή Σrοuρνόρη 36. Αθrjνα 1 04 33 � εκδόσεις Κτίστη Τηλ 2 1 0 52 23 423

ΕΥΚΛΕΙΔΗΣ Β ' 70 τ.2/59

Page 62: Ευκλειδης Β 70

π I

Η στήλη αυτή έχει ως στόχο την ανάπτυξη μαθηματικού διαλόγου. Φιλοδοξούμε να συμμετάσχουν όλοι, όσοι έχουν ένα γενικότερο ενδιαφέρον, για τα Μαθηματικά, με ιδέες σκέψεις προτάσεις και επίκαιρα θέματα.

Επιμέλεια: Γιάννης Στρατής - Βαγγέλης Ευσταθίου

Η ευθεία του Eiίler, μια συναρτησιακ-ή σχέση και το θεώρημα του Καpαθεοδωpή

Σωτήρης Ε. Λουρίδας

. . . λίγη σκέψη πριν την λύση

Απευθυνόμενοι σε εκείνους που τους αρέσουν τα Μαθηματικά προβλήματα και που

ενδιαφέρονται κύρια για την σκέψη που οδηγεί στην λύση παραθέτουμε τρία προβλήματα με τις

λύσεις τους. Το πρώτο είναι ένα γεωμετρικό θέμα που είχα κατασκευάσει για να συμπεριληφθεί

στην Shortlίsted Problems of LM. O 2008. Το δεύτερο είναι θέμα από τον διαγωνισμό Β.Μ. Ο 200 7 που έγινε στην Ρόδο και το τρίτο είναι το Θεώρημα του Καραθεοδωρή. Θα

προσπαθήσουμε να δώσουμε έμφαση και στη σκέψη που οδηγεί στην λύση των προβλημάτων

αυτών. Έχει μεγάλη σημασία η λύση ενός μαθηματικού προβλήματος να μην δίνει την αίσθηση του

«θεόπεμπτου». Σε πολλές μάλιστα περιπτώσεις η απόρριψη λαθεμένων σκέψεων που πιθανόν

γίνονται πριν από τη λύση ανοίγει την οδό που οδηγεί στην σωστή σκέψη και τελικά στην επίλυση

του προβλήματος.

Π ρόβλημα 1 ° Λ

Δίνεται κύκλος (Κ. R). Θεωρούμε ότι υπάρχει τρίγωνο ΑΒΓ με Α = 60° εγγεγραμμένο στον κύκλο

αυτό που να μην είναι αμβλυγώνιο και με την ιδιότητα GΓ = BG + KG· .J3 όπου G το κέντρο βάρος του. Να υπολογιστεί το εμβαδόν του τριγώνου αυτού (Εφαρμογή R=2).

Σκέψη ή Ανάλυση Αρχικά θα πρέπει να εκμεταλλευτούμε ότι στο πρόβλημα συνυπάρχουν το κέντρο βάρους G και το περίκεντρο Κ. Αυτά τα σημεία ορίζουν την ευθεία του Euler, στην οποία ανήκει και το ορθόκεντρο Η. Εδώ ωριμάζει η σκέψη να θεωρήσουμε το ορθόκεντρο Η. Επειδή έχουμε περιγεγραμμένο κύκλο θα πρέπει να θυμηθούμε ότι συμμετρικά του ορθοκέντρου ως προς τις πλευρές είναι σημεία του περιγεγραμμένου κύκλου του τριγώνου. Στην συνέχεια σκεφτόμαστε

Λ Λ Λ να λειτουργήσουμε το δεδομένο Α = 60°. Αυτό μας οδηγεί στην ισότητα ΒΚΓ = ΒΛΓ = 1 20°, όπου

Λ το μέσο του μικρού τόξου ΒΓ . Αυτόματα έρχεται στο μυαλό μας η μετρική σχέση ΒΓ = R .J3 . Θα μπορούσαμε, λοιπόν να έχουμε την aντιστοίχιση KG· .J3 � KG·R· .J3 = ΚG·ΒΓ που είναι το γινόμενο απέναντι πλευρών στο τετράπλευρο ΚGΒΓ. Οδηγούμαστε, λοιπόν, στην εξής λύση :

ΕΥΚΛΕΙΔΗΣ Β' 70 τ.2/60

Page 63: Ευκλειδης Β 70

--------------- Το βήμα του Ευκλείδη

Λύση Αρχικά θα αποδείξουμε ότι το τετράπλευρο GΒΓΚ είναι κυρτό δηλαδή ότι στην περίπτωση μας

Λ Λ αποκλείεται το G να βρίσκεται στο εσωτερικό του τριγώνου ΚΒΓ. Έστω Β <90° και Γ <90°.

σχήμα 1° Λ Λ

Το ορθόκεντρο Η του τριγώνου ΑΒΓ θα βρίσκεται στο εσωτερικό του. Από Α = 60° έπεται ότι ΒΚΓ = Λ Λ

2 ·60° = 1 20° = ΒΗΓ => ΒΗΚΓ εγράψιμο τετράπλευρο καθότι ΒΗΓ = 1 80° - 60° = 1 20°. Αν Λ το μέσο Λ

του μικρού τόξου ΒΓ, αυτό θα είναι συμμετρικό του Κ ως προς την πλευρά ΒΓ καθότι ΒΛΓ = 1 20° = Λ

ΒΚΓ . Έτσι έχουμε ότι το ευθύγραμμο τμήμα ΒΛ είναι συμμετρικό του ως προς την ΒΓ που σημαίνει ότι το τόξο ΒΗιΛ είναι συμμετρικό του τόξου ΒΗΚ όπου Η 1 είναι το συμμετρικό του Η ως προς ΒΓ που βρίσκεται, ως γνωστόν, στον περιγεγραμμένο κύκλο. Επειδή το σημείο Η1 βρίσκεται σε διαφορετικό ημιεπίπεδο με την ΒΓ από εκείνα που η ευθεία ΒΛ χωρίζει το επίπεδο άρα το σημείο Η, θα βρίσκεται σε διαφορετικό ημιεπίπεδο με την ΒΓ από εκείνα που η ευθεία ΒΚ χωρίζει το επίπεδο. Αυτό σημαίνει ότι για οποιαδήποτε σημείο G του ευθυγράμμου τμήματος ΗΚ το σχήμα GΒΓΚ είναι κυρτό. Αν λοιπόν G είναι το κέντρο βάρους του τριγώνου ΑΒΓ αυτό θα ανήκει στο ευθύγραμμο τμήμα ΗΚ ( ευθεία Euler) με το GΒΓΚ να είναι κυρτό. Από την σχέση GΓ=BG +KG .J3 έχουμε την σχέση GΓ·R = BG·R +KG·R· .J3

ή την σχέση GΓ·ΚΒ = ΒG·ΚΓ +ΚG·ΒΓ καθότι ΚΒ = ΚΓ = R και ΒΓ = R· .J3 . Η τελευταία σχέση Λ Λ R

προκύπτει από το γεγονός ότι (σχήμα 1 °) ΜΚΒ = 60° => ΚΒΜ = 30° άρα ΚΜ = - δηλαδή ΒΓ2 = 2

(2ΒΜ)2 � 4ΒΜ2 � 4{ R 2 - �2 ) � 3R2 => ΒΓ � R· ./3 . Επειδή στο ιαιpτό τετράπ4υρο GΒΓΚ ισχ6ει

ότι: GΓ·ΚΒ = ΒG·ΚΓ +ΚG·ΒΓ από το αντίστροφο του θεωρήματος του Πτολεμαίου κατανοούμε ότι το τετράπλευρο αυτό είναι εγγράψιμο. Αυτό σημαίνει ότι ο περιγεγραμμένος κύκλος στο τρίγωνο ΚΒΓ με την ευθεία του Euler KGH θα είχαν τρία κοινά σημεία πράγμα άτοπο εκτός αν G = Η, τελικά G = Η = Κ.

Άρα το τρίγωνο ΑΒΓ είναι ισόπλευρο με εμβαδόν 3R 2 .J3 . Για R=2 το εμβαδόν είναι 3.J3 .

4

Διερεύνηση

Στην περίπτωση

2 .J3 - R + - R = GΓ 3 3

Λ που είχαμε Β = 90° από την ισότητα BG + KG· .J3 = GΓ, προκύπτει ότι

ή (2 + .J3)R = 3 - � ΓΜ ή (2 + .J3)R = 2ΓΜ ή (2 + .J3)2 R 2 = 4ΓΜ2 ή 3

ΕΥΚΛΕΙΔΗΣ Β' 70 τ.2/61

Page 64: Ευκλειδης Β 70

-------------------------- Το βήμα του Ευκuίδη -------------------------­

(2 + ,/3) 2 R 2 � 4{ :' + 3R 2 J ή (7 + 4 J3 ) � Ι πράyμα άτοπο. Άρα δεν μπορεί Β � 90° ή f � 90°

με ταυτόχρονη ισχύ της BG + KG· J3 = GΓ, όταν

_ .. . - -σχήμα 2°

Λ Λ Λ Α =60°. Οι μετρικές σχέσεις προκύπτουν από το ορθογώνιο τρίγωνο ΑΒΓ με Α =60°, Γ =30° και ΒΓ=2R. !11 ρ{ιβλημα 2" ( Κ ,'\ llo Ο, i?'.lii'ioς 2 1H J7 ) Να βρεθούν οι συναρτήσεις f : R � R για τις οποίες ισχύει: f(f(x) +y) = f(f(x) - y) +4·f(x) ·y (1) για κάθε χ, y Ε R

Αν σκεφτούμε λίγο θα διαπιστώσουμε ότι η f(x) = Ο I R είναι μια τέτοια συνάρτηση που επαληθεύει την σχέση ( 1 ) για κάθε χ, y Ε R. Το ερώτημα που μπαίνει στις περιπτώσεις αυτές είναι: Υπάρχουν και άλλες συναρτήσεις f που να επαληθεύουν την ( 1 ) δηλαδή την σχέση f(f(x) +y) - f(f(x) - y) + 4·f(x) -y για κάθε χ, y Ε R; Η σχέση αυτή έχει διαφορετική συμπεριφορά από την σχέση f(x +y) - f(x - y) = 4xy για κάθε

R δ , ξ ' λ ' Θ ' , u + ν χ, y Ε που ο ηγει στην ε ης υση : " ετουμε χ + y = u, χ - y = ν, οποτε χ = -- και y =

2 u - ν , άρα μπορούμε να θεωρήσουμε f(u) - f(ν) = u2 - ν2 για κάθε u, ν Ε R.

2 Για ν = Ο παίρνουμε f(u) = u2 + f(O) για κάθε u Ε R. Άρα έχουμε την f(x) = χ2 + c για κάθε xER με c = f(O) . Η διαφορετικότητα στην περίπτωση μας βρίσκεται στο γεγονός ότι οι τιμές f(x) διατρέχουν το πεδίο τιμών της f που δεν γνωρίζουμε εκ των προτέρων ότι είναι το R. Έτσι οδηγούμαστε στην λύση που ακολουθεί:

Μια προφανής συνάρτηση είναι η f : R � R με f(x) = Ο αφού εύκολα βλέπουμε ότι επαληθεύει την σχέση ( 1 ) . Έστω ότι υπάρχει συνάρτηση f : R � R με f(x0):;t: Ο για κάποιο Χ0 Ε R. Αν στην σχέση ( 1 ) θεωρήσουμε y = f(x) τότε παίρνουμε f(2f(x)) = 4f(x) + f(O) ή f(2f(x)) = (2f (χ))2 + f(O) (2) για κάθε χ Ε R. Θεωρούμε τους τυχαίους πραγματικούς αριθμούς χ ι , χ2 . Για χ = χ2 και y = 2f(χ ι ) - f(x2) από την ( 1 ) παίρνουμε f(2f(x ι))=f(2f(x2)-2f(x ι ))+4f(x2)(2f(x ι )-f(x2)), ή f(2(f(x2}-f(x ι ))=f(2f(xι ))-8f(x2)f(x ι )+4[f(x2)]2 . Με βάση και την σχέση (2) παίρνουμε: f(2(f(x2)-f(x ι )))=[2f(x ι )]2-8f(x2) ·f(x ι ) + 4 [f(x2)]2+f(O) ή f(2(f(x2) - f(χ ι ))) = 4(f(x2) - f(χ ι ))2 + f(O) (3) που ισχύει για τυχαίους πραγματικούς αριθμούς χ ι , χ2 άρα και για κάθε χι , χ2 Ε R. Αν, λοιπόν, η διαφορά f(x2) - f(χ ι ) διατρέχει όλο το R τότε και η 2(f(χι ) - f(x2)) θα διατρέχει όλο το R οπότε ο τύπος θα είναι f(x) = χ2 + c με c = f(O) . Μένει να αποδείξουμε, ότι για κάθε Υ ο Ε R, υπάρχουν χ ι , χ2 Ε R ώστε f(x2) - f(χ ι ) = Υ ο ·

ΕΥΚΛΕΙΔΗΣ Β' 70 τ.2/62

Page 65: Ευκλειδης Β 70

-------------- Το βήμα του Ευκλείδη -------------'Εστω, λοιπόν, Υο τυχαίος πραγματικός αριθμός. Θεωρούμε χ = Χο και y Υ ο (f(xo) -::f:. Ο)

4f(X 0 )

οπότε για Xz = f(xo) + Υ ο και Χ ι = f(xo) -Υ ο η ( 1 ) γίνεται f(xz) - f(χ ι ) =

4f(X 0 ) 4f(X 0 )

=f(f(xo) + Υ 0 ) - f(f(xo) - Υ 0 ) = 4f(xo) · Υ 0 = Υ ο 4f(X 0 ) 4f(X 0 ) 4f(X 0 )

Άρα η διαφορά f(χ ι ) - f(xz) διατρέχει όλο το R .

Έτσι από την σχέση (3) παίρνουμε τον τύπο f(x) = χ2 + c με c = f(O)

Π ρόβλημα 3 " ( Θεώρη μα του ΚΛ ΡΑΘ ΕΟΔΩ ΡΗ, 1 873 - 1 950) Έστω τρίγωνο ΑΒΓ και εσωτερικό του σημείο

Κ. Να αποδειχθεί ότι:

� � � -(ΚΒΓ)· ΚΑ + (ΚΑΓ)· ΚΒ + (ΚΑΒ)· ΚΓ = Ο Σκέψη ή Ανάλυση

� � �

Παρατηρούμε ότι το άθροισμα (ΚΒΓ)- ΚΑ + (ΚΑΓ) · ΚΒ + (ΚΑΒ)· ΚΓ αποτελείται από

γινόμενα ίδιου τύπου δηλαδή εμβαδόν τριγώνου επί αντίστοιχη διανυσματική ακτίνα που δεν

ανήκει στις ευθείες που ορίζονται από τις πλευρές του τριγώνου αυτού, ενώ ορίζει την κοινή

πλευρά των δύο άλλων τριγώνων. Το φαινόμενο αυτό γίνεται κυκλικά. Άρα οδηγούμαστε στην

σκέψη : το άθροισμα αυτό να ανήκει σε δύο διαφορετικές ευθείες - διευθύνσεις που σημαίνει ότι

πρόκειται για το μηδενικό διάνυσμα.

λίJση Θεωρούμε υ 1 = ΓΓ με ΓΓ '_l ΑΚ και υ2 = ΒΒ ' με BB 'l_ ΑΓ. 'Ετσι έχουμε

υ 1 (ΚΑΓ) Γ Δ Α , δ , , λ , , - = = - . πο την ι ιοτητα του μερικου ογου παιρνουμε :

υ 2 (ΚΑΒ) ΔΒ � � ΚΔ = (ΚΑΓ) · ΚΒ+ (ΚΑΒ) · ΚΓ ή (ΚΑΓ)· ΚΒ + (ΚΑΒ) - Κr = [(ΚΑΓ)+(ΚΑΒ)} U Ε Α

(ΚΑΓ) + (ΚΑΒ) η

� � � � �

με ΚΑ Ε An � (ΚΒΓ) · ΚΑ Ε An . Τελικά (ΚΒΓ)· ΚΑ + (ΚΑΓ) · ΚΒ + (ΚΑΒ) · ΚΓ Ε An

./ ..... �

.·.· ...... ___ .

i �� � .. .... .

.. .

. __

. --· ·

·--.

.

..

. -�-- - - -· - · ·

-��- 'ι ·:χ�. / ·- • • • • - I • .-···_..� -........... .... _ ..... -------- :•:�::ι� . _

.

---···

�-.. .. __ .. . ·· ...... , -------- . . .. .. ____________ _

/ � ---··

_ r• Ι� "

.t /{"-- � - ·"= - �- - -' ' '

.,-, .�;.1ι - - . - -" ...... , Ί_ i � i. �

·- . __ _

·-...

·---- ·.

·-- - - - - - ---. · -... ___ __ _

Ί_ I

� � �

Όμοια αποδεικνύεται ότι (ΚΒΓ) - ΚΑ + (ΚΑΓ) · ΚΒ + (ΚΑΒ) - ΚΓ Ε Ae Αφού λοιπόν το

Page 66: Ευκλειδης Β 70

---------------------------- Το βήμα του Ευ�wη ----------------------------

ΤΡΙΓΩΝΟ - ΤΕΤΡΔΓΩΝΑ Γ. Πούλος

Το άρθρο αυτό γράφτηκε με σκοπό να παρουσιαστεί μια διδακτική πρόταση για την επίλυση Μαθηματικών προβλημάτων. Η πρόταση αυτή αποτελείται από δύο μέρη.

Το πρώτο αναφέρεται στην σημασία που έχει η σύλληψη μιας «βασικής ιδέας» για την λύση του μαθηματικού προβλήματος. Το δεύτερο αναφέρεται στην ενότητα των Μαθηματικών παρά τις επιμέρους πολλαπλές διακλαδώσεις τους. Έτσι το ίδιο πρόβλημα, είτε προσεγγίζεται από διαφορετικές μαθηματικές θεωρίες είτε από διαφορετικές σκοπιές (στοιχειώδης, ανώτερη) οι ιδιότητες του δεν μεταβάλλονται.

Ως παράδειγμα χρησιμοποιώ το μαθηματικό πρόβλημα που είναι γνωστό και ως πρόβλημα του χαμένου θησαυρού. Επί πλέον το πρόβλημα αυτό το αντιμετώπισα για πρώτη φορά τελειόφοιτος μαθητής στις εισαγωγικές εξετάσεις του πολυτεχνικού κύκλου το 1 9 7 6 και την ίδια λύση που έδωσα τότε, παρουσιάζω σήμερα.

Η λύση που ακολουθεί στηρίζεται στην βασική ιδέα της συμμετρίας

1 ο Πρόβλημα: Δίνεται ορθογώνιο τρίγωνο ΑΒΓ (Α=90°) και κατασκευάζουμε εξωτερικά τα τετράγωνα

ΑΒΔΕ και ΑΓΖΗ. Α ν Μ το μέσον της ΔΖ να αποδειχτεί ότι το τρίγωνο ΜΒΓ είναι ισοσκελές και ορθογώνιο

Λύση : Έστω ορθογώνιο τρίγωνο ΑΒΓ και τα εξωτερικά τετράγωνα ΑΒΔΕ και ΑΓΖΗ (σχήμα 1 )

Θεωρώ τα συμμετρικά σημεία Δ 1 , Ζ 1 ως προς τα Β και Γ αντίστοιχα. Στα τρίγωνα ΖΔ1 Δ και ΔΖ,Ζ τα τμήματα ΒΜ Και ΜΓ συνδέουν τα μέσα των πλευρών άρα

ΒΜ � Ζ�ι και ΓΜ � Δ�ι . Αρκεί να δείξουμε ότι zz, = ΔΔ1 . Τα ορθογώνια ΔΘΖ1 = ΖΘΔ 1 διότι

ΘΖ = ΘΔ = β+γ ΘΖ1 = ΘΔ 1 = β-γ. Επομένως ΜΒ=ΜΓ Ορθογώνιο : ΟΙ ΖΜΓ=ΖΔΖ1 ΔΜΒ =ΔΖΔ1 (Από την Παραλληλία ) και ΖΔΖΙ=45° +ΘΔΖ1 ΔΖΔ1=45° - Δ 1ΖΘ Άρα ΖΜΓ+ΒΜΔ=90°

Σχήμαl Το Πρόβλημα αυτό μπορεί να διατυπωθεί στην γενική του μορφή ως εξής

ΕΥΚΛΕΙΔΗΣ Β' 70 τ.2/64

Page 67: Ευκλειδης Β 70

-------------- Το βήμα του Ευκλείδη -------------

Πρόβλημα 2° Δίνεται τρίγωνο ΑΒΓ και κατασκευάζουμε εξωτερικά του τα τετράγωνα ΑΓ ΔΕ και

ΑΒΖΗ. Α ν Μ το μέσον της ΔΖ να αποδειχτεί ότι το τρίγωνο ΜΒΓ είναι ισοσκελές και

ορθογώνιο

Λύση Ακολουθούμε την ίδια μέθοδο όπως και στο προηγούμενο πρόβλημα (Μέθοδος συμμετρία )

και το σχήμα που προκύπτει είναι το σχήμα 2

Δηλαδή Δ ι , Ζ ι συμμετρικά των Δ,Ζ ως προς τα, Β,Γ αντίστοιχα. Φέρνουμε τις ΑΔ ΑΔ ι , ΑΖ, ΑΖι για τις οποίες ισχύει ΑΖ=ΑΖι και ΑΔ=ΑΔι διότι ΑΒ, ΑΓ μεσοκάθετοι των ΔΔ ι και ΖΖ ι

αντίστοιχα. Τα τμήματα ΜΒ= Δ,Ζ και ΜΓ= Δ z, (συνδέουν μέσα) 2 2 Αρκεί να δείξουμε ότι Δ ιΖ =ΔΖ ι Τα τρίγωνα ΑΔι Ζ=ΑΖ ιΔ διότι ΑΔ ι=ΑΔ ΑΖ=ΑΖι και ΔΑΖ1 =90° -Δ ι ΑΖ ι Δ ιΑΖ=90° -Δ 1ΑΖ 1 Άρα ΔΖ ι=ΖΔι δηλ ΜΒ=ΜΓ το ΜΒΓ ισοσκελές

Η

z1 Σχήμα 2

Ορθογώνιο : Οι γωνίες ΖΜΓ=ΜΔΖι =φ και ΔΜΒ=ΜΖΔι =ω διότι ΜΒ//Δ ιΖ καιΜΓ//ΔΖ ι Στο τρίγωνο ΚΖΔ1 ή Κεξ=ω+ΚΔ 1Ζ και στο τρίγωνο ΚΑΔ αλλά Κεξ=90°+ΑΔΚ. Άρα

ω+ΚΔιΖ=90°+ΑΔΚ ( 1 ) . Αλλά ΚΔ ιΖ=ΑΔΖι=φ+ ΑΔΚ (2) Επομένως ω + φ+ ΑΔΚ = 90° +ΑΔΚ ω + φ = 90°

Πόρισμα Από τις παραπάνω λύσεις προκύπτει ότι Με σταθερή την πλευρά ΒΓ και την κορυφή Α να κινείται στο ίδιο ημιεπίπεδο της ΒΓ τότε

το μέσο Μ της ΔΖ παραμένει σταθερό (Σχήμα 3)

ΕΥΚΛΕΙΔΗΣ Β' 70 τ.2/65

Page 68: Ευκλειδης Β 70

Η

Σχήμα 3 Ακολουθεί η λύση του ίδιου προβλήματος στα πλαίσια της αναλυτικής γεωμετρίας και των

μιγαδικών αριθμών Αναλυτική Γεω μετρία 1 ο Π ρόβλη μ α : Δίνεται ορθογώνιο τρίγωνο ABC (Α=90°) και κατασκευάζουμε εξωτερικά τα τετράγωνα

ABHD και ACFT. Αν Μ το μέσον της FH να αποδειχτεί ότι το τρίγωνο MBC είναι ισοσκελές και ορθογώνιο

Λύση : Θεωρούμε το ορθοκανονικό σύστηνα (AB,AC) (Σχήμα 4) . Έχουμε τα σημεία Α(Ο,Ο) Β( c,O)

C(O,b) Η( c, -c) F( -b, b) το μέσο Μ του τμήματος FH θα έχει συντεταγμένες ( c - b , b - c

) . Τότε 2 2 - b + c b - c - c - b b + c , , , ΜΒ = (-- , --) και MC = (-- , --) . Τα μετρα των ειναι ισα και το εσωτερικό τους 2 2 2 2 γινόμενο ίσο με μηδέν, επομένως το τρίγωνο MCB ισοσκελές και ορθογώνιο

F (-b, b) C (O , b)

Θεωρία μ ιγαδικών αριθ μών Π ρόβλη μα 2°

D

Σχήμα 4

Β (c , O)

Η (c , -c)

Δίνεται τρίγωνο ΑΒΓ και κατασκευάζουμε εξωτερικά του τα τετράγωνα ΑΓ ΔΕ και

ΕΥΚΛΕΙΔΗΣ Β' 70 τ.2/66

Page 69: Ευκλειδης Β 70

-------------- Το βήμα του Ευκλείδη ------------­

ΑΒΖΗ. Α ν Μ το μέσον της ΔΖ να αποδειχτεί ότι το τρίγωνο ΜΒΓ είναι ισοσκελές και ορθογώνιο

Λύση : Θεωρούμε Α(Ο,Ο) το σημείο του μιγαδικού επιπέδου, αν Β, Γ, Δ, Ζ Μ είναι εικόνες των

μιγαδικών αριθμών β, γ, δ ζ, μ αντίστοιχα και λάβουμε υπ όψιν την ιδιότητα:

Αν Ζz=Ζ1 (συνθ+ιημθ) <=> 1Ζ Ι 1= 1Zz1 και θ= (Z1'i2 ) η προσανατολισμένη γωνία του μιγαδικού

επιπέδου . Για τους μιγαδικούς γ, δ έχουμε δ=γ .J2 (συν π

+ι ημ 2: )=γ( l + ι ) 4 4

και για τους β, ζ ζ = β .J2 [ συν (-2: )+ι ημ (-2: ) ] = β ( 1 - ι ) 4 4

Τότε ο μιγαδικός μ = _!_ (δ+ ζ ) = _!_ ( γ( l + ι)+β( l - ι )) = _!_ (γ+β) + _!_ (γ - β) ι 2 2 2 2

Δ

ΜΒ = β - μ = β - _!_ (γ+β) _ _!_ (γ - β) ι = _!_ (β - γ) - _!_ (γ-β) i = _!_ (β - γ)( l + i)

2 2 2 2 2 - 1 1 1 . 1 1 ΜΓ = γ - μ = γ - - (γ+β) - - (γ - β) ι = = - (γ-β)ι- - (γ - β) ι = - (γ - β)( l- ι)

2 2 2 2 2

= i _!_ (β - γ)( l + i) = i ΜΒ Δηλαδή ΜΓ = i ΜΒ 2

Επομένως ΜΒ .l ΜΓ και 1 ΜΓ 1= 1 i ΜΒ 1 = 1 i 1 1 ΜΒ 1= 1 ΜΒ 1 Κλείνοντας προτείνω ένα παρόμοιο θέμα που είναι γνωστό και ως πρόβλημα του Vecten

((Θεωρούμε τρίγωνο ΛΒΓ Και κατασκευάζουμε εξωτερικά του τα τετράγωνα ΑΒΔΕ και ΑΓΖΗ τότε οι ευθείες ΒΖ, ΔΓ τέμνονται στο ύψος ΑΜ. »

ΕΥΚΛΕΙΔΗΣ Β' 70 τ.2/67

Page 70: Ευκλειδης Β 70

···· �rΕΩΜΕΤΡΙΚΕΣ•·.·.ΚΑ.τUUΥΕΣ κανονικό δεκaεπτaf1wνο

Σωτήρης Χρ. Γκουντουβάς

Οι γεωμετρικές κατασκευές για τους αρχαίους Έλληνες Γεωμέτρες ήταν παραδεκτές αν γινόταν με χρήση μόνο του κανόνα και του διαβήτη, δηλαδή με ευθείες και κύκλους που μπορούν να γραφούν με αυτά τα δύο όργανα. Κατασκευές που γινόταν με άλλα όργανα όπως γνώμονα ή μεσολάβο κλπ ήταν μη παραδεκτές. Επίσης κατασκευές που γινόταν με κινητική

γεωμετρία όπως η μέθοδος της νεύσης ήταν μη παραδεκτές. Δηλαδή χρήση άλλων γραμμών, όπως οι κωνικές τομές, η σπείρα, οι έλικες, η τετραγωνίζουσα του Ιππία κλπ, δεν επιτρεπόταν στις γεωμετρικές κατασκευές. Η απαίτηση του κανόνα και του διαβήτη μάλλον καθιερώθηκε στην Ακαδημία του Πλάτωνα. Η ευθεία στην πλατωνική φιλοσοφία αντιπροσώπευε το ανθρώπινο, τη ζωή και το φθαρτό αφού έχει αρχή και τέλος. Ο κύκλος που δεν έχει αρχή και τέλος αντιπροσώπευε το θείο, το άφθαρτο και το αέναο.

Ο Ευκλείδης που είχε θητεύσει στην Ακαδημία του Πλάτωνα και ασπαζόταν την αντίστοιχη φιλοσοφία, την απαίτηση του κανόνα και του διαβήτη την εισήγαγε εμμέσως στα Στοιχεία στα τρία πρώτα από τα πέντε συνολικά αιτήματά του( αξιώματα) . Τα τρία πρώτα αιτήματα είναι : 1 . Από κάθε σημείο μπορούμε να φέρουμε μοναδική ευθεία που να το συνδέει με οποιοδήποτε

άλλο σημείο . 2 . Το ευθύγραμμο τμήμα προεκτείνεται συνεχώς και ευθυγράμμως. 3 . Με κέντρο ένα σημείο και ακτίνα οποιοδήποτε τμήμα μπορούμε να γράψουμε κύκλο.

Το τέταρτο αίτημα αναφέρει ότι όλες οι ορθές γωνίες είναι ίσες, δηλαδή απαιτεί την ομοιογένεια του χώρου και το πέμπτο είναι το λεγόμενο "ευκλείδειο αίτημα" που αναφέρεται στην παραλληλία.

Βλέπουμε λοιπόν καθαρά την αποκλειστική αναφορά στην ευθεία και στον κύκλο δηλαδή στον κανόνα και στον διαβήτη . Όμως ρητή αναφορά του Ευκλείδη στην αποκλειστική χρήση των κανόνα και διαβήτη για τις γεωμετρικές κατασκευές δεν υπάρχει πουθενά. Επίσης ουδεμία τέτοια ρητή αναφορά υπάρχει στα έργα των άλλων αρχαίων Ελλήνων Γεωμετρών.

Παρόλα αυτά, πολλά αρχαία κείμενα αναφέρονται στην δεσπόζουσα θέση που κατέχουν η ευθεία και ο κύκλος στην Γεωμετρία.

Ο Πάππος στη Συναγωγή του (3°ς αιώνας μ.Χ) αναφέρει ότι αν ένα πρόβλημα μπορεί να επιλυθεί με ευθείες και κύκλους, τότε είναι λάθος να επιλυθεί με πιο σύνθετες καμπύλες (μέσα) .

Επίσης ο νεοπλατωνικός Πρόκλος (5°ς αιώνας μ.Χ) στα σχόλιά του στο 1 ° βιβλίο των Στοιχείων εξηγεί ότι η ευθεία γραμμή και ο κύκλος είναι τα πρωταρχικά θεμέλια των σχημάτων. Στις καμπύλες γραμμές εμπεριέχονται οι έννοιες της ευθείας και της κυκλικότητας και η κίνηση που περιγράφεται με ευθείες γραμμές και κύκλους μπορεί εύκολα να κατανοηθεί σαν φανερή και εύκολη αντίθετα, για παράδειγμα, από την κίνηση με τροχιά μια σπείρα.

Η επίλυση προβλημάτων με κανόνα και διαβήτη είναι αδύνατη για πολλά από αυτά. Για παράδειγμα τα περίφημα 'άλυτα προβλήματα' της αρχαιότητας - ο τετραγωνισμός του κύκλου, ο διπλασιασμός του κύβου ή Δήλιο πρόβλημα και η τριχοτόμηση γωνίας - δεν μπορούν να επιλυθούν με χρήση μόνο κανόνα και διαβήτη . Οι αποδείξεις της μη επιλυσιμότητας αυτών έγιναν τον 1 8° και 1 9° μ.Χ αιώνα.

Τώρα αναφορικά με την απαίτηση του κανόνα και του διαβήτη έχουμε να αναφέρουμε τα εξής. Ο Ιταλός μαθηματικός Lorenzo Mascheroni ( 1 750- 1 800) στο έργο του Geometrίa del Compasso (Paνia, 1 797) "Γεωμετρία του διαβήτη" αποδεικνύει ότι κάθε πρόβλημα που μπορεί να επιλυθεί με κανόνα και διαβήτη , μπορεί να επιλυθεί με χρήση μόνο του διαβήτη .

Το αποτέλεσμα αυτό είναι γνωστό σαν θεώρημα Mohr-Mascheroni. Ο Δανός μαθηματικός George Mohr είχε δημοσιεύσει την απόδειξη αυτή σε μια μικρή αφανή εργασία του, με τίτλο

ΕΥΚΛΕΙΔΗΣ Β' 70 τ.2/68

Page 71: Ευκλειδης Β 70

-------------- Το βήμα του Ευκλείδη

Euclίdes Danίcus (Ευκλείδης εκ Δανίας) που κυκλοφόρησε στην Κοπεγχάγη το 1 672. Η εργασία αυτή βρέθηκε σε ένα παλαιοβιβλιοπωλείο της Κοπεγχάγης το 1 928 από ένα φοιτητή μαθηματικών που την έδειξε στον καθηγητή του J. Hjelmsleν, ο οποίος αναγνώρισε την αξία της και την αναδημοσίευσε το ίδιο έτος.

Επίσης ο Γάλλος μαθηματικός Jean-Victor Poncelet ( 1 788- 1 867) απέδειξε ότι κάθε πρόβλημα που μπορεί να επιλυθεί με κανόνα και διαβήτη, μπορεί να επιλυθεί με χρήση του κανόνα και διαβήτη σταθερού ανοίγματος (σκουριασμένος διαβήτης) . Την απόδειξη αυτή την τελειοποίησε ο Ελβετός Γεωμέτρης Jacob Steiner ( 1 790- 1 863) και για αυτό η παραπάνω πρόταση αναφέρεται σαν θεώρημα Poncelet-Steiner. Το θεώρημα αυτό προϋποθέτει κανόνα και ένα μόνο κύκλο με το κέντρο του, που χαράχθηκε στο επίπεδο άπαξ. Στις αρχές του 20°υ αιώνα αποδείχθηκε ότι δεν είναι απαραίτητος ούτε ολόκληρος ο 'κύκλος Poηcelet-Steiηer' , όπως λέγεται. Ότι χρειάζεται μόνο ένα τόξο αυτού του κύκλου οσοδήποτε μικρό, μαζί με το κέντρο του .

Από τις έως τώρα διαθέσιμες πηγές, φαίνεται ότι μάλλον οι αρχαίοι Έλληνες Γεωμέτρες δεν γνώριζαν τα δύο παραπάνω θεωρήματα.

Βλέπουμε λοιπόν ότι ο συνδυασμός κανόνα και διαβήτη για τις γεωμετρικές κατασκευές, είναι άλλες φορές ανεπαρκής, και άλλες πλεονασματικός.

Το πρόβλημα της κατασκευής ενός κανονικού πολυγώνου με ν πλευρές είναι ισοδύναμο με την διαίρεση του κύκλου σε ν ίσα τόξα. Οι αρχαίοι Έλληνες Γεωμέτρες είχαν κατασκευάσει με κανόνα και διαβήτη τα κανονικά πολύγωνα με πλήθος πλευρών: α) 4, 8 , 1 6 , 32, 64, . . . δηλαδή της μορφής 2n, η = 2, 3 , 4, 5 , 6, . . . β) 3 , 6 , 1 2 , 24, 4 8 , . . . δηλαδή της μορφής 3 ·2n, η = Ο , 1 , 2 , 3 , 4, . . . γ) 5 , 1 0 , 20, 40, 80, . . . δηλαδή της μορφής 5 ·2n, η = Ο, 1 , 2 , 3 , 4, . . . δ) 1 5 , 30, 60, 1 20, . . . δηλαδή της μορφής 3 · 5 ·2n, η = Ο, 1 , 2, 3 , 4, . . .

Οι κατασκευές των παραπάνω κανονικών πολυγώνων υπήρχαν στα Στοιχεία του Ευκλείδη , αλλά και σε έργα άλλων Γεωμετρών, όπως η Αλμαγέστη του Κλαύδιου Πτολεμαίου.

Όμως το πρόβλημα της κατασκευής κανονικών πολυγώνων στην γενική του μορφή παρέμενε άλυτο για πολλούς αιώνες. Ο Αρχιμήδης (287 -2 1 2 π .Χ) στην σωζόμενη πραγματεία του <<llεpί του κανονικού επταγώνου» είχε δώσει τρόπο κατασκευής του, όμως με κινητική γεωμετρία, και ως εκ τούτου η κατασκευή ήταν μη παραδεκτή γιατί δεν γινόταν αποκλειστικά με κανόνα και διαβήτη .

Το κανονικό δεκαεπτάγωνο δεν βρισκόταν στον παραπάνω πίνακα και δεν υπήρχε από τους αρχαίους Γεωμέτρες καμία μνεία για την κατασκευή του .

Ο Carl Friedrich Gauss ( 1 777- 1 855) το 1 796 σε ηλικία 1 9 ετών απέδειξε ότι το κανονικό δεκαεπτάγωνο μπορεί να κατασκευαστεί με κανόνα και διαβήτη . Στη συνέχεια έλυσε το πρόβλημα στη γενική του μορφή του αποδεικνύοντας ότι ένα κανονικό πολύγωνο είναι κατασκευάσιμο με κανόνα και διαβήτη αν και μόνο αν το πλήθος των πλευρών του είναι της

μορφής 2α, ή 2α·Fα·Fb · · · Fn , όπου Fn= 2 2" + 1 , η=Ο, 1 ,2 , . . . είναι πρώτοι αριθμοί και διαφορετικοί

μεταξύ τους. Οι αριθμοί Fn είναι γνωστοί ως (πρώτοι) αριθμοί του Fermat. Ο Fermat ( 1 60 1 - 1 665)

ισχυρίστηκε ότι όλοι αυτοί οι αριθμοί είναι πρώτοι. Πράγματι για η = Ο, 1 ,2 ,3 ,4 προκύπτουν οι αριθμοί F0 = 3 , F 1 = 5 , F2 = 1 7 , F3 = 257, F4 = 65537 που είναι πρώτοι. Όμως για η = 5 ο αριθμός που προκύπτει, ο F5 = 4294967297, δεν είναι πρώτος αφού διαιρείται με το 64 1 , όπως απέδειξε ο L. Euler το 1 754 (F 5 = 232 - 1 = 64 1 χ67004 1 7) .

Συνοψίζοντας τα παραπάνω είναι αδύνατη η κατασκευή με αποκλειστική χρήση κανόνα και διαβήτη ενός κανονικού πολυγώνου με πλήθος πλευρών 7, 9, 1 1 , 1 3 , 1 4, 1 8 , 1 9, 2 1 , 22 κλπ.

Η απόδειξη του Gauss παρουσιάστηκε στο βιβλίο του Dίsquίsίtίones Arίthmetίcae (Αριθμητικές έρευνες) . Μπορεί να θεωρηθεί ως μία εφαρμογή της θεωρίας Galois, αν και ο Evarist Galois ( 1 8 1 1 - 1 832) δεν είχε ακόμη γεννηθεί την εποχή που ο Gauss έκανε την

ΕΥΚΛΕΙΔΗΣ Β' 70 τ.2/69

Page 72: Ευκλειδης Β 70

-------------- Το βήμα του Ευκλείδη

ανακάλυψή του . Επίσης ούτε και ο Ga1ois φαίνεται να ενεπνεύσθη τίποτε από την απόδειξη του Gauss, όταν εργαζόταν στη θεωρία των εξισώσεων.

Ο Gauss πάντως αν και άφησε πίσω του ένα τεράστιο έργο τόσο στα μαθηματικά όσο και στη φυσική - θεωρείται μαζί με τον Αρχιμήδη και τον Νεύτωνα οι κορυφαίοι μαθηματικοί όλων των εποχών - θεωρούσε την κατασκευή του κανονικού δεκαεπταγώνου σαν το κορυφαίο επίτευγμά του . Η επιθυμία του να χαραχτεί πάνω στον τάφο του το κανονικό δεκαεπτάγωνο ήταν όμοια με αυτή του Αρχιμήδη με την εγγεγραμμένη σφαίρα σε κύλινδρο. Όταν ο Gauss έκανε λόγο για αυτή του την επιθυμία σε ένα μαρμαρογλύπτη, αυτός του απάντησε ότι το κανονικό δεκαεπτάγωνο θα μοιάζει με κύκλο. Πάντως ο ανδριάντας του που βρίσκεται στην γενέθλια πόλη του, το Braunschweig της Γερμανίας, εδράζεται σε βάθρο σχήματος κανονικού δεκαεπταγώνου.

Ο Gauss αν και απέδειξε ότι το κανονικό δεκαεπτάγωνο μπορεί να κατασκευαστεί με κανόνα και διαβήτη, δεν έδωσε μέθοδο κατασκευής του. Ο πρώτος που έδωσε μία μέθοδο για την κατασκευή του ήταν ο Johannes Erchinger περίπου το 1 800, μερικά χρόνια μετά την εργασία του Gauss . Η κατασκευή είναι αρκετά πολύπλοκη και γίνεται σε 64 βήματα.

Πιο κάτω δίνουμε την κατασκευή του H.W. Richmond ( 1 893) που είναι προσαρμοσμένη σε αυτή του Erchinger και γίνεται σε λιγότερα βήματα. Β Η :VΕ ΑΤΛ

1 . Δίνεται ο κύκλος (O,R) και μια διάμετρός του στο σημείο Ρ 1 του κύκλου. 2 . Φέρνουμε την κάθετη ακτίνα ΟΒ στην διάμετρο.

3 . Στην ΟΒ παίρνουμε το σημείο J έτσι ώστε OJ= _!_ ΟΒ 4

4. Φέρνουμε το JP 1 και παίρνουμε το σημείο Ε στην ΟΡ 1 τέτοιο ώστε η γωνία OJE να είναι το Υ4 της γωνίας ΟJΡ ι .

5 . Παίρνουμε τ ο σημείο F στην αρχική διάμετρο τέτοιο ώστε η γωνία EJF = 45° . 6 . Κατασκευάζουμε το ημικύκλιο με διάμετρο FP 1 που τέμνει την ΟΒ στο Κ. 7 . Κατασκευάζουμε το ημικύκλιο με κέντρο Ε και ακτίνα ΕΚ που τέμνει την ΟΡ ι

στο σημείο Ν4. 8 . Φέρνουμε την κάθετη στο Ν4 που τέμνει τον κύκλο στο σημείο Ρ 4 .

Το τόξο Ρ 1 Ρ4 είναι τα � του κύκλου. Με την βοήθεια του τόξου Ρ ι Ρ4 βρίσκουμε τις υπόλοιπες 1 7

1 5 κορυφές του κανονικού 1 7 -γωνου με την εξής σειρά : Ρ7 � Ρ ι ο� Ρ 1 3 � Ρ ι 6� Ρ2 � Ps � Ps � Ρ ι ι � Ρ ι4 � Ρ ι 7 � Ρ3 � Ρ6 � Ρ9 � Ρ 1 2� Ρ ι s .

Οι παρακάτω διευθύνσεις στο διαδύκτιο αναφέρονται στο κανονικό δεκαεπτάγωνο http://mathworld. wolfram.com meptadecagon.html http://en. wίkipedίa.orglwίkVHeptadecagon

ΕΥΚΛΕΙΔΗΣ Β' 70 τ.2/70

Page 73: Ευκλειδης Β 70

Πρίντεζης Ιωάννης

Αγαπητοί συνάδελφοι θα ήθελα να σας επισημάνω μια αντίφαση που υπάρχει στα σχολικά βιβλία της Γενικής

Παιδείας Γ Λυκείου σχετικά με τον ορισμό της μέσης ταχύτητας. Είμαι βέβαιος ότι η αντίφαση αυτή θα έχει επισημανθεί και από άλλους συναδέλφους αλλά δεν ξέρω τι λύση έχει δοθεί.

Στη σελίδα 20 του σχολικού βιβλίου αναφέρεται ότι

, , διανυθέν διάστημα μεση ταχυτητα = , (τύπος Ι)

χρονος

Επειδή η έννοια του διανυθέντος διαστήματος αναλύεται διεξοδικά στην εφαρμογή 2 σελίδα 34 και αυτό ορίζεται πάντα θετικό, οδηγούμαστε στο συμπέρασμα ότι και η μέση ταχύτητα είναι πάντα θετική .

Αντίθετα στη σελίδα 2 1 αναφέρεται ότι αν x=f(t) είναι η συνάρτηση που εκφράζει τη θέση του κινητού κατά τη χρονική στιγμή t, τότε η μέση ταχύτητα κατά το χρονικό διάστημα από to μέχρι t0+h είναι

Δ χ ν = h (τύπος Π)

όπου Δχ= x2-x 1 =f(t0+h)-f(t0) . Στον τύπο αυτό δεν υπάρχει καμιά απόλυτη τιμή και συνεπώς μπορεί να οδηγήσει και σε αρνητικά αποτελέσματα για τη μέση ταχύτητα. Τον τύπο αυτόν χρησιμοποιεί το βιβλίο και για την επίλυση της εφαρμογής 1 σελίδα 24 όπου όμως κατ ' ευτυχή σύμπτωση τα αποτελέσματα είναι έτσι κι ' αλλιώς θετικά !

Νομίζω ότι είναι φανερές οι αντιφάσεις στις οποίες μπορούν να οδηγήσουν αυτοί οι δύο τύποι. Για παράδειγμα αν ένα κινητό κινείται πάνω στον άξονα προς τα αριστερά με ταχύτητα -2 rn!s, ο τύπος Ι βγάζει v =2 rn!s ενώ ο τύπος Π βγάζει v =-2rn!s. Ακόμα, αν ένα αυτοκίνητο πάει από την Αθήνα στη Θεσσαλονίκη με 1 OOkm/h και επιστρέψει με την ίδια ταχύτητα, ο τύπος Ι βγάζει v = 1 00 km/h ενώ ο τύπος Π βγάζει v =O km/h! ! ! Είναι περιττό να τονίσουμε τη σύγχυση που δημιουργείται όταν το κινητό αλλάζει ενδιάμεσα τη φορά της κίνησης.

Επειδή οι μαθητές εξετάζονται πανελλαδικά στο αντικείμενο αυτό και ολόκληρη η καριέρα τους μπορεί να εξαρτάται από μερικές μονάδες, πιστεύω ότι πρέπει να ξεκαθαριστεί αυτή η ασάφεια. Η προσωπική μου άποψη είναι ότι σωστός είναι ο τύπος Π αλλά δεν συμφωνούν μαζί μου όλοι οι συνάδελφοι. Θα σας ήμουν ευγνώμων αν προκαλούσατε μια υπεύθυνη απάντηση από τη μεριά του υπουργείου ή του Παιδαγωγικού Ινστιτούτου ώστε να ξέρω τι θα απαντώ στους μαθητές μου κάθε φορά που με ρωτάνε. 'Ισως επίσης να ήταν θετική η δημοσίευση αυτής της επιστολής ώστε να προκληθεί η σχετική συζήτηση .

ΕΥΚΛΕΙΔΗΣ Β' 70 τ.2/71

Page 74: Ευκλειδης Β 70

40 .

.

Ε υ κλεl δ π ς π ρ οτ εlvειι

.

«Η καρδιά των μαθηματικών είναι τα προβλήματα και οι λύσεις και ο κύριος λόγος ύπαρξης του μαθηματικού είναι να λύνει προβλήματα». Ρ. R. HALMOS

Επιμέλεια: Γ. ΤΡ Ι ΑΝ ΤΟΣ, Ν. ΑΝΤΩΝΟ Π ΟΥ ΛΟΣ, ΘΑΝ . ΚΥ Ρ Ι Α ΚΟ Π Ο Υ ΛΟΣ

ΑΣ ΚΗΣΗ 1 29 (τΕΥΧΟΥΣ 66) Θα δείξουμε τώρα ότι: « Αν δύο τρίγωνα είναι

Ονομάζουμε ρ την ακτίνα του εγγεγραμμένου όμοια, τότε ο λόγος των ακτίνων των εγγεγραμ­

κύκλου σε τρίγωνο ΑΒΓ. Οι εφαπτόμενες του μένων κύκλων τους είναι ίσος με τον λόγο των

κύκλου αυτού που είναι παράλληλες στις πλευ- περιμέτρων τους » . Πράγματι, έστω ότι τα τρί­ρές του τριγώνου ΑΒΓ σχηματίζουν με τις γωνα ΔΕΖ και ΔΈ'Ζ' είναι όμοια. Ονομάζουμε πλευρές του τριγώνου αυτού τα τρίγωνα r, r' τις ακτίνες των εγγεγραμμένων κύκλων τους, ΑΑ,Α2 , ΒΒ,Β2 , ΓΓ,Γ2 • Ονομάζουμε ρ , , ρz , ρ3 τις αντιστοίχως. Από ομοιότητες τριγώνων ( τα τρίγω­ακτίνες των εγγεγραμμένων κύκλων στα τρίγω- να ΟΔΜ και Ο ' Δ 'Μ ' είναι όμοια) βρίσκουμε ότι: να αυτά, αντιστοίχως. Να δείξετε ότι :

ρ = ρ, + ρz + ρ3 ·

(Προτάθηκε από τον συνάδελφο ΑΝΤΩΝ Η ΚΥ­

Ρ ΙΑΚΟ Π ΟΥ ΛΟ - Αθήνα ) Α

Δ

Ε Ζ Λ ΥΣ Η ( Από τον ίδιο )

Γ

Δ '

Ζ '

Ονομάζουμε Ρ την περίμετρο του τριγώνου ΑΒΓ

και Ρ, , Ρ2 , Ρ3 τις περιμέτρους των τριγώνων

εύκολα ότι: Ρ, + Ρ2 + Ρ3 = Ρ (I)

r ΔΜ ΜΕ ΕΚ ΚΖ ΖΛ ΛΔ - =--=--=--=--=--=--r' Δ'Μ' ΜΈ' Ε'Κ' Κ'Ζ' Ζ'Λ' Λ'Δ' 'Ε , r ΔΜ + ΜΕ + . . . + ΛΔ τσι, εχουμε: - = =

r' Δ'Μ' + Μ'Ε' + . . . + Λ'Δ' ΔΕ + ΕΖ + ΖΔ Ε δ ' θ ' ,

Δ'Ε' + Ε'Ζ' + Ζ'Δ' . πει η κα ενα απο τα

τρίγωνο ΑΒΓ , σύμφωνα με την προηγούμενη πρόταση έχουμε : 12 = Ρ, El. = Pz .!2 = p3 ' ρ Ρ ' ρ Ρ ' ρ Ρ

Έτσι, έχουμε: ρ, + ρz + ρ3 = Ρ, + Pz + Ρ3 � Ρ = l ρ Ρ Ρ '

Λύση έστειλε και ο συνάδελφος ΗΛΙΑΣ

ΚΩΝ/ΝΟΣ - Αλιβέρι Ευβοίας .

ΑΣΚΗΣΗ 1 30 (τΕΥΧΟΥΣ 66)

Να αποδείξετε ότι για οποιονδήποτε μιγαδικό

z, z * Ο , ισχύει: lz + i l + l� + i l > l z + �� (1)

(Προτάθηκε από τον συνάδελφο ΝΙΚΟ ΑΝΤΩ­

ΝΟΠΟΥ ΛΟ - Ιλιον) . Λ ΥΣΗ 1 " (από τον ίδιο) Αν z = i ή z = -i , τότε το ζητούμενο προφανώς ισχύει. Α ν z * ±i , τότε για την απόδειξη της ( 1 ),

ΕΥΚΛΕΙΔΗΣ ' 70 τ.2/72

Page 75: Ευκλειδης Β 70

------------- Το βήμα του Ευκλείδη -------------

αρκεί να αποδείξουμε ότι: l z + i l )1 ;ίi l > lz + ±Ι , � Im(±) > Ο � Ι± + i l > Ι± - i l , , 1 . 1 Ι z - i l 1 1 1 , , η αρκει z + ι + � > z +

� , η αρκει

. l z - i l l z2 - i 2 1 , , lz + ι l +� > lz l , η αρκει

l z - i l l z + i l · l z - i l lz + i l +-- > ή αρκεί I � I� '

l z l · l z + i l "--'--'------...,. ---'- + 1 > lz + i l (2) . Είναι: l z - ι l

lz - i l � l zi + 1 => _M_ �_Iz_l => lzl · l z + i l � l zl · l z + i j l z - i l l zl + 1 l z - il l zl + 1

lzl · l z + i l lzl · l z + i j => ι · ι + 1 � ι ι + 1 z - ι z + 1 = lzl · l z + i j + lzl + 1 > l zl · l z + i l + l z + i l = lz + i l l zl + 1 lzl + 1 όπου, οι ισότητες j z+ ij = I� + 1 και l z - i j = l z l + 1 δεν

ισχύουν συγχρόνως, (το σύστημα l z + i l = j z j + 1 και

j z - i l = lz l + 1 έχει μοναδική λύση την z = Ο , που

l z l · l z + i j αποκλείεται) οπότε

I · ι + 1 > lz + i j που είναι z - ι

η (2) από την οποία απορρέει η ( 1 ) . Λ ΥΣΗ 2 11 (από τον συνάδελφο Γ Ι ΩΡΓΟ ΤΡΙΑ­

ΝΤΟ - Αθήνα)

Αποδεικνύουμε το παρακάτω Λήμμα:

Για κάθε z ε C, z * Ο , ισχύει η ισοδυναμία

l z + i j > lz - i l � Im(z) > Ο ( 1 ) . Πράγματι

l z+ ij > lz- il �Ιz+il 2 > jz-il2 �(z+iX�-i) > (z-iX�+i) � 2i(z - �) < Ο � 4i 2 Im(z) < Ο � lm(z) > Ο . Επίσης, συμφώνως προς την ( 1 ) , έχουμε

και την ακόλουθη ισοδυναμία: - z Im(z) < Ο � lm(z) > Ο � Im(-2 ) > Ο

l z l

Για την απόδειξη της ανισότητας, έχουμε:

1 ) Έστω Ιη(z)>Ο . Τότε, επειδή ισχύει l z + i l > l z - i j ,

αρκεί να δειχθεί ότι: j z - i j + Ι± + i l � lz +±Ι , που

· · ι · ι 1 1 · ι ι · 1 · ι ι 1 1 ισχυει αφου z - ι + � + ι � z - ι + � + ι = z + �

2) 'Εστω Im(z) < Ο . Τότε, επειδή ισχύει ��+� > ��-� , αρκεί να δειχθεί ότι: l z + i j + Ι± - i l � lz +±Ι , που

· · ι · ι 1 1 · ι ι · 1 · ι ι 1 1 ισχυει αφου z + ι + � - ι � z + ι + � - ι = z + � .

3) Έστω Im(z) = Ο , δηλ. z = k ε R* . Τότε, η

αποδεικτέα ανισότητα, ισοδυναμεί με την

j k + i l + �� + i l > lk + �� ή - το αυτό- με την

.Jk2 + 1 + �:2 + 1 > ιk +�ι � (k2 + 1)(:2 + 1) > 0

που, προφανώς, ισχύει.

Λ ΥΣΗ 311 (Γεωμετρική- από τον συνάδελφο

Γ Ι ΩΡΓΟ Τ ΑΣΣΟΠΟΥ ΛΟ - Αθήνα)

'Εστω z = α + βi , με l z l = ρ > Ο , τότε _!_ = � -� i . z ρ ρ

Υποθέτουμε αρχικά ότι αβ ::;; Ο , οπότε η εικόνα

A(z) ανήκει στην ευθεία ε ι : y = � χ και η εικόνα α

Β(_!_) στην ευθεία ε2 : y = -�χ ( συμμετρική της z α

ε ι ως προς χ 'χ ). Αν θεωρήσουμε και τα σημεία

Γ(-i) και Β'(-_!_) , τότε αρκεί να δείξουμε ότι: z α 2 (ΑΓ)+(ΒΓ) > (ΑΒ') . Επειδή είναι χΑ · χ8 = (-) > 0 , ρ

ΕΥΚΛΕΙΔΗΣ Β' 70 τ.2/73

Page 76: Ευκλειδης Β 70

-------------- Το βήμα του Ευκλείδη -------------­

τα σημεία Α, Β βρίσκονται προς το αυτό μέρος του ΣημΕίωση : Η αποδεικτέα σχέση ισχύει γενικότερα

άξονα y'y , οπότε το συμμετρικό Β1 του Β ως προς

y'y θα βρίσκεται στην προέκταση της ΟΑ προς το

μέρος του Ο.

Άρα (ΑΓ) + (ΒΓ) = (ΑΓ) + (Β1Γ) > (ΑΒ1 ) > (ΑΒ')

αφού ο άξονας χ'χ είναι μεσοκάθετος του Β1Β'

και το Α βρίσκεται προς το μέρος του Β' .

Υ A(z)

χ

Γ(-i) Σχ. l Ειδικά όταν α = Ο , οπότε τα Α ,Β ανήκουν στον

y'y , έχουμε : (ΑΓ) + (Β1 Γ) ::::0: (ΑΒ1 ) > (ΑΒ')

Ενώ όταν β = Ο , οπότε τα Α , Β ανήκουν στον

χ 'χ έχουμε: (ΑΓ) + (Β1Γ) > (ΑΒ1 ) = (ΑΒ') .

χ '

χ '

Υ � Α

ο

Β ι

Γ

Υ

Β' ο Β ι

Γ

Β'

χ

Β

Σχ. 2

Β Α χ

αν στη θέση του i τεθεί λi με λ ;;t. Ο .

Λ ΥΣ Η : (Από τον συνάδελφο Σωη) ρη Ε. Λουρί­

δα - Λθi1να)

ΛίJση :

Αν z = α Ε IR * τότε lz+il +j; +iι =�α2 +1 +� �2 +1 >

>Ν +f[: = lαl + l�l ::::: Jα + � J = Jz + lJ Εξάλλου :

l z + i l = lz - i l => ( z + i ) (� + i ) = ( z - i ) (� + i ) => => i (� - z) = i ( z - �) => z - � = ο => z = � => z Ε IR * οπότε ισχύει η ζητούμενη από το προηγούμενο

βήμα. Το ίδιο και αν ll + i l = ll - i J . Α ν z Ε C -IR τότε και _!_ Ε C -IR . z Έστω l z + i l < lz - i l ( 1 ) και

ι; + il < ι; -iι (2) . Τότε (2) => 11 + i · zl < 1 1 - i · zl =>

=>l-i2 + i · zl < l-i2 - i · zl =>lz - il < lz + il πράγμα

άτοπο λόγω της ( 1 ) . Άρα οι ( 1 ), (2) αποκλείεται

να ισχύουν συγχρόνως, Αν λοιπόν l z + i l > lz - i l , τότε:

l z + i l + �� + i l > lz - i l + jl + i l ::::0: J z + ll ο · ι 1 · ι ι 1 · ι · μοιως αν ;- + ι > ;- - ι , τοτε:

Λύση έστειλε επίσης ο συνάδελφος Η Λ ΙΑΣ

ΚΩΣΤΛΣ - Αλιβi:ρ ι, Γ ιιίφγος Δεληστάθης -

Π ατήσια .

ΑΣ Κ Η Σ Η 1 3 1 ( ΤΕΥΧΟΥΣ 67 )

Αν οι αριθμοί α, β ικανοποιούν τις εξισώσεις Σχ. 3

α3 - 3α2 + 5α = 1 , β3 - 3β2 + 5β = 5 , τότε να υπο­ΕΥΚΛΕΙΔΗΣ Β. 70 τ.2/74

Page 77: Ευκλειδης Β 70

-------------- Το βήμα του Ευκλείδη --------------

λογίσετε την τιμή του αθροίσματος α + β .

( Προτάθηκε από τον συνάδελφο ΑΝΤΩΝΗ

ΙΩΑΝΝΙΔΗ - Λάρισα )

ΛΥΣΗ 18 : ( Από τον συνάδελφο ΓΙΑΝΝΗ

ΤΣΟΠΕΛΑ - Αμαλιάδα )

Έχουμε:

α3 - 3α2 + 3α - 1 = -2α } { (α - 1)3 = -2α 3 2

=> 3 β - 3β + 3β - 1 = 4 - 2β (β - 1) = 4 - 2β α-l=χ { χ3 = -2(χ + 1) {χ3 = -2Χ - 2

β�y y3 = 4 - 2(y + 1) =>

y3 = -2y + 2

=> χ 3 + y3 = -2( χ + Υ) => χ 3 + y3 + 2( χ + Υ) = Ο => (x + y)(x2 - xy + y2 + 2) = 0 (1)

Επειδή είναι χ 2 - yx + y2 + 2 > Ο , για κάθε

χ Ε R (αφού Δχ = -3y2 - 8 < 0 ,για κάθε y E R )

Η ισότητα ( 1 ) δίνει: χ + y = Ο => α + β = 2 .

Λ ΥΣΗ 28 : ( Από τον συνάδελφο Γ ΙΩΡΓΟ

Α Π ΟΣΤΟ Λ ΟΠΟΥ ΛΟ - Μεσολόγγι.)

Θεωρούμε την συνάρτηση f(x) = χ3 - 3χ2 + 5χ

χ Ε R . Προφανώς, f(α) = 1 , f(β) = 5 . Επειδή

f'(x) = 3χ2 - 6χ + 5 > Ο , για κάθε χ Ε R , η f είναι

γνησίως αύξουσα και συνεπώς 1- 1 στο R .

Παρατηρούμε : f(x) = (χ - 1)3 + 2(χ - 1) + 3 , οπότε

αν τεθεί χ - 1 = y και g(y) = y3 + 2y , είναι:

f(y + 1) = g(y) + 3 � g(y) = f(y + 1) - 3 .

με την συνάρτηση g επίσης 1 - 1 και περιττή στο

R . 'Ετσι, g(α - 1) = f(α) - 3 = 1 - 3 = -2 και

g(β - 1) = f (β) - 3 = 5 - 3 = 2 , οπότε

g( α - 1) = -g(β - 1) => g( α - 1) = g(l - β) ::::> α - 1 = 1 - β ::::> α + β = 2

Λύσεις έστειλαν επίσης, οι συνάδελφοι: Γιάννης Η λιόπουλος - Καλαμάτα, Αθανάσιος

Καλάκος - Κ. Πατήσια, Ιωάννα Γιαννακοπού­

λου - Λαμία, Γιώργος Τσαπακίδης - Κυψέλη

Αιτ/νίας, Βασίλειος Νικολάκης - Θες/νίκη,

Κων/νος Ηλίας - Αλιβέρι, Γιώργος Ν ικητάκης -

Σητεία, Βαγγέλης Μουρούκος - Αγρίνιο, Ρο­

δόλφος Μπόρης - Δάφνη, Μαραγκουδάκης

Παύλος - Πειραιάς και ο Π ολιτικός Μηχανικός

Ι ωάννης Ανδρής - Αθήνα.

ΑΣΚΗΣΗ 1 32 ( ΤΕΥΧΟΥΣ 67 ) Α ν για τους ακεραίους αριθμούς χ, y ισχύει

9χ2 + 4y2 - 9x2y2 + 6xy + 2 = Ο , να δειχθεί ότι ο

αριθμός Α = 1 000(χ4 + y4 ) + 40 1 είναι τέλεια

τέταρτη δύναμη ακεραίου αριθμού.

( Προτάθηκε από τον συνάδελφο ΓΙΩΡΓΟ ΑΠΟ­

ΣΤΟΛΟΠΟΥ ΛΟ - Μεσολόγγι )

ΛΥΣΗ: ( Από τον συνάδελφο ΒΑΓΓΕΛΗ

ΜΟΥΡΟΥΚΟ - Αγρίνιο ) . Έχουμε:

9χ2 +4y -9x2y +6xy+2 = 0 � 9χ2 +4y + 12xy =9x2y +6xy+ 1 - 3� (3x +2y)2 = (3xy+ 1)2 -3 �(3xy+1)2 - (3x +2y)2 = 3 (1) και αν τεθεί 3xy + 1 = α , 3χ + 2y = β , η ισότητα ( 1 )

γράφεται: α2 - β2 = 3 � (α + β)( α - β) = 3 , και

επειδή α, β Ε Ζ , εύκολα βρίσκουμε ότι:

(α, β) Ε { (2, 1), (2, - 1), (-2, 1) , (-2, - 1) } . Έτσι,

καταλήγουμε στα παρακάτω συστήματα:

3xy + 1 = 2} 3xy + 1 = 2 } (Sι ) ' (S2 ) 3x + 2y = 1 3x + 2y = - 1

3xy + 1 = -2} 3xy + 1 = -2 } (S3 ) ' (S4

) 3x + 2y = 1 3x + 2y = -1

Τα συστήματα (S1 ) , (S2 ) δεν δίνουν ακέραιες

λύσεις, ενώ από τα συστήματα (S3 ) , (S4)

παίρνουμε : (χ , y) = (1, - 1) και (x , y) = (-1 , 1)

αντιστοίχως, τιμές που σε κάθε περίπτωση δίνουν

Α = 1 000(χ4 + y4 ) + 40 1 = 240 1 = (±7)4 .

Λύσεις έστειλαν επίσης, οι συνάδελφοι :

Λουκάς Χυτήρης - Κέρκυρα, Αθανάσιος Καλά­

κος - Κ. Π ατήσια, Ιωάννα ΓιαννακοποίJλου -

Λαμία, Ροδόλφος Μπόρης - Δάφνη, Γιώργος

ΕΥΚΛΕΙΔΗΣ Β' 70 τ.2!75

Page 78: Ευκλειδης Β 70

-------------- Το βήμα του Ευκλείδη --------------

Τσαπακίδης - Κυψέλη Αιτ/νίας, Βασίλειος

Νικολάκης - Θες/νίκη, Γιώργος Δεληστάθης ­

Κ. Πατήσια, Δημήτριος Κουκάκης - Κιλκίς,

Κων/νος Ηλίας - Αλιβέρι, Γιάννης Τσόπελας ­

Αμαλιάδα, Αντώνης Ιωαννίδης - Λάρισα, Μα­

ραγκουδάκης Παύλος - Πειραιάς.

ΑΣΚΗΣΗ 1 33 (τΕΥΧΟΥΣ 67 )

Δίνεται τρίγωνο ΑΒΓ με πλευρές α,β,γ και

διάμετρο του εγγεγραμμένου κύκλου του ίση με

d N δ θ , 1 1 1 1 . α ει χ ει οτι: -2

+ -2

+ -2

:::;; -2

. α β γ d

(Προτάθηκε από τον Πολιτικό Μηχανικό ΙΩΑΝ­

ΝΗ ΑΝΔΡΗ - Αθήνα )

ΛΥΣΗ: ( Από τον συνάδελφο ΒΑΓΓΕΛΗ ΜΑ­

ΡΟΥΚΟ - Αγρίνιο )

'Εστω Ε το εμβαδόν, τ η ημιπερίμετρος και ρ = � 2

η ακτίνα του εγγεγραμμένου κύκλου του τριγώνου

ΑΒΓ. Από την ανισότητα αριθμητικού - γεωμετρι­

κού μέσου και τον τύπο του Ήρωνα, έχουμε:

� = (τ - β) + (τ - γ) � .j(τ - β)(τ - γ) => 2 2

α2 Ε2 1 τ(τ - α) - � (τ - β)(τ - γ) = => - :::;; ----'-----:::-"--4 τ(τ - α) α2 4Ε2

Κυκλικά:

πρόσθεση κατά μέλη, έχουμε:

1 1 1 τ( τ - α + τ - β + τ - γ) - + - + - < ____;__-------,:'------� α2 β2 γ2 - 4Ε2

τ(3τ - 2τ) τ2 τ2 1 1 = = - = -- = - = -4Ε2 4Ε2 4ρ2τ2 4ρ2 d2

Λύσεις έστειλαν επίσης, οι συνάδελφοι :

και με

Γιώργος Αποστολόπουλος - Μεσολόγγι, Αθανά­

σιος Καλάκος - Κ. Πατήσια, Ροδόλφος Μπόρης

- Δάφνη, Γιώργος Τσαπακίδης - Κυψέλη

Αιτ/νίας, Β ασίλειος Νικολάκης - Θες/νίκη,

Αντώνης Ιωαννίδης - Λάρισα, Μαραγκουδάκης

Παύλος - Πειραιάς.

ΑΣΚΗΣΗ 134 ( ΤΕΥΧΟΥΣ 67 )

Αν για τους θετικούς αριθμούς α,β ισχύει η

ισότητα α3 + β3 = α5 + β5 , τότε να δειχθεί ότι

ισχύει: α2 + β2 :::;; 1 + αβ .

(Προτάθηκε από τον συνάδελφο ΓΕΩΡΓΙΟ ΝΙ­

ΚΗΤ ΑΚΗ - Σητεία Κρήτης )

ΛΥΣΗ 18 : ( Από τον ίδιο )

Δεχόμαστε ότι α2 + β2 > 1 + αβ . Τότε

α3 (α2 + β2 ) > α3 (1 + αβ)}; β3 (α2 + β2 ) > β3 (1 + αβ) αs + α3β2 + β3α2 + βs > α3 + β3 + α4β + αβ4 => α3β2 + β3α2 > α4β + αβ4 ::::> αβ(α2β + αβ2 ) > αβ(α3 + β3 ) => αβ( α+ β) > (α+ β)( α2 - αβ + β2 ) => αβ > α2 - αβ +β2 => (α-β)2 < 0 Άτοπο. Άρα: α2 + β2 :::;; 1 + αβ .

Λ ΥΣΗ 2 8 : (Από τον συνάδελφο ΓΙΩΡΓΟ ΤΣΑ­

ΠΑΚΙΔΗ - Κυψέλη Αιτ/νίας )

Από την ανισότητα Cauchy - Schwarz έχουμε:

[(Ν)2 + <#)2 Π<·J�)2 + (.jβ)2 J

� < ..Γσ! Fα + # .Jβ )2 => (αs + βs )(α + β) � (α3 + β3 )2 => α + β � α3 + β3 => (α + β) � (α + β)(α2 - αβ + β2 ) =>

Λύσεις έστειλαν επίσης, οι συνάδελφοι: Γιάννης Ηλιόπουλος - Καλαμάτα, Λουκάς

Χυτήρης - Κέρκυρα, Γιώργος Αποστολόπουλος

- Μεσολόγγι, Αθανάσιος Καλάκος - Κ. Πατή­

σια, Ιωάννα Γιαννακοπούλου - Λαμία, Ροδόλ­

φος Μπόρης - Δάφνη, Βασίλειος Νικολάκης ­

θες/νίκη, Γιώργος Δεληστάθης - Πατήσια,

Δημήτριος Κουκάκης - Κιλκίς, Βαγγέλης Μου­

ρούκος - Αγρίνιο, Γιάννης Τσόπελας - Αμαλιά­

δα, Αντώνης Ιωαννίδης - Λάρισα και ο πολιτι­

κός Μηχανικός Ιωάννης Ανδρής - Αθήνα,

Μαραγκουδάκης Παύλος - Πειραιάς.

ΕΥΚΛΕΙΔΗΣ ' 70 τ.2/76

Page 79: Ευκλειδης Β 70

Τα Μαθηματικά μας διασκεδάζουν Τα μαθηματικά αν και είναι επιστήμη που απαιτεί αυστηρή διατύπωση, έχουν τη μαγεία να αποσπούν το ενδιαφέρον όλων των ανθρώπων. Επινοήσεις σε προβλήματα ή ασκήσεις με κατάλληλο τρόπο διατυπωμένα εξάπτουν το πνεύμα, διεγείρουν τη φαντασία και κεντρίζουν την περιέργεια. Πρώτοι οι Αρχαίοι Έλληνες όπως ο Διόφαντος, ο Ζήνωνας κ. ά. μας δίδαξαν αυτά τα μαθηματικά. Στη στήλη αυτή θα παρουσιάζουμε θέματα τα οποία δεν απαιτούν ιδιαίτερες μαθηματικές γνώσεις αλλά μας διασκεδάζουν με την εκφώνησή τους ή τη λύση τους και είναι μια ευχάριστη και συναρπαστική ασχολία .

Επιμέλεια : Παναγιώτης Π. Χριστόπουλος

Γνωρ ίζετε ότι •:• Το 1 977 λύθηκε το πρόβλημα των 4 χρωμάτων μετά από 1 25 χρόνια από τους Apel και Haken

και είναι το πρώτο πρόβλημα στην ιστορία των μαθηματικών που λύθηκε με την βοήθεια του Ηλεκτρονικού Υπολογιστή . (τα 4 χρώματα είναι ο ελάχιστος αριθμός χρωμάτων που χρειάζονται για να χρωματίσουμε ένα χάρτη έτσι ώστε δυο γειτονικές χώρες να μην έχουν το ίδιο χρώμα.Guthrie 1 852)

•:• Το 1 742 ο Ρώσος μαθηματικός Christian Gοldbach(Γκόλντμπαχ) σε επιστολή που έστειλε στον Euler διατύπωσε την εικασία ότι «κάθε άρτιος(ζυγός) ακέραιος αριθμός μεγαλύτερος του 2 μπορεί να γραφεί ως άθροισμα δυο πρώτων αριθμών». Πρόβλημα που και σήμερα παραμένει άλυτο. Έγινε την περασμένη 1 Οετία ευρύτατα γνωστό από το βιβλίο του Α. Δοξιάδη «0 θείος Πέτρος και η εικασία του Γκόλντμπαχ».

•:• Το 1 882 με το θεώρημα του Hermite-Lindemann αποδείχθηκε μετά από 2 .300 χρόνια ότι δεν είναι δυνατός ο τετραγωνισμός του κύκλου με κανόνα και διαβήτη . Καθώς και το Δήλειο πρόβλημα (2πλασιασμός του κύβου), αλλά και η τριχοτόμηση της γωνίας.

•:• Το 1 995 ο Wiles Andrew έδωσε την τελική απόδειξη στο θεώρημα του Fermat αφού πέρασαν περισσότερα από 350 χρόνια από τότε που διατυπώθηκε. (Δηλαδή ότι η εξίσωση xn+yn=zn δεν ισχύει για κανένα ακέραιο n>2) .

•:• Το Sudoku (θέση του αριθμού), παιχνίδι από την Ιαπωνία πρωτοεμφανίστηκε στην Ευρώπη το 2004, πατέρας του παιχνιδιού ο Euler.

•:• Το 1 900 σε ένα ναυάγιο ανοικτά των Αντικυθήρων Σύμιοι σφουγγαράδες ανακάλυψαν τη λεγόμενη σήμερα «μηχανή των Αντικυθήρων», την αρχαιότερη σωζόμενη διάταξη με γρανάζια που χρονολογείται 1 50- 1 00 π.Χ. Ο μηχανισμός είναι μέρος αστρονομικού οργάνου, είναι ημερολόγιο, δείχνει γεωγραφικό μήκος και πλάτος, τη χρονιά των Ολυμπιακών αγώνων, των Νεμέων, των Πυθίων κ.ά. ακόμα ερευνάται η λειτουργία του .

Ευχαριστώ τον Χημικό κ. Δημήτρη Καρβελά, και τον Μαθηματικό κ. Αργύρη Καντεμίρη για τις εργασίες που έστειλαν. Ο κ. Καρβελάς μου έστειλε προβλήματα- αινίγματα και τις απαντήσεις τους. Ακόμη έστειλε και την εργασία του στα κριτήρια διαιρετότητας που δίνουν την απόδειξη τέτοιων προβλημάτων. Μερικά από αυτά είναι προβλήματα για να μαντεύει κανείς ηλικίες αλλά και τα μυστήρια των ακεραίων αριθμών όπως λέει. Ένα από αυτά είναι: ο μαγικ6ς κίJκλος, που δημοσιεύουμε.

Ο κ. Καντεμίρης έστειλε το πρόβλημα: ο Δ ιευθυντής του ορυχε ίου, που επίσης δημοσιεύουμε. Το πρόβλημα αυτό μπορεί να πάρει οχτώ διαφορετικές μορφές, μια από αυτές είναι: η προσφορά του

ΕΥΚΛΕΙΔΗΣ Β' 70 τ.2/77

Page 80: Ευκλειδης Β 70

------------- Τα Μαθη ματικά μας διασκεδάζουν ------------­

Χαλίφη, που έχουμε_δημοσιεύσει στο τεύχος 63 το 2007. Εδώ θα επισημάνουμε, ότι για κάποια αινίγματα δίνουμε μόνο την απάντηση και όχι την μαθηματική απόδειξη, επειδή αυτή συνήθως στηρίζεται στη θεωρία αριθμών (κριτήρια διαιρετότητας) ή άλλες θεωρίες και δεν είναι στο πνεύμα και για το χώρο της στήλης.

Ο μαγικός κύκλος Ο Δημήτρης λέει στην Εύα πολλαπλασίασε τον αριθμό 37 με ένα αριθμό του οποίου τα ψηφία να

έχουν άθροισμα 1 8 και στο γινόμενο πρόσθεσε την ηλικία σου.(Εστω ότι η Εύα μετά από αυτό βρήκε τον αριθμό Χ).

Εύα διάλεξε ένα ψηφίο του μαγικού κύκλου όποιο θέλεις και διάβασε στη συνέχεια ακλουθώντας δεξιόστροφη ή aριστερόστροφη φορά τον εννεαψήφιο αριθμό που προκύπτει. Πρόσθεσέ τον στον Χ που βρήκες ποιο πριν και πες μου το αποτέλεσμα. Εύα: λέει το αποτέλεσμα

Δημήτρης: Εύα τώρα ξέρω την ηλικία σου. Εύα: Σοβαρά πώς;

Ο Διευθυντής του ορυχείου

; : · .- .-:.::....-- · ..

.. - -=- - · +. • . - · • ' ' "

- �π rrι1 . �� - :+._�.,:;����: �; 1:;: i�� �. ? ��--,.φ1- rr ι - · . : · ; .ι- .. · • .. . "'�-�--!' -- • � .... Τρείς δημοσιογράφοι πήγαν σε ένα ορυχείο στη Νότια Αφρική για να πάρουν συνέντευξη από το

Διευθυντή του ορυχείου. Όταν τελείωσε η συνέντευξη ο Διευθυντής ζήτησε από του δημοσιογράφους να του λύσουν το εξής πρόβλημα: <<Στο ορυχείο δουλεύουν λευκοί και μαύροι εργάτες, τώρα θα έρθουν εδώ τρείς που από το κάρβουνο φαίνονται όλοι μαύροι. Έχετε δικαίωμα να τους κάνετε από μια μόνο ερώτηση για να βρείτε από τις απαντήσεις τους αν είναι μαύροι ή λευκοί, να ξέρετε μόνο ότι οι μαύροι λένε πάντα ψέματα ενώ οι λευκοί πάντα την αλήθεια». Έλυσαν το πρόβλημα οι δημοσιογράφοι;

ΕΥΚΛΕΙΔΗΣ Β' 70 τ.2/78

Page 81: Ευκλειδης Β 70

------------- Τα Μαθη ματικά μας διασκεδάζουν -------------

Ο Γρίφος του Einstein

Υπάρχουν 5 σπίτια πέντε διαφορετικών χρωμάτων. Σε κάθε ένα σπίτι ζει ένας άνθρωπος διαφορετικής εθνότητας. Οι πέντε ιδιοκτήτες πίνουν ένα συγκεκριμένο είδος ποτού, καπνίζουν μία συγκεκριμένη μάρκα τσιγάρων και έχουν ένα συγκεκριμένο κατοικίδιο. Όλοι έχουν μεταξύ τους διαφορετικά κατοικίδια, διαφορετικές μάρκες τσιγάρων, διαφορετικά είδη ποτών και σπίτι με διαφορετικό χρώμα. Η ερώτηση είναι: «Ποιος έχει το ψάρι;».

Στοιχεία του προβλή ματος: Ο Άγγλος μένει στο κόκκινο σπίτι. Ο Σουηδός έχει ένα σκύλο. Ο Δανός πίνει τσάι. Το πράσινο σπίτι είναι αριστερά από το άσπρο σπίτι. Ο ιδιοκτήτης του πράσινου σπιτιού πίνει καφέ. Αυτός που καπνίζει Pall Mall εκτρέφει πουλιά. Ο ιδιοκτήτης του πράσινου σπιτιού καπνίζει Dunhill . Αυτός που μένει στο μεσαίο σπίτι πίνει γάλα. Ο Νορβηγός μένει στο πρώτο σπίτι. Αυτός που καπνίζει Blends μένει δίπλα σε αυτόν που έχει γάτες. Αυτός που έχει το άλογο μένει δίπλα σε αυτόν που καπνίζει Dunhill . Ο ιδιοκτήτης που καπνίζει Blue Esters πίνει μπίρα. Ο Γερμανός καπνίζει Prince. Ο Νορβηγός μένει δίπλα στο μπλε σπίτι. Αυτός που καπνίζει Blends έχει ένα γείτονα που πίνει νερό. (Ο Είηsteίη_όταν τον προηγούμενο αιώνα έγραψε το Γρίφο είπε ότι μόνο το 2% των ανθρώπων μπορεί να απαντήσει. Τι λέτε;)

Η Β α ρ κά δ α τέσσερα ζευγάρια πάνε εκδρομή . Ένα ποτάμι στο δρόμο τους δεν έχει γεφύρι και πρέπει να το περάσουν με μια βάρκα 2 ατόμων. Καμία γυναίκα δεν πρέπει να μείνει κάπου με άλλους άνδρες όταν δεν είναι εκεί

ΕΥΚΛΕΙΔΗΣ Β' 70 τ.2179

Page 82: Ευκλειδης Β 70

------------- Τα Μαθη ματικά μας διασκεδάζουν ------------­

και ο άνδρας της. Το ποτάμι είναι πλωτό και στη μέση υπάρχει βράχος που μπορεί να χρησιμοποιηθεί ως ενδιάμεσος σταθμός. Μπορείτε να τους βοηθήσετε;

Τα βαρέλια Τρείς φίλοι θέλουν να μοιράσουν 2 1 βαρέλια κρασί 7 γεμάτα, 7 άδεια και 7 μισογεμάτα( ή μισοάδεια), ώστε να πάρουν την ίδια ποσότητα κρασιού και βαρελιών. Μπορείτε να τους βοηθήσετε;

Το κρασί Δύο φίλοι έχουν μια νταμιτζάνα με 1 2 λίτρα κρασί . Πώς θα την μοιράσουν αν διαθέτουν μόνο 2 άδεια δοχεία 7 λίτρων και 5 λίτρων;

ΕΥΚΛΕΙΔΗΣ Β' 70 τ.2/80

Page 83: Ευκλειδης Β 70

Ο μαγικός κύκλος

' ' ' ·-· · --/ .: -

Ο Δημήτρης χώρισε τον αριθμό που του ανακοίνωσε η Εύα σε τριψήφια τμήματα αρχίζοντας από δεξιά προς τα αριστερά και προσθέτει αυτά τα τριψήφια τμήματα. Εάν το άθροισμα είναι αριθμός με περισσότερα από τρία ψηφία, επαναλαμβάνει την ίδια διαδικασία μέχρις ότου ο αριθμός να είναι τριψήφιος. Τον τριψήφιο αυτό αριθμό τον διαιρεί δια 1 1 1 και το υπόλοιπο της διαίρεσης αυτής είναι η ηλικία της . . . Εύας ! π . χ. έστω ότι η Εύα είναι 23 ετών και αριθμός 65430 που το άθροισμα των ψηφίων του είναι 1 8 . Άρα Χ= 37.65430+23=2420933 . Αν από τον μαγικό κύκλο διάλεξε τον αριθμό 6 και με δεξιόστροφη φορά έχει τον 675724488 τότε 2420933 + 675724488 = 678 1 4542 1 . Ανακοινώνει αυτόν στο Δημήτρη και αυτό(; 678+ 1 45+42 1 = 1 244, 1 +244=245 , 245 : 1 1 1 δίνει υπόλοιπο . . . 23 !

Ο Διευθυντής του ορυχείου Οι δημοσιογράφοι συσκέπτονται και ρωτάνε τον πρώτο εργάτη : Είσαι μαύρος ή άσπρος; Αυτός απαντά σε μια γλώσσα μου δεν κατάλαβαν τίποτα. Ρωτάνε το δεύτερο εργάτη : τι μας είπε ο συνάδελφός σου ότι είναι μαύρος ή άσπρος; Και αυτός απαντά: σας είπε ότι είναι άσπρος. Ρωτάνε και τον τρίτο: Οι δύο προηγούμενοι συνάδελφοί σου μας είπαν την αλήθεια; Και αυτός απαντά: ναι και οι δυο είπαν αλήθεια. Τότε οι δημοσιογράφοι συσκέφτηκαν ως εξής: Ο πρώτος αν είναι άσπρος θα πει αλήθεια «είμαι άσπρος» αν είναι μαύρος θα πει ψέματα «είμαι άσπρος» άρα σε κάθε περίπτωση ο πρώτος είπε «είμαι άσπρος» . Δηλαδή ο δεύτερος μας είπε αλήθεια άρα ο δεύτερος είναι άσπρος. Ο τρίτος μας είπε «ναι και οι δυο είπαν αλήθεια» που αυτό είναι αλήθεια για τον δεύτερο άρα είναι αλήθεια και για τον πρώτο. Δηλαδή και οι τρείς είναι λευκοί. (Λύστε εσείς το πρόβλημα με: οι λευκοί να λένε ψέματα και οι μαύροι την αλήθεια).

1

Κατοικ σκύλος Άρα το Ψ ΑΡΙ είναι στο πρώτο σπίτι που μένει ο Νορβηγός.

Η Βαρ κάδα Τα ζευγάρια είναι Α 1 Γ l , Α2Γ2, Α3Γ3, Α4Γ4 . Παίρνει την βάρκα η Γ 1 και μεταφέρει μία -μία όλες τις γυναίκες στο βράχο. Γυρνά ξανά στην όχθη Α παίρνει τον άνδρα της Α 1 και τώρα αυτός αφού αφήσει την γυναίκα του στο βράχο γυρνά στην όχθη Α και μεταφέρει έναν-έναν τους άνδρες στην όχθη Β . Ύστερα πηγαίνει στο βράχο παίρνει τη γυναίκα του και πηγαίνουν στη Β όχθη . Δίνει την βάρκα στον Α2 που παίρνει την Γ2 από το βράχο και πηγαίνουν στην Α όχθη . Εκεί μένει ο Α2 και φεύγει με την βάρκα η Γ2 για να μεταφέρει τις Γ3 και Γ4 από το βράχο στη Β όχθη ου τις περιμένουν οι άνδρες τους. Τέλος επιστρέφει η Γ2 στην Α όχθη παίρνει και τον άνδρα της (Α2) και πηγαίνουν στην Β όχθη που τους περιμένουν τα 3 άλλα ζευγάρια. Τα βαρέλια

I Φίλοι

I Σύνολο Το κρασί 1 2 , 7 , 7 , 2 , 2 , 9 , 4, 4, 1 1 , 1 1 , 6, 6 7 , Ο, 5 , 5 , 7 , 3 , 3 , 7 , Ο, 1 , 1 , 6 5 . 5 , Ο , 5 , 3 , Ο , 5 , 1 1 , 1 , Ο , 5 , Ο

11 Γεμάτο 11 Άδειο I Μισό � 1 Ι 1 5

3 3 3 3 7 7 7

Page 84: Ευκλειδης Β 70

από r ιs εκδόσε ιs <<ΔΙΟ ΦΑΝΤΟΣ>>

Γιώργος Μ. Μιχαηλίδης

ΜΑθΗΜΑΤΙΚΑ \ ΓΛΥΚΕΙΟΥ

ΤΟΜΟΣ Α' θετική - Τεχνολογική Κατεύθυνση

• Ι Ιαράγωγος • Συνέχεια

• Όριο

8 (·)ι-οιρία - �χιiλια • Λυμένα llαραl'ιrίγματα

8 :\lrflnl!ιoλι•·ιiα - ΟμαiΊοπιιίηuη 8 ΛωιήΙJεις για λιiηη

8 Εριtιτψιrις Κιιωνιiηοη; 8 Θέματα ·ιιιι διαγωνiπμαtιι

< Ν

..J < ce Ζ (.,.) < ­w .... c:ι <

:Ξ w w :Ξ .... w < ..J ::ΞΞ α::ι σ α: Ω..

••

Γιώργος Μ. Μιχαηλίδης

ΜΑθΗΜΑΤΙΚΑ Γ ΛΥΚΕΙΟΥ

ΤΟΜΟΣ !f θετική - Τεχνολογική Κατεύθυνση

ΟΛΟΚΛΗΡΏΜΑΤΑ

8 Θεω(Ιία • Μ εθΟΟολογία

• Παρατηρήσει-; - Σχιiλια 8 Λυμένα Ιlα(Ιαδε(γματα

• Ευωτήοεις Κατανόηιιης • Ασκήσεις - θέματα

Γιώργος Μ. Μιχαηλίδης

ΜΑθΗΜΑΤΙΚΑ ΓΛVΚΕΙΟΥ

θετική - Τεχνολογική Κατεύθυνση

eξs-ι;άαsις aooe - aooe

πιaανι2 etματα

Γιώργος Μ. Μιχαηλ!δης Octavian Ν. Stanasila

(!'lui. Urιν. - -)

Γιώργος Μ. Μιχαηλ!δης

cc (.;) ϊ= cc cr: :E C> w -' ι- 1-cc < :Ξ !:: cc ::::! t,;Ι ι:Ω - cc 1- ι::σ c.? σ i= cr: cc a.. ι- cc c.? -

ι a:: _ σ - w ι- ::ι: � �-:::ι .....

Octavian Ν. Stanasila (!'lui. UrW -JIJGaiiUthamt)

ΔιοΦΑΝΤΟΣ ΕΚΔΟΣΕΙΣ